questions Flashcards
ovarian tumor - histological variant: choriocarcinoma
malignant cytotrophoblast/synciotrophoblast
biggest risk factor to type 1 endometrial cancer?
exposure to unopposed estrogen b/c type 1 is estrogen-dependent cancer
type 1 or 2 endometrial cancer has worse prognosis?
type 2, especially clear cell and papillary serous as they are considered high grade
ovarian tumor - histological variant: clear cell carcinoma
hobnail cells
ovarian tumor - histological variant: dysgerminoma
sheets of lymphocytes/germ cells
ovarian tumor - histological variant: endodermal sinus/yolk sac tumor
schiller-duval bodies
ovarian tumor - histological variant: granulosa cell tumor
call-exner bodies
ovarian tumor - histological variant: serous tumor
psammoma bodies
ovarian tumor - histological variant: brennor tumor
walthard nest; bladder epithelium
presentation of GU rhabdomyosarcoma
arises from any mesenchymal tissues of any body site. can cause vag bleeding (most common sx), vag discharge, vaginal mass, urinary freq or obstruction, bowel issues
ovarian tumor - histological variant: immature teratoma
immature neuro-epithelium
ovarian tumor - histological variant: embryonal carcinoma
malignant cytotrophoblast/synciotrophoblast
ovarian tumor - histological variant: krukenberg tumor
signet cells
atypical ductal hyperplasia - relative risk of breast cancer
4.5-5x risk
dx of post-molar GTN made by either:
a) hCG plateau (four values w/in +/- 10%) over 4 wks,
or
b) hCG rise of 10% across 3 values over 2 wks
needs CXR for staging and determining high vs low risk dz
greatest risk factor for breast cancer
age
most common presentation of fallopian tube cancer
postmenopausal vaginal bleeding w/ watery vaginal discharge (50% of pts). hydrops tubae profluens = watery vagina ldischarge can be clear/yellow/copious amt. usually dx’ed late during w/u for r/o endometrial CA w/ PMB
characteristic of paget’s dz of nipple/breast
usually direct extension of high grade ductal carcinoma in-situ, though invasive carcinoma may be present. classically w/ crusting scaly rash emanating from nipple or areola. need biopsy
uterine papillary serous carcinoma histology
psammoma bodies
characteristic of HPV-independent vulvar cancers
inflammatory disorders of vulva can cause squamous hyperplasia/vulvar dystrophy which can lead to this
most common side effect of radiation therapy and how it presents
atrophy of epithelium, causing diarrhea, acute cystitis, vaginal mucositis, skin erythema
risk of endometrial CA based on pre-surgical biopsy findings
simple hyperplasia w/o atypia - 1%
simple hyperplasia w/ atypia - 3%
complex hyperplasia w/o atypia - 8 %
complex hyperplasia w/ atypia- 29%
endometrial intraepithelial neoplasia - 40%
US findings on partial molar pregnancy
focal cystic changes in placenta,
ratio of transverse to AP dimension of GS >1.5
HPV E6 and E7 oncoprotein action
small zinc binding protein that bind to cell cycle regulatory protein p53,
affects Rb protein
mature teratoma - most likely cancerous component
squamous cancer cell transform w/ <2% chance usually in postmenopausal
most commonly found ovarian cancer a/w endometriosis
clear cell adenocarcinoma (hobnail cells) / type of epithelial ovarian cancer
common presentation of complete molar pregnancy
only paternal DNA (no fetal parts develp), higher hCG lvls, a/w lutein cysts, medical complications
spread of vaginal cancer
distal 1/3 of vagina - drains to superficial and deep inguinal nodes then femoral nodes
upper vagina - drains to external/internal iliac (pelvic) nodes
in what pattern and nature does ovarian cancer most commonly spread?
they EXFOLIATE malignant cells into peritoneal cavity = transcoelomic spread. they follow normal circulation (clockwise) of peritoneal fluid and implant in right pericolic gutter and undersurface of right hemi-diaphragm. the omentum and all other surfaces of the peritoneum are at risk of spread via this mechanism
treatment for placental site trophoblastic tumor?
poorly responsive to chemo. hence needs hyst also b/c mets are rare
most common site for mets in GTN
lungs
risk of ovarian cancer for BRCA 1/2 carriers by age of 70
BRCA 1 - 40%
BRCA 2 - 15%
HPV with most carcinogenic potential
HPV 16 then 18
tx for advanced cervical CA
whole pelvic external beam radiation therapy with brachytherapy and concurrent weekly cisplatin therapy
two types of vulvar cancer
type 1 - age 35-65, a/w cervical neoplasia , h/o STD, HPV, VIN, h/o warts, h/o smoking
type 2 - 55-85yo, a/w vulvar inflammation, lichen sclerosis, squamous cell hyperplasia
annual MRI for breast screening recommended for:
1st degree relative of breast cancer genetic mutation carrier, but untested
20% lifetime risk of breast cancer
radiation therapy to chest between 10-30yo
age 25-29yo and know BRCA pathogenic variant
inflammatory breast cancer - dx criteria
rapid onset of erythema, edema, peau de orange appearance, warm, with or w/o palpable mass
duration of history no more than 6 months
erythema occupying at least 1/3 of breast
pathologic confirmation invasive carcinoma
main tx for DCIS of breast
lumpectomy with or without radiation or tamoxifen
lifetime risk factor for ovarian cancer
1.3% = 1 of 75
signs of partial molar pregnancy. how does it happen?
presence of fetal parts, most common karyotype 69 XXX.
happens when 23X containing haploid egg gets fertilized by 2 sperms (23 X or 23 Y). results in trisomy 69 XXX, 69 XYY, 69 XXY
signs of complete molar pregnancy. how does it happen?
no fetal parts seen.
inactivated 23 haploid egg gets fertilized by 23X sperm which then replicates forming a 46XX diploid of full PATERNAL origin. it is also possible to be fertilized by 2 separates sperm and 46 XY diploid is possible, all from PATERNAL origin
complete molar pregnancy presentation and mgmt
early preg vag bleeding, snowstorm appearance on US w/o fetal parts
preop: ABO, baseline labs, coags, CXR, bHCG, thyroid fxn if hyperT suspected
d&c
postop: trend bHCG to r/o GTN
most common GYN Cancer
cervical cancer
key difference btwn uterine sarcoma and endometrial carcinoma
size of tumor
> 5cm is stage 1B
what type of antidepressant class should be avoided w/ tamoxifen
SSRIs - may theoretically reduce effectiveness of tamoxifen. SNRIs are fine
risk of GTN after d&c w/ complete molar
15% . may be as high as 35% in high risk dz
having multipartiy is increased risk for cervical CA?
bc there is higher chance of having had multiple sexual partners, more likelihood of HPV exposure/STD exposure
how does mgmt change if stage 1A or 1B ovarian cancer ruptures inside belly intraop
upstaged to at least stage 1C, which will likely now require chemo
desires fertility, complex hyperplasia w/o atypia. mgmt?
progestin like megace 80mg qD for 3 months, repeat EMB then reassess effectiveness of current therapy
difference in step between modified rad hyst vs rad hyst
resection of parametrial tissue medial to ureter for mod rad hyst
rad hyst has ureters completely dissected down to insertion of bladder. all parametrial tissue and all other ligaments are resected at their attached points
CA125 proteins can be found in what tissues
genital tract epithelium, pancreas, gallbladder, stomach, kidney, lung, breast, heart.
inflammation of any of these peritoneal surfaces may lead to elev CA125
no matter how ovarian cancer recurs, second line chemo is
NOT curative
risk reducing surgeries for BRCA carriers
bilat mastectomy - risk reduction of 85-100% in breast cancer
BSO - risk reduction of 80% of ov cancer, 68% of all cause mortality
most important risk factor for epithelial ov cancer
age
what % of mature teratoma becomes SCC?
1%
simple ovarian cysts <10cm are almost always benign and can be watched.
postmenopausal pt CA125 cutoff is around 35
no cutoff for premenopausal CA125 though
if there is a fixed mass, whether pre or post meno, refer to gyn onc
adnexal mass US BENIGN features
unilocular cyst
solid components with largest being <=7mm largest diameter
acoustic shadows
smooth multilocular contour
no detectable blood flow on doppler
adnexal mass US MALIGNANT features
irregular solid
ascites
>=4 papillary structures
irregular multilocular solid tumor w/ greatest diameter of >=10cm
very high color content on color doppler
cervical cancer is still clinically staged, with recent 2018 FIGO cervical CA staging criteria allowing _ with physical exam
any imaging modality and/or pathology findings
ov ca risk with BRCA 1 and 2 by age 70
1 - 39-46%
2- 10-27%
long term use >5yrs of cOCPs has increase risk of
cervical cancer
what age is appropriate for risk reducing BSO for BRCA1 and 2
35-40 for 1
40-45 for 2
diff btwn ileus and SBO
ileus - transient, dilated loops of bowel on KUB, pt may still pass flatus, decreased/absent bowel sounds
SBO - air fluid lvls or areas of collapsed bowel distal to obstruciton, often n/v, high-pitched/tinkering bowel sounds
serious complication during suction d&c
uterine perf, especially cervical perf — high risk of bowel/bladder/vasculature injury. may needs hsc/cysto/DRE/lsc/ex lap
why does UOP decreased during robotic surgery/deep t-berg position/pneumo?
release of renin, aldosterone, ADH –> reduced GFR –> oliguria
other physiologic changes w/ robot/t-berg –> decreased CO, acidemia, oliguria, difficulty with ventilation
a fever in a pt 12hrs postop surgery w/o any complaints otherwise – mgmt?
expt mgmt
A fever in the first 24 hours after surgery may represent a noninfectious pathway (atelectasis, hypersensitivity rxn, pyrogenic rxn to tissue trauma). In the absence of complications or definitive infection, no additional workup is needed. If the fever persists beyond the 24-hour period, additional testing may be necessary to identify both infectious and noninfectious causes
A patient presents for her 1 week postoperative follow-up following an uncomplicated cesarean delivery. Her incision and surrounding skin shows significant erythema, warmth, induration with an area of fluctuance, and with a moderate amount of purulent drainage from the left apex of the incision, which has separated superficially 1–2 cm. The fascia appears intact. Her temperature is 38.6°C (101.5°F). Which of the following is the MOST appropriate management?
Severe wound infections involving purulent drainage and associated surrounding soft tissue infection require incision and drainage with debridement, wound culture, initiation of antibiotics, and daily wound care. Delayed closure is not contraindicated in a wound infection, provided the infection has been cleared and healthy granulation tissue has commenced.
managing preop sugar for diabetic
Preoperatively, it is important to both ensure adequate glucose control and prevent hypoglycemia, ketosis, electrolyte or fluid imbalance, or marked hyperglycemia. Diabetes mellitus is also associated with increased risk of perioperative infection and postoperative cardiovascular morbidity and mortality. Although a preoperative target glucose level has not been precisely established, glucose generally should be below 200 mg/dL preoperatively and intraoperatively, and postoperatively glucose should be between 140 and 180 mg/dL. In patients with high glucose levels preoperatively, cancellation should be considered, especially with glucose readings above 400 mg/dL.
With regard to medications:
Patients taking long-acting insulin at night or in the morning should take one-half to two-thirds of the usual dose; with intermediate-acting insulin taken twice daily, the normal dose should be given the night before and long-acting insulin should be taken at one-half to two-thirds of the usual dose.
Short-acting insulin should be held on the morning of surgery because it increases the risk of hypoglycemia.
Sulfonylureas such as glyburide increase the risk of hypoglycemia and should be held on the morning of surgery.
Thiazolidinediones may worsen fluid retention and can lead to heart failure and should be held on the morning of surgery.
Metformin increases the risk of renal hypoperfusion, lactic acidosis, and tissue hypoxia and should be held on the morning of surgery.
Sodium-glucose cotransporter 2 inhibitors increase the risk of hypovolemia and should be held on the morning of surgery.
is bowel prep necessary for lsc gyne surgery?
Bowel preparation has shown no statistical difference in the incidence or morbidity of bowel injury during gynecological surgery
diff btwn topic hemostatic agents (everything else) vs biologic active agents (floseal,tisseel)
Topical hemostatic agents are a useful adjunct to manage bleeding during surgery. Hemostatic agents can be physical agents or biologically active agents. Physical hemostatic agents provide a matrix to concentrate hemostatic factors, absorb water and allow thrombi to easily form. They require an intact coagulation cascade. Physical agents are best used in situations with minimal bleeding/oozing and become less effective with brisk or heavy bleeding.
Biologically active agents help augment hemostasis. These are more useful with brisk bleeding and can be used in the setting of coagulopathy. The main disadvantage of biologically active agents is that they are not always readily available and they can be expensive.
most common sx a/w SBO
periumbilical pain
PE after gyn surgery. how long to be on coumadin?
Patients with a pulmonary embolus (PE) or deep venous thrombosis (DVT) provoked by surgery or other transient risk factors should be treated with anticoagulation for 3 months.
Which of the following types of hysterectomies has the GREATEST risk for ureteral injury?
lsc hyst
US appearance of endometrioma
unilocular, hypoechoic cyst with diffuse low-level echoes and ground-glass appearance.
symptoms consistent with a nerve injury likely secondary to compression from improperly placed retractors over the psoas muscle. Which of the following symptoms is MOST consistent with this injury?
The most common setting for a femoral nerve injury is during pelvic surgery, such as with an abdominal hysterectomy due to the use of deep and lateral retractors. The femoral nerve emerges from the border of the psoas muscle and can be compressed against the pelvic side wall when retractors are improperly positioned.
Femoral nerve injuries result in both sensory and motor deficits, such as anesthesia of the anterior and medial thigh and weakness in the quadriceps and iliopsoas muscles. Consequently, patients with a femoral nerve injury are unable to flex the hip and extend the knee.
tx for septic abortion - abx
Antibiotic Treatment for Septic Abortion
Triple therapy is necessary to cover the following 3 groups:
-gram-positive anaerobes and aerobes
-resistant gram-negative aerobes
-gram-negative anaerobes
ex) amp/gent/clinda
Infection is often polymicrobial. Common organisms include E. coli, GBS, anaerobic streptococci, Bacteroides, and staphylococci.
Which of the following comorbidities would place a patient at the HIGHEST risk in a patient undergoing laparoscopic surgery?
COPD is nonreversible. It is a chronic disease leading to scarring in the lungs and decreasing a patient’s ability to exchange oxygen and carbon dioxide. This often leads to baseline respiratory acidosis caused by hypercarbia. When performing laparoscopic surgery, carbon dioxide is used to insufflate the peritoneum and is rapidly absorbed and enters the bloodstream. When this occurs, the patient becomes more acidotic due to hypercarbia. For most patients without underlying respiratory disease, this can be managed by increasing the respiratory rate. However, for patients with severe COPD, the existing damage to the lung parenchyma inhibits carbon dioxide exchange out of the lungs, increasing the risk of complications during laparoscopic surgery. Thus, severe COPD places this patient at the highest risk of morbidity or mortality.
Which of the following antibiotics is required preoperatively prior to a hysterectomy?
Cefazolin, a first generation cephalosporin, can be used alone as an antibiotic prophylaxis for patients undergoing a hysterectomy
bicornuate uterus
uterus in which the fundus is indented >1 cm. This anomaly results from only partial fusion of the Müllerian ducts.
Müllerian agenesis (referred to as Mayer–Rokitansky–Küster–Hauser syndrome),
congenital absence of the vagina with variable uterine development.
septate uterus
normal external surface but has 2 endometrial cavities, in contrast to a bicornuate uterus which has an indented fundus and 2 endometrial cavities. A septate uterus results from a defect in canalization or resorption of the midline septum between the 2 Müllerian ducts.
unicornuate uterus
asymmetric lateral fusion defect. One Müllerian duct develops normally, with a normal fallopian tube and cervix, but the other Müllerian duct fails to develop and can have various configurations
Uterine didelphys, or double uterus
occurs when the 2 Müllerian ducts fail to fuse. This defect has the appearance of 2 uteri and 2 cervixes. Uterine didelphys has clinical significance in that it is associated with spontaneous abortion rates of 32% and preterm birth rates of 28%. Additionally, there also appears to be an increased rate of fetal growth restriction.
Necrotizing fasciitis may be divided into two microbiologic categories
polymicrobial (type I) and monomicrobial infection (type II). Polymicrobial (type I) necrotizing infection is caused by aerobic and anaerobic bacteria.
Monomicrobial (type II) necrotizing infection is usually caused by GAS or other beta-hemolytic streptococci. Infection may also occur as a result of Staphylococcus aureus
diagnostic imaging of choice for a ureterovaginal fistula
Intravenous pyelography (preferred) and retrograde pyelography
what is the doulbe dye test and how does it work
uses methylene blue (or sometimes indigo carmine) along with Pyridium. The patient ingests the Pyridium and the Methylene blue is instilled into the bladder using a urethral catheter. The Pyridium will turn the urine orange and the methylene blue will turn the urine blue. A tampon is then placed into the vagina. If the tampon turns blue, suspect a vesicovaginal fistula. If the tampon turns orange, suspect a ureterovaginal fistula. If the tampon turns both colors, suspect a combination of a vesicovaginal and a ureterovaginal fistula. however Oral pyridium alone is not sufficient enough to test for a ureterovaginal fistula to make formal dx
if you suspect UV fistula, you should consider confirming on IV pyelography or retrograde pyelography
Which of the following is the next BEST step in the management of a hemodynamically unstable patient with an incomplete abortion?
suction d&c. also consider starting transfusion as you roll out if they’re very unstable
lsc trochar placement of non obese vs obese
In a non-obese patient inserting the trocar at a 45 degree angle ensures that you are avoiding major vessels. At this angle, the abdominal wall thickness varies from 2 to 3 cm and distance to the major vessels averages 6 to 10 cm. Always remember: Patients should NOT be in Trendelenberg position during initial trocar placement.
A 90 degree angle should never be used when entering the abdomen with a trocar; entering the abdomen at a 90 degree angle puts the surgeon at a largely increased risk of hitting the aorta with the trocar, as the aorta is generally less than 4 cm from the umbilical stalk.
triangular incision pain above pfannenstiel incision after CS
The iliohypogastric nerves run along the lateral border of the rectus muscles bilaterally, but there are terminal branches that can be affected even if there is not a full transection of the nerve at the lateral border of the rectus muscle. Injury to iliohypogastric or ilioinguinal nerves may occur from entrapment of the nerves by sutures at the lateral poles of transverse fascial incisions, direct nerve trauma, neuroma formation, or from neural constriction caused by the normal scarring/healing process. Both of these nerves function to provide sensation in the suprapubic area. Damage to these nerves can cause numbness and pain.
The characteristic triad of symptoms are:
sharp, burning, lancinating pain radiating from the incision to the suprapubic area, labia/scrotum, or thigh
paresthesias over these areas
pain relief after infiltration with a local anesthetic.
3 days of abx tx for TOA do not improve pt sx (not necessarily worsening). what next?
If after 48–72 hours of treatment with antibiotics, patients with a TOA still do not improve, CT- or US-guided percutaneous drainage of the abscess should be arranged. Whereas, if patients worsen on IV antibiotics, surgery should promptly be performed.
ovarian torsion US findings
A rounded and enlarged ovary compared with the contralateral ovary
Decreased or absent Doppler flow within the ovary
Heterogeneous appearance of the ovarian stroma
Multiple small peripheral follicles
coild vascular pedicle “whirlpool sign”
concerns for nec fascitis after CS over incision site and compromised fascial closure, sepsis. mgmt?
This patient has evidence of necrotizing fasciitis with sepsis. This is an emergency, as delay of surgery by as much as 24 hours increases mortality by a factor of 9. This patient also needs IV antibiotics that cover for a variety of organisms. Currently vancomycin, a carbapenem and clindamycin are used. Once the wound is debrided, it must be closed. Primary closure is simple closure with suture or staples and is appropriate for clean and uninfected wounds. Secondary closure is closure with wet to dry dressings or a wound vac, and is used for wound infections. Tertiary closure involves leaving a wound open to see if infection develops and if it doesn’t, closure is done with suture or staples.
Since this patient has an active wound infection, it should be closed by secondary intention. Therefore the best answer is antibiotics, emergent wound debridement with secondary closure.
Which of the following is the BEST marker of nutritional status that will predict poor postoperative outcomes in an elderly patient?
Low serum albumin (< 3.0 g/dL in the absence of hepatic or renal failure) is the best predictor of nutritional status. It is a marker of a negative catabolic state and is the best predictor of poor outcome after surgery.
Signs of Severe Malnutrition in the Elderly
Risk Factor Parameter
BMI BMI < 18.5
Serum albumin Serum albumin < 3.0 g/dL in the absence of hepatic or renal failure
Unintentional weight loss > 10–15% within the most recent 6 months
Which of the following CORRECTLY lists the layers of the bladder from the inside to the outside?
The bladder layers in order from the inside of the bladder to the outside of the bladder are:
transitional epithelium (urothelium)
lamina propria
submucosa
detrusor muscle (muscularis propria)
adventitia (serosa)
Which of the following is the MOST appropriate initial step in management of an immediate postoperative fever?
Immediate postoperative fever (38.0°C or 100.4°F) is common and, when present in the first 24 hours, is most likely of benign etiology – continue observation and routine postoperative care.
Common causes of immediate postoperative fever include:
reaction to medications
blood transfusion reaction
trauma and inflammation from the operation itself
preexisting infections
malignant hyperthermia (occasionally)
Most commonly, such immediate fevers will abate spontaneously or after removing the offending insult, if identified. Thus, a workup for infection, DVT, abscess, or septic thrombophlebitis is not initially necessary or cost-effective at this point.
Persistent postoperative fever, or one that begins after 1–2 days postoperatively, would be more concerning for an infectious process, soft-tissue abscess formation, or vascular etiology, indicating the need for further workup.
androgen insensitivity syndrome
Androgen insensitivity is an absence of the X-chromosome gene responsible for testosterone receptor function. It has an estimated incidence of 1/60,000 and these individuals have an XY karyotype and normally functioning male gonads that produce normal male levels of testosterone. Because of the lack of receptors on target organs, there is lack of male differentiation of the external and internal genitalia. Because Müllerian duct regression is a product of anti-Müllerian hormone secretion, there are no female internal genitalia either. Breasts develop because of the increased aromatization of androgens to estrogens in the periphery.
Testes that are intra-abdominal or in the inguinal canal have an increased risk of developing malignancy (gonadoblastoma), with an incidence reported to be approximately 20%. However, these malignancies rarely occur before the age of 20 and, therefore, it is usually recommended that the gonads be left in place until after puberty is completed to allow full breast development and full epiphyseal closure to take place.
how to plan fascial closure for high risk unhealthy pts doing open gyne surgery
A key point of this topic is fascial incision healing. According to TeLinde’s Operative Gynecology, fascial incisions heal rather slowly, at a rate of 10%, 25%, 40%, and 50% of their original preoperative strength by weeks 1, 2, 3, and 4, respectively. The majority of fascial dehiscences occur between postoperative day 2 and 12; however, it has been reported up to day 18.
Thus, in most patients, 2 weeks is considered an adequate time for fascia to regain enough strength to resist average movement and stresses. Some high-risk patients (i.e., the patient in this question) may require 3 weeks. PDS and polyglyconate retain 80% of their initial tensile strength after 2 weeks and degrade slowly thereafter, which is why PDS is oftentimes the suture of choice for more complicated midline vertical fascial closures.
17yo w/o uterus. why order karyotype?
The strongest reason to order a karyotype in the described patient is to distinguish between androgen insensitivity syndrome (AIS) and Mayer–Rokitansky–Küster–Hauser syndrome (Müllerian agenesis). This is because MRKH and AIS have a different karyotypes: 46,XX and 46,XY respectively. both have absence of uterus
EMB is a low-risk and convenient procedure that can be completed without anesthesia in the office; however, the limitation of the procedure is that approximately 0.9% of biopsies will fail to detect a cancer that is present within the endometrium. That said, EMB is the most cost-effective method.
High yield statistics to keep in mind:
EMB samples approximately 5–15% of the endometrial cavity.
About 30–40% of patients noted to have complex hyperplasia have concomitant endometrial cancer.
In a patient at high risk for fascial dehiscence during laparotomy, which of the following absorbable suture materials is MOST appropriate to use for closing fascia?
PDS (polydioxanone) suture. 50% tensile strength at 4 wks vs vicryl (polyglactin) same but at 2 wks
regret over fertility sterilization
Most women who choose sterilization do not regret their decision. The CREST study reported the at-5-years risk of regret is ~7%. Factors that are associated with increased risk of regret include young age, non-white race, unmarried marital status, marital discord, and change in marital status after sterilization.
mgmt skin incision infection after gyne surgery
Individuals heal through a sequence of physiologic events that include inflammation, epithelialization, fibroplasia, and maturation. Hysterectomies are classified as “clean contaminated” cases. Pre-op antibiotic prophylaxis decreases the risk of postoperative infection. However it still remains one of the most common postoperative complications.
A wound infection is usually localized to the skin and fatty tissue above the fascia. The diagnosis of a postoperative wound infection is usually made several days after surgery, on postoperative day 4 or 5. Signs and symptoms include pain, tenderness, erythema, wound induration, fever, and drainage of pus from the incision. The diagnosis can usually be made by inspection and palpation of the wound.
Infected wounds are opened, explored, drained, irrigated, debrided, and dressed. After exploration, the wound is packed until granulation tissue is present and then closed in a delayed fashion, or the wound is allowed to heal by secondary intention.
The need for antibiotic therapy is determined by the extent of the infection, presence of systemic manifestations, and comorbidities of the patient (e.g., immunocompromise, diabetes, chronic steroids). Obesity is another major risk factor. As the BMI increases for women undergoing abdominal hysterectomy, so does the risk of surgical site infections and wound complication.
what should be done if dermoid cyst ruptures
When a dermoid cyst ruptures, the abdomen should be copiously irrigated to avoid chemical peritonitis and chronic pain from adhesions
highest risk of ureteral injury amongst modes of hysterectomies
TLH
risk of d&c
Risks of D&C
risks associated with anesthesia
vasovagal reaction
perforation of the uterus
bowel injury
bleeding
need for further procedures
Indications of a vasovagal reaction may include hypotension, bradycardia, nausea, vomiting, diaphoresis, pallor, or loss of consciousness. Hypotension may also occur due to anesthesia.
cyclic pelvic pain or pressure consistent with regular menses but with primary amenorrhea. The physical exam demonstrates a bulging mass at the vaginal introitus, often with no obvious vaginal orifice. There is a bulging blue perineal membrane at the inferior edge of a palpable fluctuant mass consistent with hematocolpos.
imperforate hymen
cruciate incision / drainage
how to avoid postop pulm complications
Early ambulation, elevating the head of the bed, removing subglottic secretions, and using orogastric tubes all reduce the risk of postoperative pneumonia.
transverse vaginal septum
A transverse vaginal septum arises in the setting of a failure of the canalization of the vaginal plate or failed Müllerian duct fusion. They can develop at any level within the vagina. Thickness can vary from less than 1 cm to 6 cm. A septum may be obstructive or non-obstructive, and presentation will depend on this characteristic. In an obstructive transverse vaginal septum, an accumulation of blood and mucus may present as a pelvic mass.
Symptoms may be similar to those of an imperforate hymen, including cyclic abdominal pain, lack of menstruation, and possibly a lower abdominal or pelvic mass.
outcome of surgery for primary infertility 2/2 endometriosis
Surgery for endometriosis has been shown to improve pain and increase chances of fertility. How much infertility is improved is unclear. Several randomized trials have concluded that surgery for endometriosis results in an increased chance of pregnancy and ongoing pregnancy rate after 20 weeks. Analysis of results showed the number needed to treat was 12 but this number depends on the severity of disease.
Most patients with endometriosis who undergo surgery for their disease will experience a decrease in their pain. A more significant improvement has been found with worse disease. In the first double-blind, randomized, controlled trial approximately 62.5% of women had improvement in their chronic pelvic pain, especially when the implants were resected rather than cauterized. There is, however, a significant recurrence rate.
Infections with Clostridium species are often described during test vignettes, with findings such as air or gas within tissues noted on radiographic evaluation, tissue with crepitation, or tissue or organs with discoloration. Be on the lookout for these clues, leading you to a diagnosis of infection with Clostridium and almost always requiring immediate surgical attention.
What is the BEST management of a small bowel needle stick injury?
In the event of a hemostatic puncture injury or needle stick injury, copious irrigation and suction as well as administration of antibiotics are recommended
Which of the following surgical techniques will decrease the risk of an iliohypogastric or ilioinguinal nerve injury?
Minimizing the use of cautery at small bleeding vessels is appropriate to decrease nerve injury. Cautery is likely to injure terminal ends of sensory fibers that run alongside the vessel bundles.
changes of lab value trends with DIC
The plasma fibrinogen level is decreased in acute DIC. In contrast there is an elevated in the fibrin split/fibrin degradation products and D-dimer.
Answers A & B & C & E: In acute DIC activated partial thromboplastin time (aPTT) is elevated as well as prothrombin time (PTT) and thrombin time (PT). Additionally D-dimer is elevated as well as fibrin degradation products. Laboratory findings that are decreased include platelets, plasma fibrinogen, factor V, and factor VIII.
massive transfusion protocol
Massive transfusion is defined as a transfusion of 10 or more units of packed red blood cells within 24 hours, transfusion of 4 units of packed red blood cells within 1 hour when ongoing need for more blood is anticipated, or replacement of a complete blood volume. The recommended initial transfusion ratio for massive transfusion protocol for packed red blood cells:fresh frozen plasma:platelets is in the range of 1:1:1 and is designed to mimic replacement of whole blood.
Results from a number of observational studies suggest that patients with severe trauma, massive blood replacement, and coagulopathy have improved survival when the ratio of transfused Fresh Frozen Plasma (FFP, in units) to platelets (in units) to red blood cells (RBCs, in units) approaches 1:1:1. The physiology supporting the 1:1:1 (FFP:platelets:RBCs) approach derives from the existence of the acute coagulopathy of trauma and the dilute nature of conventional blood products. More of any one product merely dilutes the other two.
Although transfusion is often lifesaving in obstetrics, usage of blood products, particularly in the setting of massive transfusion, is not without risk. Massive transfusion is associated with hyperkalemia from packed red blood cells and citrate (used as a preservative in stored blood products) toxicity that will typically worsen hypocalcemia. The combination of acidosis, hypocalcemia, and hypothermia all contribute to worsening coagulopathy and increased morbidity.
Which of the following statements regarding the use of bowel prep for gynecological procedures is MOST accurate?
Bowel prep is indicated in gynecological surgeries when there is increased risk of adhesive disease. Procedures such as staging for gynecological cancers or laparotomy or laparoscopy for advanced endometriosis are examples of procedures for which a bowel prep may be used because of high risk of bowel injury.
Which of the following does represent an endpoint of a hysteroscopic resection of a uterine septum
When identifying the endpoint to a septoplasty, a level line at the level of the tubal ostia, serosal transillumination of the hysteroscope at the uterine fundus, increased tissue vascularity, and bleeding are signals for completion of the procedure
Which of the following methods is the MOST effective form of emergency contraception
The copper IUD is the most effective form of emergency contraception. It is also the only form of emergency contraception that may provide continued contraception.
Ulipristal is also highly effective alternative (not as high as copper IUD)
You inform the patient that infections that can be transmitted through blood include all of the following
HIV ,Epstein-Barr virus, hepatitis C, hepatitis B, and human herpesvirus 8 etc can all be transmitted though a blood transfusion
Which of the following values for endometrial thickness is used as a cutoff to predict the need for surgical intervention after first-trimester miscarriage
Reeves et al. showed only a 40% positive predictive value for endometrial stripe thickness, even when adjusting the cutoff. The decision for surgical intervention in the setting of a missed abortion is a clinical decision. Ultrasonographic findings may help guide the decision, but there is no established cutoff for endometrial thickness following medical management of a missed abortion
Which of the following treatment options has been shown to significantly reduce the need for surgical management of small-bowel obstructions?
Gastrografin, a water-soluble contrast, is a studied and effective conservative treatment for small-bowel obstruction that draws fluid into the lumen of the bowel, decreasing edema and increasing peristalsis.If initial treatment with Gastrografin is not successful in resolving a small-bowel obstruction, it can be repeated to a total dose of 100 mL.
Which of the following is TRUE regarding nonsteroidal anti-inflammatory drugs’ mechanism of action as it relates to platelet function?
Nonsteroidal anti-inflammatory drugs (NSAIDs) decrease inflammation by nonselectively inhibiting both cyclooxygenase (COX)-1 and cyclooxygenase (COX)-2 enzymes. Their effect on platelet function is mediated by COX-1 inhibition, thereby decreasing production of thromboxane A2, which decreases platelet aggregation.
Which of the following sutures retains tensile strength for the LONGEST period of time?
Polydioxanone (PDS) is a synthetic absorbable suture that is a pliable monofilament. More than 90% of initial tensile strength is maintained by the end of the first postoperative week, 80% at 2 weeks, 50% at 4 weeks, and 25% at 6 weeks. Complete absorption time is 180 days (6 months). PDS is commonly used for closure of the fascia, especially in patients who are at high risk for wound infection and disruption.
vestibulodynia
Pelvic floor rehabilitation and medications (such as anticonvulsants and antidepressants) are the preferred treatments for vestibulodynia, a chronic pain disorder. When a patient diagnosed with vestibulodynia has failed all other medical therapies, the only remaining option is to surgically remove the vestibule via vestibulectomy
Surgical treatment is reserved for patients who continue to be symptomatic despite these treatments and desire definitive management.
Vestibulectomy is associated with a relief of symptoms in 50–60% of patients. The outer incision extends circumferentially from the periurethral glands along Hart’s line to the contralateral glands. The proximal vaginal margin is just inside the hymenal ring. This horseshoe-shaped epithelium is superficially excised and sent for histologic diagnosis, which usually shows chronic nonspecific periglandular inflammation. For patients with more limited areas of pain, the entire vestibule does not need to be removed.
Which of the following is TRUE regarding the risk of vesicovaginal fistula after hysterectomy?
Radical hysterectomy, where the upper one-third of the vagina is removed, carries the highest risk of VVF, with an estimated incidence of 1/81 cases.
Which of the following nerves is MOST likely injured from a self-retaining retractor during abdominal hysterectomy?
The femoral nerve is the nerve that is injured in this case with compression of a self-retaining retractor against the psoas muscle.
Femoral nerve injuries lead to loss in hip flexion, adduction, and knee extension. Sensory losses after femoral nerve injury include those of the anterior and inferomedial thigh and medial calf.
Which of the following is the MOST appropriate management for a suspected septic abortion with CT findings concerning for a Clostridium infection?
The general management of septic abortion is evaluation and stabilization of vital signs, initiation of broad-spectrum antibiotics, and prompt evacuation of uterine contents.
However, this patient also presents with a Clostridium infection. Findings that may be noted in the presence of a Clostridium infection include lower abdominal tenderness, crepitation of pelvic and abdominal structures, a “woody-appearing” uterus on laparoscopy, and air within the myometrium on radiologic imaging. With the clinical presentation of a septic abortion and findings suggestive of Clostridium, the treatment of choice is hysterectomy with bilateral removal of the adnexa. The procedure can be performed with laparoscopic surgery if indicated.
Which of the following describes the CORRECT management of a 5 mm full-thickness small bowel injury?
A full-thickness injury to the small bowel should be repaired via double-layer closure with suture lines perpendicular to the long axis of the bowel to avoid narrowing of the bowel lumen. The first layer may be a continuous or interrupted 3-0 delayed absorbable suture that closes the mucosa and muscularis. The serosal layer is then closed with 3-0 interrupted silk sutures placed perpendicular to the long axis of the bowel.
Irrigation and administration of IV antibiotics are necessary after the repair to decrease bacterial counts.
Single-layer repair is appropriate for serosal injuries.
Which of the following is TRUE regarding the physiology of the corpus luteum?
The corpus luteum is sustained when pregnancy occurs and the implanted embryo secretes chorionic gonadotropin. The corpus luteum is formed after ovulation from the follicle. The secretion of hCG sustains the corpus luteum so that it can produce progesterone. The secretion of progesterone sustains the pregnancy until the placenta takes over at around 10 weeks’ gestation.
pathogenesis of endometriosis
-Retrograde menstruation leading to attachment and implantation of endometrial glands and stroma on the peritoneum
-Increase in cyclooxygenase-2 (COX-2) leading to local overproduction of prostaglandins
-Increase in aromatase activity leading to overproduction of local estrogen
-Progesterone resistance decreases the antiestrogenic effect of progesterone, which amplifies the local estrogenic effect
A stable patient who has failed medical management for a VTE (i.e. she forms another DVT or progresses to a PE) is a candidate for
placement of a filter in the inferior vena cava (IVC). The rationale behind this decision is that the patient is at such a high risk of recurrent VTE that it is better to mechanically prevent emboli from traveling than to continue anticoagulation, which may be a detriment to the patient’s health.
Which of these signs is the MOST useful in diagnosing a deep vein thrombosis (DVT), and what is the BEST next step in her work-up?
A difference in calf circumference is the most predictive sign of those listed in the diagnosis of DVT. Homans’ sign is unreliable for the presence of DVT. Erythema, warmth, or tenderness are non-specific findings that may add to the overall clinical suspicion for DVT
A full lower extremity Doppler ultrasound is the best first step in the evaluation of DVT where the pre-test probability is high. D-dimer is highly sensitive, but not specific, and can be falsely elevated postoperatively and is therefore not the best next step.
Antibiotics are not indicated for laparoscopic procedures without entry into the bowel or vagina. This includes diagnostic laparoscopy, operative laparoscopy, or laparoscopic tubal ligation.
mgmt of vulvar hematoma
There are multiple trains of thought regarding the management of vulvar hematomas. The most preferred initial management is the observation of a hematoma assuming it appears stable and the patient appears stable. This will generally decrease the risk of infection and a potentially difficult surgery which often has no endpoint.
Should the spread of the hematoma be diffuse or if the patient appears to be becoming unstable, it may be necessary to be more aggressive and perform a surgery. There are 2 separate thoughts on surgery. The first is to perform a large incision and attempt to identify a bleeding vessel to stop the bleeding. The other is to perform a small incision for drainage only and to place a drain to discontinue the growth of the lesion. Neither method is preferred or shown to have better outcomes and often the decision is based on the situation at hand.
Interventional procedures and hemostatic agents may be a useful therapy if the bleeding is unable to be controlled and needs a different type of therapy.
ureteral injuries and mgmt
It may help to classify Ureteral injuries based on location:
1. Ureteropelvic junction (UPJ)
2. Abdominal ureter - renal pelvis to the iliac vessels
3. Pelvic ureter - pelvic brim to the uterine artery
-A pelvic ureteral injury should be repaired with ureteral reimplantation with/without psoas hitch and/or Boari flap
-Abdominal ureter injury can be repaired by ureteroureterostomy, transureteroureterostomy or ureteral reimplantation
-UPJ injury can be repaired with ureteropyelostomy or ureterocalycostomy.
Which of the following is a contraindication to radiofrequency endometrial ablation?
This patient is having a NovaSure, or radiofrequency device, endometrial ablation. There are two major contraindications. They are pregnancy and cancer. When a pregnancy is not diagnosed, the procedure will cause an abortion. When cancer is not identified prior to the procedure, the procedure will cause difficulty adequately staging the cancer and determining subsequent therapy, which will be necessary.
There are several other contraindications to performing the procedure, including the following: endometrial length less than 4 cm, uterine width less than 2.5 cm, IUD in place during the procedure, large fibroid or other structure distorting the endometrium, prior classical cesarean section, active infection or pelvic inflammatory disease, medicinally induced unstable endometrium, or desired future fertility.
truth about LEEP and CKC regarding future pregnancies
When comparing LEEP to CKC, CKC is associated with approximately twice the risk of premature rupture of the membranes, preterm delivery, and very low birth weight of less than 2500g (SGA, not necessarily FGR); 5-8% vs 10-16%. It is important to know that cervical lengths will need to be monitored in those patients who have previously had a cervical excision procedure.
diagnosed with a small vesicovaginal fistula with red granulation tissue around it. What is the BEST next step in management?
Surgical closure in 6–12 weeks allows maturation of the fistula, and bladder catheterization in the interim will prevent worsening of the fistula as well as reduce symptoms.
Vesicovaginal fistulas may be diagnosed by instilling methylene blue into the bladder via a catheter. A tampon or piece of gauze is then placed into the vagina and checked for dye.
Repair of a vesicovaginal fistula involves excising the vaginal epithelium around the fistula in a 2–3 cm diameter. Two or three layers of 2-0 or 3-0 absorbable sutures are placed in a transverse interrupted fashion to allow closure without tension.
UAE contraindications
Absolute contraindications to uterine artery embolization: asymptomatic fibroids (intramural fibroids are fine), pregnancy, pelvic inflammatory disease, uterine malignancy.
Relative contraindications to uterine artery embolization: desire for future pregnancy, postmenopausal, pedunculated submucosal or subserosal fibroids, contraindication to radiologic contrast agents.
FYI UAE is First line for bleeding due to uterine AV malformation
skin prep for pts before lsc hyst
In preparation for surgery, patients need to shower with an antiseptic soap prior to their procedure, and they need to avoid shaving any hair surrounding the surgical site.
In a 14-year-old girl who has not yet had a period but has a blue-red tinted bulge at the vagina, what is the next BEST step?
The incidence of imperforate hymen is approximately 1 in 1,000 live-born females. Although diagnosis is mostly clinical and depends on signs of secondary sexual characteristics, primary amenorrhea, and a bulge at the introitus, a transverse vaginal septum may present with the same features and is best treated with resection rather than a simple cruciate incision. Therefore, ultrasonography of the pelvis can help delineate what is behind the blockage.
noncommunicating uterine horn with a unicornuate uterus
This is a rare occurence and has a high incidence of concurrent absence of the ipsilateral kidney or portions of the urinary system. The complication describes a patient with a pregnancy within the horn, thought to occur when the sperm travels from the contralateral fallopian tube into the accessory horn. When this occurs, rupture of this structure generally occurs at less than 20 weeks; when identified, removal of the accessory horn should occur as soon as possible.
There are 3 other variations that make up the class II Müllerian anomalies:
unicornuate uterus without accessory structures
unicornuate uterus with communicating accessory horn
unicornuate uterus with nonfunctioning, noncommunicating accessory horn.
The uterine septum is the most common of all anomalies and has a high rate of first- and second-trimester abortions. It is best resolved with hysteroscopic surgery.
Müllerian agenesis of the vaginal portion of the Müllerian structures generally can be resolved if the uterus and vagina communicate. If they do not, however, there is a high risk of recurring infections when a communication is made, and these patients often require hysterectomy.
classic sign of PCOS
According to the Rotterdam criteria of 2003, ultrasound findings for PCOS would include 12 or more small follicles on an ovary in women aged 18–35 years. The follicles may be oriented in the periphery, giving the appearance of a “string of pearls.”
staging for ovarian cancer
Surgical staging for ovarian and fallopian tube cancer includes total abdominal hysterectomy, bilateral salpingo-oophorectomy, pelvic washings, peritoneal biopsies, para-aortic and pelvic lymph node dissection, and partial or total omentectomy. Other indications for omentectomy include debulking of major omental tumor burden. The patient’s overall prognosis is improved if immediate surgical staging and/or debulking is performed by a well-trained gynecologic oncology surgeon.
is removing submucosal fibroid helpful w/ infertility?
In a systematic review by Pritts, et al. (subsequently cited in the ASRM Practice Committee guidelines for the use of myomectomy to improve fertility), infertile women who underwent hysteroscopic removal of submucosal fibroids had pregnancy rates 2× higher than infertile women whose fibroids were left in situ. Of note, this trend was NOT seen with ongoing pregnancy or live birth rates which remained unchanged compared with infertile women who did not undergo myomectomy.
Fibroids are the most common tumor in the female reproductive tract and have a cumulative incidence of 70% among reproductive age women. Submucosal fibroids and intramural fibroids that distort the endometrial cavity are associated with a reduced chance of achieving pregnancy.
In the right hands, dilation and evacuation is preferable over medical managment of a missed abortion after 16 weeks’ gestation.
Performing an elective bilateral oophorectomy at time of hysterectomy in a premenopausal patient with no family or personal history of cancer may increase her risk for:
Removing the ovaries at time of hysterectomy, in a premenopausal women, will increase her risk for all cause mortalityespecially if she is not taking estrogen therapy.
Available evidence shows that performing a bilateral oophorectomy prior to menopause increases the rate of all cause mortality, especially in those women who do not take estrogen therapy. This is thought to be secondary to the hypoestrogenic state.
Which of the following is the MOST EFFECTIVE method used to prepare a stenotic cervix for hysteroscopy?
A Cochrane review published in 2015 demonstrated that laminaria may be more effective than misoprostol at ripening a cervix in preparation for hysteroscopy.
The presenting patient has a history of disordered proliferative endometrium on a recent endometrial biopsy. She is now in the emergency department with symptomatic anemia and vaginal bleeding. Which of the following is the MOST appropriate management option for a fertility-desiring patient with prolonged and excessive vaginal bleeding not responsive to medical therapy?
A dilatation and curettage (D&C) is the most appropriate treatment for the patient, who needs immediate relief of her prolonged and excessive bleeding and who has failed hormonal therapy. This patient may desire future fertility; therefore, an endometrial ablation or hysterectomy is not an appropriate choice for surgical management.
For patients who have not yet failed medical therapy, there are a number of treatment modalities available. IV estrogen would be an appropriate first-line treatment for acute abnormal uterine bleeding with a thin endometrial lining, whereas a progestin would be more appropriate for a thickened endometrial lining. Tranexamic acid is an antifibrinolytic agent that may be used either intravenously or orally in the treatment of acute abnormal uterine bleeding.
Intrauterine tamponade with a 30 mL Foley catheter is another modality that may assist in an episode of acute abnormal uterine bleeding.
how to manage DVT / PE short term and long term
Venous thromboembolism is a major cause of morbidity and mortality in the postoperative period. It can occur at any time and is a significant contributor to sudden death.
Once a deep vein thrombosis (DVT) or pulmonary embolism (PE) is diagnosed, immediate anticoagulation is indicated. The initiation of anticoagulation is not to help dissolve the already-present clot, but to prevent more clots from forming.
Two forms of therapy are acceptable in the acute period: low-molecular-weight heparin (i.e., Lovenox) 1 mg/kg twice daily or intravenous unfractionated heparin with varying protocols. Eventually, the patient will need to transition to warfarin for 3 months for a DVT and 6 months for a PE.
viral infection risk w/ blood transfusion?
The risk of acquiring HIV from a blood transfusion is between 1 and 2 million transfused donor products. The risk of acquiring hepatitis B is much higher at 1 in 200,000 transfused donor products.
SAFEST method of miscarriage mgmt in 2nd Trimester
d&e
Sheehan syndrome
state of hypopituitarism resulting from an infarct of the pituitary secondary to postpartum hemorrhage or shock, causing partial or complete loss of the anterior pituitary hormones
Stein–Leventhal syndrome
another name for polycystic ovary syndrome. Although polycystic ovary syndrome can be associated with infertility, it is characterized by clinical or laboratory findings of hyperandrogenism, polycystic ovaries seen on imaging, and oligo or anovulation
Swyer syndrome
46,XY complete gonadal dysgenesis. These individuals have normal female external genitalia, with normally formed fallopian tubes and uterus. The gonads are nonfunctional and are usually removed because of risk of cancer. These individuals may become pregnant with donor eggs/embryos.
Which of the following sonographic characteristics of an ovarian cyst is MOST concerning for malignancy?
An ovarian mass with papillary projections or excrescences, >3 mm in thickness, and location inside the cystic portion is most predictive of a malignant tumor.
One in 70 women will develop ovarian cancer in their lifetime; it has a 90% survival rate in 5 years if caught in the early stages. It is therefore very important to be able to decipher the difference between benign and malignant masses on ultrasonography.
Contraindications to HSG
-Pregnancy
-Pelvic infection (even if the patient is receiving antibiotic therapy)
-History of a severe reaction to iodinated contrast (seizure, cardiac arrest, cardiac arrhythmia, unconsciousness)
-Active undiagnosed vaginal bleeding
A 24-year-old G4P2012 Jehovah’s Witness is undergoing emergent cesarean hysterectomy secondary to hemorrhage. Prior to the procedure, she clearly stated her refusal of foreign blood products. The cell saver is unavailable, and she is losing consciousness. Which of the following is the MOST appropriate infusion for this patient to receive?
Although in a massive transfusion protocol ideally patients should receive blood products if possible, and in a 1:1:1 ratio of platelets to packed red blood cells (PRBCs) to plasma, this patient has refused this.
Volume expanders are most appropriate for a patient who refuses blood products with hemorrhage: crystalloid and colloid solutions. In general, for patients with severe volume depletion or hypovolemic shock, crystalloids are typically preferred over colloid-containing solutions. Colloid-containing solutions are rarely used as first-line resuscitative fluids for the management of hypovolemia and hypovolemic shock. Hyperoncotic starch should, in general, be avoided since its use is associated with an increased risk of kidney dysfunction and mortality.
Per the American College of Surgeons/National Surgical Quality Improvement Program (ACS/NSQIP), chest radiographs are also recommended for patients meeting the following criteria:
-Acute cardiopulmonary disease suspected on basis of history and physical exam. Includes patients who smoke, have asthma, or chronic obstructive pulmonary disease (COPD)
-Older than age 70 with history of stable chronic cardiopulmonary disease and without a recent chest radiograph within the past 6 months
-To establish baseline chest radiograph if patient may require an ICU stay
-Undergoing major surgical operation including abdominal, thoracic, cardiac, some esophageal, thyroidectomy, other head and neck, neurosurgery, and lymph node procedures
What is the MAXIMUM fluid deficit allowed when normal saline is used for hysteroscopy?
Normal saline (NS), an isotonic low-viscosity fluid, is an appropriate fluid medium when performing mechanical or bipolar morcellation because it is associated with fewer side effects as a consequence of fluid overload.
Although it has a more favorable profile than other media, NS still requires some monitoring because the overload can result in pulmonary edema and possibly death. In healthy patients, the cutoff is 2,500 mL.
Answer A: 300 mL is the suggested cutoff for elderly patients when high-viscosity fluids are used.
Answer B: 500 mL is the suggested cutoff for healthy nonelderly patients when high-viscosity fluids are used.
Answer C: 1,000 mL is the suggested cutoff for healthy patients when a hypotonic solution is used.
Which of the following would be the MOST helpful diagnostic modality for identifying the location of a fistulous tract between the bladder and the vagina?
Vesicovaginal fistula may be identified with multiple methods including visualization of dyes or laboratory evaluation. However, locating it is the next step for surgical evaluation and exploration.
Cystourethroscopy directly visualizes the abnormality, and can assess the bladder for residual injury, surgical materials and the number of intravesical fistula orifices. Very small fistulas will be difficult to appreciate on bimanual or speculum examinations. Dye studies alone are not sufficient to completely evaluate the number and location of urogenital fistula. The intravenous pyelography (IVP) is less useful for noting any disruption in ureteral integrity, because it may miss ureteral leakage that is immediately adjacent to the trigone when dye filling the bladder obscures a small leak. Small amounts of dye may not show up on conventional radiography, and puddling may result from ureteral or bladder leakage, or both. When a normal renal unit is seen on IVP, but the ureter is never visualized, complete transection preventing accumulation of dye in the ureter must be considered. A recent study of obstetric vesicouterine fistula demonstrated that pelvic magnetic resonance imaging (MRI) may be more sensitive than IVP or computed tomography (CT).
Dye tests are helpful in assessing whether or not a fistula is present, but are not sufficient to completely evaluate the number and location of urogenital fistulas. Dyed sterile fluid (e.g., sterile infant formula, or indigo carmine or methylene blue mixed with saline, where available) may be instilled into the bladder through a bladder catheter. In combination with the use of blue dye in the bladder, use of oral phenazopyridine (e.g., pyridium) will distinguish a fistula communicating between the vagina and ureter (orange urine) from located in the bladder (blue urine).
Which of the following is the MOST appropriate management of a patient with diabetes who presents to the emergency department with a 4-cm labial abscess?
Vulvar abscesses can present as skin and hair follicle infections, bartholin gland abscess, skene gland abscess, or secondary to wound/hematoma infections. These are often polymicrobial and MRSA tends to be involved. Small abscesses <2cm should undergo incision and drainage (I&D). If the abscess is >5cm or the patient is immunocompromised, the I&D is better done in the OR rather than in the office. When drained, cultures should be collected. After incision and drainage, the wound can be packed daily with wet to dry dressings or a vessel loop can be placed to allow for drainage. Vulvar abscesses can recur and further management may involve marsupialization vs duct excision depending on the location.
Patients with diabetes are considered immunocompromised; if they present with a vulvar abscess they should be admitted for inpatient care as there is always concern for necrotizing fasciitis until proven otherwise.
Which of the following modalities is MOST appropriate for initial management of a cervical ectopic pregnancy in a patient who desires future fertility?
Methotrexate is the treatment of choice for cervical ectopic pregnancy. It can be administered either systemically or via intra-amniotic injection.
Use of this modality avoids disturbing the pregnancy, thus decreasing the risk of catastrophic hemorrhage.
what should u do if you have bladder dome injury >2cm or multiple small ones during hyst?
finish hyst first if possible then repair
2 layer closure for >=2cm injury, check w/ backfill, keep foley for 7-10 days then f/u cystogram to confirm closure of defect
When there are multiple cystotomies in close proximity to each other in the bladder dome, an option is to incise the small defects to combine them into a single defect before beginning the repair. When the repair is completed, the closure should be tested to ensure it is water-tight by instilling dye into the bladder catheter and observing the repair site laparoscopically. The bladder will re-epithelialize within 3–4 days and will regain its normal strength after 21 days.
Which of the following is considered the gold standard for the assessment of tubal patency?
Diagnostic laparoscopy with chromopertubation remains the gold standard investigation in the assessment of tubal patency. It provides a direct view of the pelvic organs, and also offers the possibility to treat minor pathology discovered during the investigation.
Methylene blue dye is inserted through a cannula in the cervix to demonstrate tubal patency. Spillage of the dye from each tube is noted via direct visualization as a confirmation of tubal patency. If a repair procedure for tubal occlusion is performed, chromopertubation is repeated at the end of the procedure.
Diagnostic laparoscopy requires admission to hospital and general anesthesia and, as a result, is the most invasive and expensive.
HSG
Hysterosalpingogram (HSG) is typically the first-line test used for tubal patency. It involves inserting a cannula into the cervix and passing radio-opaque fluid into the uterine cavity and fallopian tubes, demonstrating their outline. The test is performed under X-ray screening on an outpatient basis.
If the HSG is normal, this finding can be relied upon in 97% of cases. However, if the HSG is abnormal, the diagnosis can only be relied upon in 34% of cases (false positive rate 66%), and a laparoscopy is required to confirm the nature of the abnormality.
Proximal tubal occlusion on HSG often represents testing artifact due to tubal spasm or poor catheter positioning leading to unilateral tubal perfusion. Given these deficiencies, findings of proximal tubal occlusion on HSG could be confirmed by secondary testing.
Management of an expanding labial hematoma involves which of the following?
surgical exploration and ligation of bleeding source
In an elderly patient with multiple comorbidities who does not desire further sexual intercourse, which of the following is the MOST appropriate surgical option to treat symptomatic prolapse?
In an elderly patient with multiple comorbidities who does not desire further sexual intercourse, which of the following is the MOST appropriate surgical option to treat symptomatic prolapse?
failed conservative approach to VV fistula. waht next?
Foley catheter drainage and decompression of the bladder may enable reepithelialization and closure of small vesicovaginal fistulas. In the remainder of cases in which conservative measures fail, surgical management is the best and most effective option. The Latzko procedure is a technique for fistula repair via the vaginal route and provides the highest chance for successful closure at 80%–90%.
A 23-year-old patient presents to the hospital ED with bleeding, cramping, and passage of clots, and is 10 weeks pregnant by her last period. Ultrasonography confirms an intrauterine pregnancy with no fetal heartbeat. On exam, the patient has an open cervical os, and active bleeding is noted. Which of the following is the MOST appropriate treatment for this patient?
suction d&c
Which of the following is TRUE regarding the management of a vulvar hematoma following trauma?
A Foley catheter should be placed on initial inspection of a vulvar hematoma because it may be difficult to pass a catheter without sedation if swelling from the initial trauma or hematoma continues. Swelling from trauma can make it difficult to void after trauma, secondary to edema or pain.
Stable vulvar hematomas may be managed conservatively with Foley catheter placement, ice packs, and pain medications. Expanding hematomas require surgical evaluation.
Which of the following MOST accurately describes which fluid media can be safely used when monopolar electrosurgical instruments are used for fibroid resection?
When an operative hysteroscopy is performed with a monopolar electrosurgical device, the distending fluid medium must be electrolyte-poor, such as glycine, sorbitol, or mannitol.
Normal saline contains electrolytes and cannot be used with monopolar electrosurgical energy. Its use is acceptable during resections with bipolar electrosurgical devices.
The physician has been called to the ED to evaluate a 19-year-old G1 at 19 weeks’ gestation. She reports routine prenatal care without complications. She is in the ED for intermittent abdominal pain over the last week that suddenly became worse tonight and has not gone away. She was told on her last ultrasound that she had an ovarian cyst, but she remembers nothing else. Which of the following ultrasound findings would necessitate surgery in the gravid patient?
Surgery for adnexal masses in pregnancy is dependent on a number of factors. Ovarian masses are seen in up to 6% of women up to 14 weeks. Up to 4% of these women have complications from these cysts: torsion, malignancy, or dystocia at delivery. Intermittent pain may represent ovarian torsion, which occurs most often with cysts that are between 5–10 cm in size.
Given our patient’s history, absent reversed or reduced doppler flow likely indicates torsion and thus that ultrasound finding would be most indicative of requiring surgery. The best time for a surgery to occur is in the second trimester. Complications increase significantly when surgery occurs after 23 weeks. The risk of a preterm delivery increases greatly, as high as 50% in one study. Ultrasound findings may be the key portion of an exam to indicate the need for surgery vs. expectant management.
Following oophorectomy at 6 weeks’ gestation, which of the following is MOST accurate regarding postoperative management?
Bottom Line: When a corpus luteum is removed prior to 10 weeks’ gestation, progesterone must be supplemented until 10 weeks’ gestation, by which time the placenta will be the primary producer of progesterone.
Progesterone supplementation is required as progesterone is largely produced by the corpus luteum until about 10 weeks of gestation. Pulsatile pituitary luteinizing hormone and eventually human chorionic gonadotropin from the implanting pregnancy stimulate progesterone production by the corpus luteum, which is essential for pregnancy maintenance until about the seventh week, when progesterone production is sufficient. The seventh to tenth week is considered a period of shared function between the corpus luteum and the placenta, at which point the placenta emerges as the major source of progesterone synthesis, and maternal circulating levels progressively increase.
When a corpus luteum is removed while treating adnexal torsion in pregnancy prior to 10 weeks’ gestation, progesterone must be supplemented until 10 weeks’ gestation, by which time the placenta is the primary site of progesterone production. Suitable progesterone regimens are listed below.
Pregnancy is associated with an increased risk of ovarian torsion, accounting for 10–22% of cases of torsion. The incidence of torsion in pregnancy is unknown, but some studies show that if a persistent adnexal mass measures equal to or greater than 4 cm, the incidence is 15%. Torsion is most likely to occur between 10 and 17 weeks of gestation, but can occur at any time including the postpartum period.
A 27-year-old G5P2032 patient has an immediate postoperative hemorrhage following a suction dilatation and curettage (D&C) for a missed abortion. Her vital signs include a blood pressure level of 90/60 mm Hg and pulse rate of 135/min. The bleeding is not responsive to misoprostol or Methergine. Which of the following is the MOST appropriate management of this patient?
This patient has a hemorrhage following a D&C for a missed abortion that is unresponsive to uterotonics. She has tried 2 medications, but neither has been successful. The most likely cause of a postoperative hemorrhage in this condition is atony.
Atony should first be treated using medication. Generally, this works, but if it is unsuccessful, other questions need to be asked. The first is whether there are still retained products in the uterus. This is best accomplished using a repeat curettage, possibly under ultrasound guidance. Once the absence of retained tissue is confirmed and if the hemorrhage persists, uterine artery embolization or a Foley balloon in the uterus may be used.
A 25-year-old G2P1011 woman with a history of migraines presents to the office with her husband. They have been trying to conceive for 14 months following a missed abortion at 9 weeks gestation for which she had a D&C. Which of the following is the BEST next step in management?
When intrauterine adhesions are suspected, a hysterosalpingogram is indicated.
Intrauterine adhesions characteristically appear within the uterine cavity as irregular and angulated filling defects. Intrauterine adhesions may develop in up to 8% of patients after a single curettage for pregnancy loss. Asherman syndrome includes a wide spectrum of scarring, including filmy adhesions, dense bands, and complete obliteration of the uterine cavity. Although many affected patients are asymptomatic, the most common presenting problem is infertility, which occurs in up to 40%–45% of patients.
Amenorrhea may be observed with extensive intrauterine scarring. In less severe cases, patients may present with hypomenorrhea or with recurrent pregnancy loss. Hysteroscopic lysis of adhesions is the preferred surgical management. High-dose estrogen therapy or placement of an intrauterine Foley catheter can prevent postoperative recurrence of adhesions.
Which of the following is the MOST appropriate entry when performing diagnostic surgery in a patient during the early first trimester who has an adnexal mass?
Once the decision for surgery has been made, the type of entry is the next decision to be made.
Prior to completion of the first trimester, an infraumbilical laparoscopic entry is appropriate. After the first trimester, open technique is preferred with placement of a towel clamp if air escapes the trocar site. Palmer’s point is reserved for more advanced pregnancies when entry using an umbilical approach would increase risk of uterine damage or entry. As the uterus reaches 20 weeks’ gestation, the uterus will be closer to the umbilicus. Open abdominal incisions are best reserved for larger lesions or patients who are more unstable. When deciding between laparoscopy or laparotomy, common sense should dictate the decision.
When performing abdominal insufflation, it is important to remember that insufflation greater than 15 mm Hg can lead to hypercarbia, acidosis, and uteroplacental hypoperfusion.
This patient is in the first trimester and is best served by infraumbilical laparoscopic entry.
Which of the following is the MOST common sign or symptom associated with necrotizing fasciitis?
Most patients with necrotizing fascitis (75%) have edema that extends beyond the erythema. Significant edema may cause compartment syndrome with myonecrosis and require immediate fasciotomy. Necrotizing fasciitis is an infection of the deep soft tissues. It results in the destruction of the muscle fascia and the overlying subcutaneous fat. Risk factors for necrotizing fasciitis include recent trauma or surgery, immunosuppression (including HIV, diabetes, and neutropenia), malignancy, obesity, and alcoholism. Necrotizing fasciitis can be divided into 2 categories of microbes:
Polymicrobial (type 1) necrotizing infection is caused by aerobic and anaerobic bacteria. At least one anaerobic species (most commonly Bacteroides, Clostridium, or Peptostreptococcus) is isolated in combination with Enterobacteriaceae (e.g., Escherichia coli, Enterobacter, Klebsiella, Proteus) and at least one facultative anaerobic streptococci.
Monoclonal (type II) necrotizing infection is caused by group A strep (GAS) or other beta-hemolytic streptococci. Less commonly, the infection may be caused by Staphylococcus aureus.
Differentiating between cellulitis and necrotizing fasciitis can be challenging when presenting symptoms are non-specific. Since necrotizing fasciitis spreads within the subcutaneous tissue with relative sparing of the overlying skin, treating clinicians commonly underestimate the extension and seriousness of the infection.
A 24-year-old G1P0 presents to the office for a concern about “masses” on her vulva. After a thorough history and examination, the patient is diagnosed with genital warts. She would like treatment because the warts are causing her discomfort. Which of the following is the MOST appropriate treatment of genital warts in pregnancy?
Trichloroacetic acid is an immune-based treatment for treating genital warts and is the preferred treatment in pregnancy because it has no systemic absorption.
When using medical treatment, a response should be observed in 3 weeks with resolution in 6–12 weeks.
Which of the following is the BEST entry in a patient with numerous pelvic and abdominal surgeries in the past?
Patients with a history of multiple abdominal or pelvic surgeries are likely to have severe intraabdominal adhesions. Entry at Palmer’s point is typically above the level where adhesions are found in patients who are at risk.
Palmer’s point is 3 cm lateral of midline and 3 cm below the left rib cage, and the trocar/Veress needle is directed 15° cephalad. Unless the patient has had a splenectomy or significant intraabdominal trauma, this is the best place to enter the abdomen, even when performing pelvic surgery, and longer instruments may be necessary.
The levonorgestrel pills and ulipristal acetate work by
inhibiting or delaying ovulation. While levonorgestrel delays follicular development, ulipristal acetate inhibits follicular rupture. The copper IUD affects sperm viability and function to prevent fertilization.
A 17-year-old patient presents to the emergency department with new-onset right-sided pain. The pain is described as waxing and waning in nature and is associated with nausea and vomiting. She denies vaginal bleeding. Abdominal exam shows significant tenderness in the right lower quadrant without rebound or guarding. Beta-human chorionic gonadotropin (hCG) is negative. Pelvic ultrasonography demonstrates a 6 cm right ovarian cyst. Which of these findings is MOST sensitive and specific for her condition?
An adnexal mass greater than 5 cm places the patient at increased risk of ovarian torsion. The most common etiologies in those with ovarian torsion are functional ovarian cysts or dermoid cysts. In a series of cases with suspected torsion, the most sensitive and specific sonographic findings were ovarian edema and relative enlargement of ipsilateral ovary. Other findings include absent arterial flow, which is present in only 40% of cases.
absent blood flow on doppler is very specific but not sensitive as a half the times blood flow is normal
A patient presents with leakage of urine with coughing and sneezing. She denies urgency and a urine culture is negative. Her post void residual is 230 mL. She desires treatment. Which of the following is the next BEST step in her management?
urodynamics
The patient described in the question has an increased postvoid residual and therefore the best next step is urodynamic testing to evaluate for overflow incontinence.
TrueLearn Insight: Urodynamic testing does not improve outcomes in women treated with mid-urethral slings for uncomplicated stress urinary incontinence and is therefore not indicated in patients with a normal postvoid residual.
Intraoperative fluid resuscitation:
1) Crystalloid 2) Colloid 3) Blood. In the event you have already provided crystalloid and the patient is refusing blood, opt for an appropriate colloid, like Albumin.
Which of the following is TRUE regarding follow-up of medical abortion?
follow up with phone / clinic, recommended by FDA
inferior epigastric artery
branch of the external iliac artery with its origin just above the inguinal ligament. It courses superiorly and medially towards the umbilicus. It lies just medial to the round ligament as the ligament passes through the deep inguinal ring into the inguinal canal and lateral to the obliterated umbilical vessels. It pierces the fascia transversalis to enter the rectus sheath anterior to the arcuate line. It ascends behind the rectus muscle, supplying the lower central part of the anterior abdominal wall, and anastomoses with the superior epigastric artery.
An incidental finding of a corpus luteum is most likely noted during which days of a typical 28-day menstrual cycle?
The normal female menstrual cycle typically lasts 28–35 days. Day 1 is the first day of the menstrual cycle by convention. The follicular phase includes the first 14–21 days of the menstrual cycle, and the luteal phase is 14 days. The luteal phase typically does not change in length; however, the follicular phase may vary by several days. Ovulation typically occurs within 24–36 hours of the luteinizing hormone (LH) surge, and formation of the corpus luteum is noted as early as 1 hour after ovulation. In a typical 28-day cycle, the corpus luteum would be expected to be seen on imaging as early as day 14.
An incidental finding of a corpus luteum is most likely noted during which days of a typical 28-day menstrual cycle?
The normal female menstrual cycle typically lasts 28–35 days. Day 1 is the first day of the menstrual cycle by convention. The follicular phase includes the first 14–21 days of the menstrual cycle, and the luteal phase is 14 days. The luteal phase typically does not change in length; however, the follicular phase may vary by several days. Ovulation typically occurs within 24–36 hours of the luteinizing hormone (LH) surge, and formation of the corpus luteum is noted as early as 1 hour after ovulation. In a typical 28-day cycle, the corpus luteum would be expected to be seen on imaging as early as day 14.
During laparoscopic surgery, what is the maximum intraabdominal pressure allowed in order to ensure central venous return and diaphragmatic excursion?
Though venous pressure refers to the average blood pressure in the venous system, central venous pressure (CVP) describes the blood pressure in the thoracic vena cava as it empties into the right atrium. It is the best approximation for the right atrium filling pressure, and by extension the right ventricle filling pressure. It is important not to increase the pressure in the abdomen above the CVP because it will decrease cardiac output by decreasing the preload.
During gynecologic surgery, common pneumoperitoneum pressures are low (8 mm Hg), standard (12 mm Hg), and high (15 mm Hg). However, intraabdominal pressures over 20–24 mm Hg create excessive resistance for ventilation.
Which of the following MOST accurately describes postexposure HIV prophylaxis after a sexual assault?
Postexposure prophylaxis for HIV in the nonoccupational setting includes 28 days of multidrug HAART therapy and is most effective when administered within 72 hours of exposure.
Which of the following is an expected sonographic finding during the midportion of a normal menstrual cycle?
A simple left ovarian cyst with thickened irregular cyst wall is the most likely ultrasound finding to support mittelschmerz. Mittelschmerz is pain that occurs around the time of ovulation; for many patients this is acute in nature and single sided.
In terms of the menstrual cycle, an average cycle is 28 days, with 14 days in the follicular phase and 14 days in the luteal phase. On day 14, ovulation occurs, and the follicle previously containing the oocyte becomes the corpus luteum. If an ultrasound were to be performed at this time, a corpus luteum would likely be seen and would appear as a simple cyst with a thickened irregular cyst wall.
mild endometriosis
Mild endometriosis consists of superficial implants that are less than 3 cm in aggregate and are scattered on the peritoneum and ovaries. No significant adhesions are present.
In a patient with severe chronic obstructive pulmonary disease (COPD) requiring constant oxygen therapy who is also undergoing hysterectomy, which of the following is the MOST advantageous surgical approach?
This patient is in end-stage lung disease. Thus, it is already difficult to create oxygen exchange, to maintain oxygenation, to obtain lung compliance, and to overcome intra-abdominal pressure (IAP). Endoscopic, vaginal, and open abdominal surgery, though generally higher in morbidity secondary to infection and healing, are not going to increase abdominal pressure directly during the case. Of these, vaginal surgery is the least invasive option.
Laparoscopic surgery creates a pneumoperitoneum and when this occurs IAP exceeds physiological thresholds. With this increased abdominal volume, compliance declines and IAP continues to climb. In these situations, the degree of oxygenation is based on the patient’s position. When the patient is placed into the steep Trendelenberg, as often is needed on the Robotic system, there is increased risk of laryngeal edema increasing risk of stridor. There is also decreased functional residual capacity (FRC), and ventilation/perfusion mismatch is worsened. This significantly worsens the morbidity and mortality in the patient with end-stage respiratory disease.
On a diagnostic laparoscopy as part of an infertility workup, the physician notices filmy, violin string-like adhesions between the anterior portion of the liver and the anterior abdominal wall. Which of the following treatments could have prevented the cause of this finding?
The patient described in this case has classical findings of Fitz-Hugh–Curtis syndrome. This ascending pelvic inflammatory infection is most commonly caused by chlamydia or gonorrheal infection. Acute Fitz-Hugh–Curtis syndrome can be identified laparoscopically by a patchy purulent exudate on the anterior surface of the liver. Chronic periphepatic fibrosis appears as fibrinous exudate often described as “violin string-like” adhesions.
This ascending infection could have been prevented with treatment of the venereal infection with ceftriaxone and doxycycline.
Cefoxitine and oral doxycycline are recommended for inpatient treatment of PID. Metronidazole is added on when PID is complicated by wet mount positive for bacterial vaginosis OR when pelvic abscesses are seen on imaging.
Ceftriaxone alone is not recommended for PID. Ceftriaxone does treat gonorrhea; however, when treating for gonorrhea you should empirically treat for chlamydia too.
Doxycycline alone is not the recommended treatment for PID. Doxycycline would only treat chlamydia
A 58-year-old gravida 0 postmenopausal woman presents for her annual exam. She denies any complaints. On bimanual exam, you note a small, mobile, nontender uterus with regular contour as well as right adnexal fullness. Transvaginal ultrasonography reveals a right unilocular, thin-walled ovarian cyst measuring 7 cm in diameter with smooth borders. Which of the following is the next BEST step in management?
repeat US in 3-6months
Ultrasonographic findings suggestive of a benign adnexal mass include unilocular, thin-walled, sonolucent cysts with smooth, regular borders. The majority of these types of cysts are benign regardless of menopausal state or cyst size.
In a large prospective study, more than 2,700 postmenopausal women with unilocular cysts measuring 10 cm or less were followed with ultrasonography every 6 months. After a follow-up of more than 6 years, no cancers were detected, and spontaneous resolution of the cyst occurred in two-thirds of the women. Therefore, simple cysts up to 10 cm in diameter measured by ultrasonography are almost universally benign.
What is the time cut-off above which a geriatric surgical patient would be at a high risk for falls when asked to perform a timed up and go test (TUGT), wherein she stands up from a chair, walks 10 feet, then returns to her chair and sits down?
Geriatric patients are at increased risk for falls, especially when they are postsurgical and in unfamiliar environments. A single fall may have catastrophic consequences for the patient.
The American College of Surgery and National Surgical Quality Improvement Program (NSQUIP) recommend that all geriatric patients who are scheduled for surgery be evaluated for fall risk. They recommend using the timed up and go test (TUGT) to assess their fall risk. This involves the patient standing up from a standard chair, walking 10 feet, turning around, returning to the chair and sitting down. This task should be completed unassisted and should take no more than 15 seconds.
Patients who take more than 15 seconds are at high risk of falls and the provider should consider referral to physical therapy.
Which of the following BEST represents the sensitivity of ultrasonography in diagnosing an ovarian torsion?
Ultrasound is an excellent imaging modality to detect adnexal torsion, with reported sensitivity up to 92% in recent studies
Ovarian torsion is a clinical diagnosis, and these patients present with acute, severe, unilateral, lower abdominal and pelvic pain, in addition to nausea and vomiting.
Ultrasound is the imaging modality of choice. While older studies reported sonographic sensitivity of 50–75% for ovarian torsion, other more recent studies have reported sensitivities of 72.1%–80% and specificities of 95%–99% in the general population. In the adolescent population, ultrasound is the imaging modality of choice with 92% sensitivity and 96% specificity in detecting adnexal torsion.
There are several sonographic findings suggestive of ovarian torsion:
Unilateral ovarian enlargement
Ovarian edema – hyperechogenic ovary with echogenic stroma
Peripherally displaced follicles
Free fluid
Coiled vascular pedicle “whirlpool sign”
If the ovary is normal sized on ultrasound, it is unlikely to be twisted, although it is possible for torsion to occur with a normal-appearing ovary or fallopian tube.
How should a patient, who is 28 weeks pregnant, be positioned for a laparoscopic cholecystectomy?
When laparoscopy is performed during pregnancy, regardless of gestational age, the patient needs to be in the left lateral recumbent position to prevent uterine compression on the IVC.
Laparoscopy during pregnancy has been proven safe through a number of retrospective studies. It is safe in all trimesters but is preferred during the second trimester because the first trimester is associated with increased risks of pregnancy loss and spontaneous abortion and the third trimester is associated with increased risk of preterm labor.
Regardless of the gestational age at which surgery is performed, the positioning—left lateral recumbent—is the same because the uterus is displaced in order to prevent compression of the vena cava and venous return.
Which of the following is more likely when comparing open techniques (ie, Hasson) with closed techniques (ie, Veress needle) for laparoscopy?
Abdominal entry with an open, Hasson, technique is associated with lower risk of vascular injury. While this is generally thought to be the case, there are no randomized controlled trials to evaluate this. There is no statistical difference in bowel injury rates.
Laparoscopic Entry Technique Risk of Vascular Injury
Open (Hasson) 0%
Closed (Veress) 0.44%
Therefore, of all of the answer choices, decreased risk of vascular injury is the most appropriate answer.
Which of the following is an appropriate treatment to help manage a 5 cm, type 1 uterine fibroid noted on a saline infused sonohysterogram?
Leiomyomas (fibroids) can be located in various places. Their location will determine the method of treatment.
Type 1 fibroids are submucosal with less than 50% of the fibroid being located in the uterine cavity. Type 0 fibroids are pedunculated (intracavitary). Type 0 and type 1 fibroids can be removed via hysteroscopy. A laparoscopic myomectomy is more appropriate for type 2–7 fibroids, as they are more easily accessible from the abdominal cavity.
What medical management option is specifically FDA-approved for the treatment of acute abnormal uterine bleeding?
Intravenous administration of estrogen is an appropriate initial medical management option that is the only FDA-approved treatment for acute abnormal uterine bleeding. This works well particularly in the pediatric population, and is extremely efficacious at halting the continuous flow. Studies have shown that oral therapy is, also, an efficacious therapy.
The estrogen works by rapidly causing proliferation of the endometrium to prevent shedding and works on most forms of dysfunctional uterine bleeding. Caution is advised with estrogen use in older women. Although it still may be efficacious, there is a higher chance of thromboembolism as women age, and particularly if there are other risk factors (e.g., smoking).
Initial management of a simple vesicovaginal fistula that is diagnosed approximately 2 weeks following a total abdominal hysterectomy would include which of the following?
Once the diagnosis is made, the bladder must be drained via Foley catheter for 4 weeks for resolution of symptoms. Occasionally, this may result in healing of the fistula. If the fistula is still present then surgery must be performed.
Most vesicovaginal fistulas can be closed transvaginally and the technique most commonly used for simple fistulas is called the Latzko technique. This surgical technique involves cannulating the vesicovaginal fistula with a lacrimal duct probe, small feeding tube, or pediatric Foley catheter to help the surgeon pull the involved area of vagina toward the introitus to facilitate vaginal dissection. The vaginal epithelium is incised around the fistula. Vaginal epithelial flaps are then raised and removed in a circle with a diameter of 2–3 cm around the fistula tract. Two layers of 2-0 or 3-0 absorbable sutures are placed in a transverse interrupted fashion to close the defect without tension.
However, surgical repair of a vesicovaginal fistula should be delayed 6–12 weeks after the gynecologic surgery to allow granulation tissue to dissipate. During this waiting period, catheterization of the bladder may decrease symptoms.
A 37-year-old G2P2 presents to the office for an annual examination. She has no medical problems and complains only of pelvic pain. She reports that she has regular 28-day menstrual cycles and states that the first day of her last menstrual period was 24 days ago. On ultrasonography performed the same day, the endometrial thickness is 14 mm and 3 layers are visible. If the physician were to take a biopsy of the patient’s endometrium at this visit, what would the pathology report be expected to say?
The first half of the menstrual cycle is the proliferative phase, where endometrial epithelial and stromal cells divide under the influence of estrogen. The second half is the progesterone-dependent secretory phase, characterized by hypertrophy of uterine glands and decidualization of stromal cells. After ovulation occurs, the secretory phase is initiated by the production of large amounts of progesterone by the corpus luteum, leading to increasingly elongated glands and abundant spiral arterioles.
Answer A: Atrophic endometrium is typically seen in postmenopausal women or in those who do not have proper hormonal stimulation of the endometrium.
Answer B: Dyssynchronous endometrium would be expected on the pathology report of an individual who has irregular shedding of the endometrium secondary to unopposed estrogen or as a result of anovulation.
Answer C: Hyperplastic endometrium is an abnormal finding on an endometrial biopsy and is a precursor lesion to endometrial cancer.
A patient undergoes pelvic ultrasound today, and her last menstrual period began 20 days ago. She reports regular periods every 28–30 days that last for approximately 4 days. Which of the following ultrasound findings would be MOST expected?
The endometrium goes through cyclical changes throughout the menstrual cycle, beginning with preparing for possible implantation from an embryo and ending with sloughing of the lining when no embryo is received. The appearance of the endometrium is also demonstrable on transvaginal ultrasonography; using average thicknesses, a rough estimate about timing of the menstrual cycle can be made.
In general, the endometrial thickness increases most dramatically during the follicular period because of estrogen secretion from the dominant follicle. The luteal phase is marked by continued growth, but at a slower rate, and proliferation of secretory glands that prepare the surface for the pending embryo. Finally, when progesterone supplementation from the corpus luteum is not rescued by an embryo, the lack of progesterone causes sloughing of the endometrium, and the cycle starts anew the next month. The endometrium is thickest at the time of ovulation (highest estrogen content), and the presence of an adnexal cyst most likely confirms a corpus luteum.
The patient has 28- to 30-day cycles, meaning she ovulates on day 14–16; thus, day 20 would be firmly in the luteal phase of the menstrual cycle. You would therefore expect to see a thickened endometrium with the presence of a corpus luteum.
ovarian torsion US findings
Ultrasound is the inital imaging study of choice for patients with suspected ovarian torsion. Ultrasound findings that are suggestive of ovarian torsion include all of the following: an enlarged ovary compared to the contralateral ovary, an ovarian mass, heterogeneous appearance of the ovarian stroma, abnormal ovarian location, and decreased or diminished Doppler flow within the ovary.
echogenic finding would be peripheral not central due to displaced follicles from edematous changes
In which of the following scenarios should an IUD be removed?
An IUD can become malpositioned in the uterus and lead to pain and irregular bleeding. This can present as an IUD embedded in the myometrium, protruding through the serosa, partially expelled through the cervix, or rotated on its axis.
If the IUD is not positioned correctly, there is concern for contraceptive failure, as well as organ perforation so it must be removed when recognized.
Answer A: Actinomyces noted on Pap smear cytology is a common finding in approximately 7% of patients. It generally requires no treatment.
Answer B: It is not necessarily the patient’s IUD causing the fever.
Answer C: A history of gonorrhea is not a contraindication to IUD insertion.
Answer D: Pelvic inflammatory disease may be treated with antibiotics and the IUD can remain in place.
In which of the following scenarios should an IUD be removed?
An IUD can become malpositioned in the uterus and lead to pain and irregular bleeding. This can present as an IUD embedded in the myometrium, protruding through the serosa, partially expelled through the cervix, or rotated on its axis.
If the IUD is not positioned correctly, there is concern for contraceptive failure, as well as organ perforation so it must be removed when recognized.
Answer A: Actinomyces noted on Pap smear cytology is a common finding in approximately 7% of patients. It generally requires no treatment.
Answer B: It is not necessarily the patient’s IUD causing the fever.
Answer C: A history of gonorrhea is not a contraindication to IUD insertion.
Answer D: Pelvic inflammatory disease may be treated with antibiotics and the IUD can remain in place.
Which of the following treatment options is appropriate for a perimenopausal woman with anemia and acute abnormal uterine bleeding secondary to anovulation?
Abnormal uterine bleeding (AUB) may be acute or chronic and is defined as bleeding from the uterine corpus that is abnormal in regularity, volume, frequency, or duration and occurs in the absence of pregnancy. Acute AUB refers to an episode of heavy bleeding that, in the opinion of the clinician, is of sufficient quantity to require immediate intervention.
Hormonal management is considered the first-line of medical therapy for patients with acute AUB without known or suspected bleeding disorders. Treatment options include IV conjugated equine estrogen, combined oral contraceptives (OCs), and oral progestins. Antifibrinolytic drugs, such as tranexamic acid, are also a considered treatment option. Tranexamic acid works by preventing fibrin degradation.
Which of the following activities is MOST useful for a preoperative functional assessment of an elderly patient?
Elderly patients who have limited functional capacity are at higher risk of significant postoperative morbidity, as the deconditioned patient has little reserve to recover from the stress of surgery. This places the patient at risk for poor health outcomes, falls, decreased mobility, prolonged hospitalization, and death.
When considering surgery on a geriatric patient, it is important to assess the patient’s preoperative functional status. Functional status can be estimated in the office by evaluating the following:
Components of functional status
Nutritional status, eating, feeding
Continence
Transferring
Dressing
Bathing
A proper assessment of functional status can assist pre- and postoperative planning. Some patients may benefit from rehabilitation, improved nutrition, or physical therapy.
The pregnancy rate following excision of an endometrioma is:
Endometriosis is a common cause of pelvic pain and can lead to infertility. The mechanism of infertility is not specifically known, but endometriosis can damage the fallopian tubes and cause unfavorable inflammatory environment in the pelvis that can hinder sperm function. Removal of endometriomas has been found to increase fertility rates.
Pregnancy Rate
Endometrioma excision 60.9%
Endometrioma drainage and ablation 23.4%
Surgical management of endometriosis can improve pregnancy rates dramatically. Excision of an endometrioma results in a pregnancy rate of 60.9%, whereas drainage and ablation of an endometrioma results in a pregnancy rate of 23.4%.
The American Society for Reproductive Medicine (ASRM) has a commonly used classification/staging system for endometriosis. The utility of this system is for uniformly documenting operative findings of endometriosis. The scoring has not proved to correlate well with pain nor fertility outcomes.
The stages are based on a point system in which endometriotic implants are classified according to their location (peritoneum and ovary) and whether they are superficial or deeply invasive and whether the posterior cul-de-sac is partially or completely obliterated, and adhesions that are based on location (ovary, tube) and whether or not they are filmy or dense. Deep invasion, particularly of larger implants, portends a score of 20 and is the second highest modifier behind complete obliteration of the cul-de-sac (score of 40).
What is the next BEST step in management in a stable patient with a newly identified perforated IUD?
If the strings are not seen protruding from the cervical os, a pelvic US should be performed. If the IUD is not seen in the uterus on US, an X-ray of the abdomen and pelvis should be obtained. If imaging reveals that the IUD has perforated the uterus, the patient should undergo laparoscopic removal.
Which of the following is CORRECT regarding successful medical management of an ectopic pregnancy?
Using the single-dose protocol, a decline ≥15% on days 4 to 7 is considered a success. Beta-hCG values can then be followed weekly, and as long as there is at least a 15% decline, surveillance can be continued. If there is < 15% decline, the dose can be repeated.
An important point is that treatment day 1 is considered day 1, which is a common misunderstanding for some residents.
Beta-hCG levels should rise between days 1 and 4, rather than decline. This is due to the rupture of ectopic syncytiotrophoblasts and the hCG they contain. This should be expected and should not alert the physician to failure of treatment unless the levels between days 4 and 7 do not decline as expected.
The dosage of leucovorin, a folic acid analogue given on non-methotrexate treatment days in the multidose protocol, is 0.1 mg/kg rather than 1.0 mg/kg.
Which of the following is the MOST appropriate management of a 28-year-old G0 who has infertility and chronic pelvic pain from endometriosis and an ultrasound that shows a 4-cm endometrioma?
Excision of endometriomas 3 cm or larger is recommended to improve fertility.
Risk Factors for Endometrial Carcinoma
Age
Obesity
Use of unopposed estrogen
History of PCOS, DM II, atypical glandular
cells on pap smear
Family history of endometrial carcinoma
The classic findings of a hemorrhagic cyst are as follows
thin-walled cyst with fibrin strands and low-level echoes.
A 70-year-old woman with no history of postmenopausal bleeding or cancer presents after a transvaginal ultrasound revealed an endometrial stripe of 6 millimeters. What is the BEST next step in management?
The appropriate work-up for a thickened endometrial stripe in an asymptomatic postmenopausal woman is not well-defined. According to the American College of Obstetrics and Gynecology, an endometrial stripe of 4 millimeters or less can be considered normal, but the significance of the presence of an endometrial stripe greater than 4 millimeters in a postmenopausal woman without vaginal bleeding has not been established, and may not necessitate further evaluation.
Each woman’s risk factors should be evaluated individually to determine the appropriate evaluation. In this patient with no significant medical or family history, routine gynecologic care is acceptable.
What is the MOST appropriate next step in management of a 7 cm hypoechoic, thin-walled, ovarian cyst in a 30-year-old woman with left lower quadrant fullness who is actively attempting to become pregnant?
A hypoechoic, thin-walled cyst is a simple cyst. These types of cysts are very likely to be benign in any age group and resolve spontaneously. Expectant management is an option in patients with benign adnexal masses such as simple ovarian cysts.
However, if there is any suspicion for malignancy or if the adnexal mass is causing symptoms, surgical intervention is warranted. Endometriomas and hydrosalpinges can cause issues with fertility and surgical intervention is indicated in those instances.
Characteristics suspicious for malignant ovarian masses include:
wall thickening or septations measuring > 3 mm in width
papillary excrescences
solid components.
This patient likely has a simple cyst with minimal symptoms, and desires to maintain fertility. Simple cysts usually resolve spontaneously. Surgical intervention is only recommended if the patient is symptomatic or if the cyst is greater than 10 cm. Gynecologic oncology referral is not warranted unless there is suspicion for malignancy. Fertility preservation should be paramount in adolescent patients and premenopausal patients who have not completed childbearing.
Observation with repeat ultrasound in 6 months is the treatment of choice for asymptomatic simple cysts and benign-appearing masses measuring up to 10 cm because there is < 1% chance of malignancy, even in postmenopausal women.
Which of the following is the MOST reliable early sonographic evidence of an intrauterine pregnancy?
The yolk sac is a concentric structure that definitively identifies the location of the pregnancy, and its location in the endometrial cavity confirms an intrauterine pregnancy.
This sonographically appears after the double decidual sign. The double decidual sign is defined as 2 concentric echogenic rings surrounding the gestational sac and usually appears around 4–5 weeks; however, it is not the most reliable as it can be confused with a pseudogestational sac.
Which of the following statements is MOST accurate regarding the use of long-acting reversible contraception in adolescents?
Long-acting reversible contraception is considered first-line contraception therapy for adolescents. ACOG recommends that adolescents have access to all contraceptive methods that are approved by the FDA. Ultimately, patient choice should be the principal factor when prescribing contraception to adolescents. ACOG, CDC, and AAP all support the use of LARCs for adolescents given their high efficacy and good safety profile. Evidence shows that 82% of adolescents at risk of unintended pregnancy were using contraception, but only 59% used a highly effective method, including hormonal method or IUD.
IUDs and contraceptive implant should be offered routinely to adolescents. The US MEC classifies IUDs in nulliparous women and adolescents (aged 20 years or younger) as Category 2 and the contraceptive implant as Category 1. Data suggest that LARC use by adolescents remains much lower than in other age groups, although discontinuation for dissatisfaction is no higher in this group than in others. In the Contraceptive CHOICE study, 62% of the 1,054 adolescents and young adults, aged 14–20 years, chose LARC; satisfaction and continuation rates were high. Use of LARC increased substantially in nulliparous women, from 2.1% in 2009 to 5.9% in 2012.
Satisfaction and continuation of LARC methods are high among adolescents. Continuation at 12 months was 81% for LARC methods (75.6% for the copper IUD, 80.6% for the levonorgestrel IUD, 82.2% for the implant) compared with 44% for non-LARC methods (47.3% for DMPA, 46.7% for OCPs, 40.9% for the patch, 31.0% for the ring) among 15–19 year olds.
Concerns with difficulty in placement of IUDs appears to limit providers’ willingness to offer IUDs to nulliparous and adolescent women. Evidence shows high success rates in placement of IUDs during the first attempt, at 95.5%. Uterine perforation rate is no higher in adolescents than in adult women. There is some evidence to show that IUD expulsion rates are higher in adolescents than in older women, but this should not be a deterrent to placement in adolescents.
What is the incidence of urinary tract infection in sexually active young women?
UTI is a frequent diagnosis made in women. The incidence may be stated as 0.5–0.7 per person-year. The most common causative agent is E. coli.
Which of the following is a validated tool to assess the risk of elder abuse?
Screens and Tools to Detect Abuse
BASE (Brief Abuse Screen for the Elderly)
Validated tool to assess the risk of elder abuse
EAI (Elder Assessment Instrument)
To identify victims of elder abuse
CTS (Conflict Tactics Scale)
Validated tool to assess the risk of abuse
CASE (Caregiver Abuse Screen)
To identify abuse of older people by an informal caregiver
Which of the following patients is the BEST candidate to use saline infusion sonography (sonohysterography)?
Tamoxifen is a selective estrogen receptor modulator (SERM) that has a pro-estrogenic effect on the endometrium leading to focal or diffuse thickening (hyperplasia or malignancy). Abnormal uterine bleeding associated with tamoxifen use is most often caused by endometrial polyps.
Sonohysterogram helps in visualizing focal lesions as endometrial polyps or submucosal fibroids by outlining the endometrial cavity. The examination is typically scheduled early in the first 10 days of the menstrual cycle, as focal irregularities in the contour of the secretory endometrium may be mistaken for small polyps or focal areas of endometrial hyperplasia if done later in the menstrual cycle.
Saline infusion sonohysterography better outlines the endometrial cavity and is of great value in outlining focal lesions such as endometrial polyps or submucosal fibroids. The examination is typically scheduled early in the first 10 days of the menstrual cycle. Sonohysterography is contraindicated in active pelvic inflammatory disease (PID). Abnormal uterine bleeding associated with tamoxifen use is mostly caused by endometrial polyps, which can be visualized by sonohysterogram.
Which of the following is a treatment for overactive bladder and is correctly paired with its mechanism of action?
Mirabegron is a β3 adrenergic receptor agonist used to treat overactive bladder. β3 adrenergic receptors facilitate relaxation of the detrusor muscle. Therefore, by stimulating this receptor, symptoms of overactive bladder are ameliorated.
Overactive bladder (OAB) affects 16% of women and is characterized by urinary urgency with or without urinary incontinence and is manifested by increased daytime frequency and nocturia. Increased excitability of the detrusor muscle leads to an exagerated response to stimuli and involuntary contractions.
Overactive bladder can be diagnosed based on patient history and physical exam. A urinalysis should be performed to rule out infection. A voiding diary or urodynamic testing may assist in making the diagnosis of OAB. First line treatment involves behavior modification and lifestyle changes followed by pharmaceutical agents, and lastly, procedure-based interventions.
Which of the following is MOST consistent with a diagnosis of a lactotroph macroadenoma (prolactinoma)?
For pituitary adenomas, a lactotroph macroadenoma is a lesion that is 1 cm or greater in diameter and arises from the anterior pituitary with an increase in serum prolactin above the normal range. A lactotroph microadenoma is a lesion that is less than 1 cm in diameter arising from the anterior pituitary with an increase in serum prolactin above the normal range.
Which of the following is the MOST likely organism associated with chronic IUD use upon its removal?
Most cases of A. israelii have been described in women with an IUD in place for an average of 8 years. It has been implicated in the development of pelvic inflammatory disease (PID), but incidentally diagnosing the bacteria on a Papanicolaou smear in an asymptomatic woman is not grounds for removal.
The IUD should be removed and antibiotics given only when the patient is symptomatic.
Which of the following is the MOST likely organism associated with chronic IUD use upon its removal?
Most cases of A. israelii have been described in women with an IUD in place for an average of 8 years. It has been implicated in the development of pelvic inflammatory disease (PID), but incidentally diagnosing the bacteria on a Papanicolaou smear in an asymptomatic woman is not grounds for removal.
The IUD should be removed and antibiotics given only when the patient is symptomatic.
Which of the following symptoms is necessary to diagnose female orgasmic disorder?
DSM-V defines the criteria for female sexual dysfunction. The following are recognized as female sexual dysfunctions:
female sexual interest/arousal disorder
female orgasmic disorder
genito-pelvic pain/penetration disorder
substance/medication-induced sexual dysfunction
other specified sexual dysfunction
other unspecified sexual dysfunction
DSM-V Criteria for Female Orgasmic Disorder
Symptoms present for a minimum of 6 months
Marked delay in, marked infrequency of, or absence of orgasm
OR markedly reduced intensity of orgasmic sensations in almost all occasions of sexual activity
Cause clinically significant distress
Which of the following is the MOST common cause of vaginitis in the pediatric population?
Group A beta-hemolytic Streptococcus (GAS) is the most common infectious agent found in prepubertal females. GAS is found in 7% to 20% of pediatric girls with vulvovaginitis and is treated with amoxicillin 40 mg/kg TID for 10 days.
Infectious vulvovaginitis often presents with a malodorous, yellow or green purulent discharge and vaginal cultures are routinely obtained. Children may transmit respiratory flora from the nose and oral pharynx to the vulva. These respiratory pathogens include: GAS (Streptococcus pyogenes), Staphylococcus aureus, Haemophilus influenzae, Streptococcus pneumoniae, Neisseria meningitidis, and Moraxella catarrhalis.
Infectious Causes of Vulvovaginitis in Prepubertal Females
Organism Incidence
Group A beta-hemolytic Streptococcus 20%
Candida 3–4%
Gardnerella vaginalis 4.2%
Neisseria gonorrhoeae Rare
What is the EARLIEST age at which a patient with no risk factors can receive the meningococcal vaccine?
The appropriate age to start vaccinations to prevent meningococcal disease is 11–12 years old. A booster dose is required at the age of 16–18 years during the ages when adolescents are at highest risk of meningococcal disease.
Children with coexisting conditions that result in immunosuppression, such as asplenia or HIV, can receive the vaccine earlier.
BP stages?
Blood Pressure Category Systolic Diastolic
Normal < 120 < 80
Elevated 120–129 < 80
Stage 1 130–139 80–90
Stage 2 >140 >90
Blood pressure levels need to be reproducible in order to diagnose a patient with hypertension. Causes for secondary hypertension should be evaluated in patients who are resistant to treatment or were diagnosed with hypertension at a young age.
A person can be considered to have fibromyalgia if:
1) He or she has a WPI (widespread pain index score) ≥7 and symptom severity scale (SS) score ≥5 or a WPI 3 to 6 and SS score ≥9
2) Symptoms have been present for at least 3 months
3) He or she does not have a disorder that would otherwise explain the pain
A patient with pain in both upper and lower arms, shoulders, chest, and abdomen and SS score of 9 has a WPI of 7 and therefore meets the diagnostic criteria.
Symptom areas include:
Neck
Left jaw
Right jaw
Left shoulder
Right shoulder
Left upper arm
Right upper arm
Left lower arm
Right lower arm
Chest
Abdomen
Upper back
Lower back
Left buttock
Right buttock
Upper left leg
Lower left leg
Upper right leg
Lower right leg
These are added up to get a widespread pain index (WPI) score.
Symptoms are then scaled and added up:
0 no problem
1 mild or intermittent
2 moderate, often present
3 severe, constant
A nulliparous woman who utilizes an intrauterine device presents for an annual exam. She has been very happy with the device and indicates no complaints today. The results of her pap indicate she has Actinomyces as seen on the cytology. How should this be managed?
In general, Actinomyces is a rare infection. However, about 7% of asymptomatic women with an intrauterine device may have Actinomyces found on cytology. An incidental finding of Actinomyces on cytology does not need to be treated if there are no symptoms of pelvic inflammatory disease. The IUD can be left in place and no antibiotics are warranted.
If pelvic actinomycosis were to occur, it is characterized by granulomatous pelvic abscesses. The prevalence of pelvic actinomycosis is estimated to be less than 0.001%.
Which of the following is TRUE regarding substance abuse cessation?
There are many potential methods for discontinuation of drug use. One of the more controversial topics recently is the notable improvement in compliance noticed with incentive-based rehabilitation therapy. Also useful is the tracking of these patients with drug testing, which has been demonstrated with nicotine.
Behavioral support has been shown to be beneficial in treating substance abuse and has been noted as useful in tobacco cessation when treating prior drug users.
Answer A: Behavioral support has been found to significantly improve compliance with substance abuse. Tobacco cessation is improved with these methods.
Answer C: Financial incentives to patients actually provides significant cost savings to the government and to the patient if utilized adequately and, despite the dislike of this idea by others, it may be a future direction for substance abuse.
Answer D: Immediate gratification offers great potential in cessation programs, creating improved reliability for patients undergoing treatment.
Answer E: Drugs have different levels of addictiveness. Cocaine and heroin are cited as two of the more addictive ones.
A 42-year-old woman presents to the office for routine gynecologic care. Her BMI is 32 and she is otherwise healthy. She inquires about weight loss strategies. With regard to physical activity, which of the following would be considered adequate?
Exercise is an important component of weight loss, weight maintenance, and overall health maintenance. The CDC recommends that people exercise at a vigorous rate for a total of 75 minutes per week, or at a moderate rate for 150 minutes per week.
CDC Recommendations According to Exercise Type and Duration
Vigorous exercise
(75 mins per week)
Jogging/Running
Swimming laps
Roller boarding
Most competitive sports
Jumping rope
Moderate exercise
(150 mins per week)
Walking (15-minute mile)
Yard work
Shovelling snow
Actively playing with children
Biking at a casual pace
The best answer is jogging (10-minute mile) for 30 minutes, 3 times per week, as this is vigorous exercise for a total of 90 minutes, which is higher than the 75 minutes recommended.
A 60-year-old postmenopausal woman presents with nipple discharge from the right breast for the past 6 months. No mass is palpated upon exam. Which of the following characteristics would be MOST consistent with nipple discharge representing malignancy in this patient?
Women who present with concerning nipple discharge should certainly have an ultrasound performed. Women over the age of 30 should also have a mammogram performed. Any intraductal findings on imaging should be further evaluated with biopsy. Additionally, if the discharge is bloody or uniductal, regardless of imaging results, the patient should be referred to a surgeon for further evaluation, which may include galactography or ductoscopy.
Pathologic nipple discharge characteristics
uniductal
unilateral nipple
expressed without manipulation (spontaneous discharge)
persistent
associated with a mass or skin changes
bloody in color
Characteristics of nipple discharge that are less concerning for intrinsic disease of the breast
bilaterality
multiductal
non-spontaneous
milky or green color
otherwise healthy premenopausal pt arrives w/ milky like nipple discharge. why
indicative of galactorrhea and is usually multiductal and bilateral. In premenopausal women, pregnancy should first be ruled out. Galactorrhea is not a disease intrinsic to the breast; however, it cannot be considered completely harmless since it is caused by hyperprolactinemia, which is seen in a number of health issues; these include hypothyroidism and prolactin-secreting tumors. Additionally, medications that inhibit dopamine can cause hyperprolactinemia. Poor fitting brassieres that cause chronic breast stimulation can also be to blame.
A 20-year-old woman presents to the emergency department after a sexual assault 10 hours earlier at a college party. She did not know the assailant. Which of the following prophylactic measures is recommended by the CDC?
All patients who experience a sexual assault should be offered prophylaxis for sexually transmitted infections (STIs).
The CDC recommends treatment for gonorrhea, chlamydia, hepatitis B, and trichomonas as outlined in the table below.
CDC Recommendations for STI Prophylactic Treatment
Pathogen Treatment (prophylaxis)
Chlamydia Doxycycline 100 mg PO BiD for 7 days OR azithromycin 1 g PO
Gonorrhea Ceftriaxone 500 mg IM for weight <150 kg OR 1000 mg for wight >150 kg
Trichomoniasis Metronidazole 2 g PO
Hepatitis B Initiate hepatitis B vaccine series (1st dose at presentation)
HIV postexposure prophylaxis is recommended if the assailant is known to be HIV positive, and other prophylaxis depends on the individual situation for each patient.
In the United States, which of the following contributes to the highest number of deaths during pregnancy?
Cardiovascular conditions are the leading cause of pregnancy-related deaths in the U.S. Cardiovascular disease affects approximately 1–4% of the nearly 4 million pregnancies in the United States each year. This rising trend in maternal deaths related to cardiovascular disease appears to be due to acquired heart disease. Cardiovascular diseases constitute 15.3% of U.S. pregnancy-related deaths.
Per ACOG PB 212, in the United States disease and dysfunction of the heart and vascular system, collectively referred to as “cardiovascular disease”, is now the leading cause of death in pregnant women and women in the postpartum period. Cardiovascular disease accounts for 4.23 deaths per 100,000 live births. In fact, some of the most recent data indicate that the number of pregnancy-related deaths secondary to cardiovascular diseases is even higher, up to 26.5% of U.S. pregnancy-related deaths.
Which of the following represents the incidence of vulvovaginal lichen planus in the general population?
Bottom Line: Lichen planus can have cutaneous, oral, and vulvovaginal manifestations. Although lichen planus is only found in 1% of the population, oral lichen planus coexists with vulvovaginal lichen planus 20–25% of the time.
TrueLearn Insight: Remember that lichen Planus is characterized by Pruritic Purple Polygonal Papules. This can help distinguish it from lichen sclerosus and lichen simplex chronicus.
Which of the following represents the incidence of vulvovaginal lichen planus in the general population?
Bottom Line: Lichen planus can have cutaneous, oral, and vulvovaginal manifestations. Although lichen planus is only found in 1% of the population, oral lichen planus coexists with vulvovaginal lichen planus 20–25% of the time.
TrueLearn Insight: Remember that lichen Planus is characterized by Pruritic Purple Polygonal Papules. This can help distinguish it from lichen sclerosus and lichen simplex chronicus.
A 28-year-old woman has been taking leuprorelin for 6 months because of a history of endometriosis and pain. Which of the following medications is approved by the Food and Drug Administration as add-back therapy for patients on leuprorelin?
Leuprorelin (Lupron) is a common therapy for endometriosis. Prolonged use (i.e., greater than 6 months) can lead to GnRH-associated bone loss. Add-back therapy offers the benefit of stopping bone loss while not reducing the efficacy of the medication in the treatment of endometriosis-related pain.
Norethindrone 5 mg each day is an FDA-approved medication for add-back therapy in patients taking leuprorelin for more than 6 months
Which of the following is the CORRECT incidence of perinatal depression?
Perinatal depression includes both major and minor depressive episodes that occur during pregnancy or the first 12 months postpartum. It is extremely common and affects approximately 1 in 7 women (14.3%).
Which of the following is the approximate lifetime risk of ovarian cancer in someone who tests positive for BRCA1 mutation?
Hereditary syndromes, in particular BRCA germline mutations, make up approximately 10% of ovarian cancers that are diagnosed but only 3–5% of breast cancer. BRCA1 is found on chromosome 17 and BRCA2 on chromosome 13.
The lifetime risk of ovarian cancer is 39–46% with BRCA1.Hereditary syndromes, in particular BRCA germline mutations, make up approximately 10% of ovarian cancers that are diagnosed but only 3–5% of breast cancer. BRCA1 is found on chromosome 17 and BRCA2 on chromosome 13.
The lifetime risk of ovarian cancer is 39–46% with BRCA1. risk associated with BRCA2 (12–20%).
What dose of calcium should a 52-year-old otherwise healthy woman be taking?
Calcium and Vitamin D are important for the development of bones. A deficiency in calcium can lead to osteoporosis, whereas a deficiency in vitamin D can result in softer bone that is poorly mineralized. This poorly mineralized bone develops into rickets in children and osteomalacia in adults. For this reason, the Institution of Medicine has issued recommendations for individuals starting at age 9 regarding the daily amount of calcium (mg/day) and vitamin D (international units/day).
As you age, you need less calcium in your diet and more vitamin D.
The Institution of Medicine’s Recommendations for Daily Intake of Calcium and Vitamin D
Age Calcium Allowance (Recommended Dietary) Vitamin D (Recommended Dietary)
9–18 1,300 mg/day 600 IU/day
19–50 1,000 mg/day 600 IU/day
51–70 1,200 mg/day 600 IU/day
71+ 1,200 mg/day 800 IU/day
A 35-year-old woman presents for her well-woman exam. She has no complaints. On speculum exam, a smooth, symmetrical, white, rounded elevation is noted in the cervical tissue with small, thin blood vessels coursing over the surface. What is the MOST LIKELY diagnosis?
Nabothian cysts, also known as epithelial inclusion cysts, are benign cervical lesions that are often smooth, clear, and white or yellow in appearance. They may also have small, branching blood vessels that course over the lesion. These cysts occur during squamous metaplasia, when squamous epithelium covers over glandular, columnar cells; mucoid material from these glandular cells continues to accumulate beneath the squamous cells, leading to the appearance of a cyst.
Asymptomatic nabothian cysts typically require no therapy but do have the potential to grow. Large nabothian cysts can cause vaginal fullness or pain. At times, these cysts can be obstructive, making Pap testing or cervical examination difficult. Treatment typically consists of ablation with electrocautery or excision.
A 35-year-old woman presents for her well-woman exam. She has no complaints. On speculum exam, a smooth, symmetrical, white, rounded elevation is noted in the cervical tissue with small, thin blood vessels coursing over the surface. What is the MOST LIKELY diagnosis?
Nabothian cysts, also known as epithelial inclusion cysts, are benign cervical lesions that are often smooth, clear, and white or yellow in appearance. They may also have small, branching blood vessels that course over the lesion. These cysts occur during squamous metaplasia, when squamous epithelium covers over glandular, columnar cells; mucoid material from these glandular cells continues to accumulate beneath the squamous cells, leading to the appearance of a cyst.
Asymptomatic nabothian cysts typically require no therapy but do have the potential to grow. Large nabothian cysts can cause vaginal fullness or pain. At times, these cysts can be obstructive, making Pap testing or cervical examination difficult. Treatment typically consists of ablation with electrocautery or excision.
A 27-year-old woman presents to the clinic with a mass in her right breast for the past 6 months. She denies fevers or chills, nipple discharge, and any recent trauma to the breast. On examination, the physician notes a 3-cm smooth, firm, tender mass in the right upper quadrant of her right breast. The left breast is unremarkable. What is the BEST next step in management?
This patient presents with a palpable mass for the past 6 months. The basic differential for this presentation includes cyst, benign solid mass, and a malignancy.
This patient’s presentation, in the context of her age, is most consistent with a benign cyst, which is classically characterized as: smooth, firm, and sometimes tender with sharp borders. Cysts can also present as ill-defined masses with various textures. Ultimately, physical exam cannot distinguish a benign cyst from more worrisome pathology. In this age group (less than 30), the best next step in management is to perform an ultrasound.
A woman who is less than 30 years old and presents with a palpable breast mass can be reasonably evaluated with ultrasound as the first step in management after you have performed a thorough history and physical (above she’s observed already for 6 months). Observation is also reasonable for 1–2 menstrual cycles with a young patient who presents with a non-concerning mass (based on history and physical exam) that has been present for only a short time.
Cysts can be further classified based on ultrasound appearance. Simple cysts are well-circumscribed and anechoic (do not have internal echoes), and do not have solid components or internal vascularization/Doppler flow. Posterior wall enhancement is seen with simple cysts. These simple cysts can either be watched or aspirated if symptomatic. Complex cysts have thick walls and/or septa. Additionally, they may have solid components and do not have posterior wall enhancement. Complicated cysts do not have solid components, thick walls, thick septa, or vascular flow; however, they do have internal echoes due to debris within the cyst.
The treatment for complex or complicated cyst is based on its etiology. The basic differential for a complex or complicated cyst includes:
abscess
hematoma
fat necrosis
galactocele
duct ectasia
malignancy
cyst on breast. difference btwn core needle biopsy vs fine needle aspiration
Answer A: Core needle biopsy is a method of acquiring specimen for histologic evaluation. A large-bore (12–16 gauge) needle is used. Palpation or imaging assists the practitioner in placement of the needle in order to sample the lesion. When performing a core needle biopsy, the physician is also able to place a clip by the lesion in order to mark it—this can be helpful in the case of repeat imaging or if further surgery is needed. With the patient above, core needle biopsy may eventually be needed but is not the best next step.
Answer B: Fine needle aspiration (FNA) allows for cytologic evaluation. Fine needle aspiration uses a small-bore needle (21–25 gauge). Disadvantages to FNA include: 1) it requires that the pathologist has extensive experience in interpretation of such a specimen; 2) a significant number of specimens will result as “non-diagnostic”; 3) the procedure has a relatively high false negative rate; 4) given that FNA provides a cytologic specimen, it cannot definitely distinguish between in situ and invasive cancer.
Some practitioners may argue that an attempt at cyst aspiration is a reasonable first step in management if one suspects a breast cyst; after all, if aspiration yields a fluid that is not bloody and the mass disappears, the fluid may be discarded without need for pathologic evaluation and the issue may be resolved. However, the American Congress of Obstetrics and Gynecology recommends imaging before aspiration. The rationale behind this approach may stem from the fact that it is difficult to distinguish a cystic lesion from a solid mass based on physical exam alone. Aspiration is an invasive procedure relative to ultrasound and in the case of a solid lesion, the sample obtained is often difficult to pathologically interpret, often requiring further evaluation (re-biopsy) with other approaches (such as core needle biopsy). Therefore, an evaluation of the lesion with imaging prior to an invasive procedure is arguably most appropriate.
when would you do breast US vs mammo+US if a breast cyst is palpated during exam?
Due to the increased density of the breast, mammogram is generally less useful for evaluation of breast masses in women less than 30 years old. If the patient had presented at the age of 30 or older, the best next step in management would have been to obtain a diagnostic mammogram with ultrasound to follow (it is also reasonable to order the ultrasound and mammogram at the same time).
Which of the following muscles are MOST likely to play a role in vaginismus?
The muscles that comprise the levator ani group (pubococcygeus, iliococcygeus, and puborectalis), and possibly others muscles, are likely involved in vaginismus.
According to the Diagnostic and Statistical Manual of Mental Disorders, 5th edition (DSM-5), vaginismus is listed as a “genito-pelvic pain/penetration disorder”. Traditionally, vaginismus has referred to involuntary contraction of the muscles of the pelvic floor surrounding the vaginal orifice. However, there is no consensus on exactly which muscles are involved in vaginismus and studies that name specific muscle groups as the source of spasm do not describe the method used to arrive at their conclusions. Furthermore, evidence of electromyography (EMG) studies does not definitively support muscle spasms as the etiology of vaginismus.
Which of the following treatments is appropriate for the management of hot flashes in patients who have an intact uterus?
Vasomotor symptoms are a bothersome component of the postmenopausal changes experienced by women. They can persist for up to 10 years without any relief.
Hormonal therapy is an option for managment. Progesterone must be given in conjunction with estrogen in women with an intact uterus. The risk of endometrial hyperplasia increases in women who are placed on unopposed estrogen therapy.
Progesterone alone is not indicated for first-line treatment of vasomotor symptoms. Progesterone alone may improve vasomotor symptoms but there are limited data on the safely of progestin alone and in the WHI study there was an increased risk of breast cancer in the progesterone-plus-estrogen arm but not in the estrogen-alone arm.
Which of the following regimens is MOST appropriate for cleaning the vaginal probes between patients?
High-level disinfection is required with vaginal probes, and 70% alcohol or soaking it for 2 minutes in 500 ppm chlorine is an example of a high-level disinfectant.
The CDC has approved several different measures for cleaning vaginal probes between uses. They recommend condoms or probe covers for each use, but failure rates have been reported as high as 65–81% and therefore cleaning with high-level disinfectant is important. Cleaning in between is imperative for patient safety.
Recommended Regimen for Cleaning Vaginal Probes
Wipe gel from the probe
Rinse probe with soap and water
Soak probe for 2 minutes in 500 ppm chlorine
Rinse again
Air dry
According to most recent data, which of the following is the prevalence of obesity in adults in the United States?
According to data collected for the National Health and Nutrition Examination Survey (NHANES), between 2017 and March 2020, the prevalence of obesity in adults in the United States is approximately 41.9%.
Which of the following is the MOST common side effect of the Paragard (copper/TCu380A) intrauterine contraceptive device?
The biggest patient complaint after IUD placement is heavier, crampier periods the first year after placement.
The most common side effects of Paragard use are dysmenorrhea as well as heavier, longer periods with spotting between periods. For most women, these side effects diminish after the first 2 to 3 months. If switching from a hormonal birth control method, periods may seem heavier. Although menstrual blood loss has been noted to increase in copper intrauterine device (IUD) users, the reported decrease in hemoglobin concentration is between 0.36 and 0.94 g/dL over a 12-month period. Nonsteroidal anti-inflammatory medications have been shown to improve the amount of bleeding and cramping in these patients.
The copper T380A IUD is a T-shaped device of polyethylene wrapped with copper wire around the stem and arms. Studies indicate that the copper IUD exerts its contraceptive effects primarily by preventing fertilization through inhibition of sperm migration and viability.
The U.S. Food and Drug Administration (FDA) has approved use of the copper IUD for up to 10 continuous years, during which time it remains highly effective. It has a reported failure rate at 1 year of 0.8 per 100 women, and a 10-year failure rate comparable with that of female sterilization (1.9 per 100 women over 10 years).
Side Effects and Complications of Paragard IUD Placement
dysmenorrhea
menorrhagia
intermenstrual bleeding
expulsion
perforation
failure resulting in pregnancy
pelvic inflammatory disease
difficult removal
An 8-year-old girl presents for recurrent vaginal itching and a foul-smelling discharge. She has noted intermittent vaginal spotting. An external physical exam reveals no distortion of the labial architecture. What is the MOST LIKELY diagnosis?
A foreign body is a common cause of acute and chronic recurrent vulvovaginitis and manifests clinically as vaginal discharge, intermittent bleeding or spotting, and/or a foul smelling odor. The most common foreign body that is found is toilet paper, but can also be hair bands, small toys, batteries, etc. The foreign body should be removed which can be accomplished with a swab or irrigation of warm water. It is recommended to apply topical anesthetic agent to the introitus such as viscous xylocaine jelly. Examination under sedation and/or anesthesia may be necessary for extraction of larger foreign bodies and those that cannot be removed with irrigation.
Of note, if the patient has pain and a gray watery discharge, an intravaginal battery should be suspected, and emergency removal under general anesthesia is warranted. After removal, the vaginal mucosa should be carefully inspected by vaginoscopy to determine the depth of the burn. Depending upon the depth of the burn, assessment of the bladder or rectal mucosa may also be indicated.
Which of the following is MOST diagnostic for adenomyosis?
The diagnosis of adenomyosis is made after histologic confirmation of endometrial glands and stroma that are noted to extend into the myometrium, usually on pathologic review of a uterine specimen after hysterectomy. The most common symptoms of adenomyosis include heavy menstrual bleeding and dysmenorrhea.
Imaging may be helpful in evaluation for adenomyosis. Ultrasound imaging may demonstrate an enlarged uterus where the posterior wall is thicker than the anterior wall, and multiple bands of alternating bright and dark echoes.
Which of the following thrombophilias requires pharmacologic prophylaxis during pregnancy and the postpartum period given the high risk of VTE, regardless of family and personal history of VTE?
Even in the absence of other risk factors, women who are known to be homozygous for the factor V Leiden mutation or prothrombin gene mutation should receive pharmacologic prophylaxis during pregnancy and the postpartum period, given the high risk of VTE.
Current ACOG recommendations regarding thromboprophylaxis for pregnancies complicated by inherited thrombophilias state that low-risk thrombophilias without previous personal history of VTE do NOT require anticoagulation during pregnancy and the postpartum period. All the answer choices listed above are low-risk thrombophilias except for homozygous factor V Leiden mutation.
Low-risk thrombophilias without previous VTE:
factor V Leiden heterozygous
prothrombin G20210A heterozygous
protein C deficiency
protein S deficiency
High-risk thrombophilias without previous VTE:
factor V Leiden homozygous
prothrombin G20210A homozygous
heterozygous for factor V Leiden and prothrombin G20210A mutation
antithrombin deficiency
The prevalence of the factor V Leiden mutation in European populations is approximately 5%. The mutation renders factor V Leiden refractory to proteolysis by activated protein C. Women who are heterozygous for factor V Leiden have been observed to account for approximately 40% of cases of VTE during pregnancy.
The treatment of factor V Leiden thrombophilia has not been shown to decrease the risk of continued recurrent miscarriages. There are inconsistent associations between any inherited thrombophilias and recurrent pregnancy loss or stillbirth. A Cochrane review also concluded that there is insufficient evidence to support the use of anticoagulants (aspirin or low-molecular-weight heparin) in women with recurrent pregnancy loss and an inherited thrombophilia.
A 35-year-old gravida 2, para 2 woman presents to the office because of a history of frequent urinary tract infections. She asks the best way to prevent future recurrences. Which of the following is the MOST effective way to prevent recurrences?
More than one-half of all women will have a urinary tract infection (UTI) in their lifetime, and approximately 3%–5% will be diagnosed with “recurrent UTI,” defined as ≥2 recurrences in 6 months or ≥3 in 1 year. There are three evidence-based interventions for recurrent UTI:
- The MOST effective strategy to reduce the incidence of recurrences is prophylactic or intermittent antibiotics, which can reduce the risk by 95%. A culture must be obtained prior to prescribing prophylaxis. Below are options for prophylaxis based on antimicrobial sensitivities. Nitrofurantoin is considered first line based on expert opinion.
- Proven therapies include hydration with an additional 1.5 L of water daily, which was proven to decrease cystitis occurences in a randomized clinical trial of premenopausal women with at least three UTIs in the past year.
- Vaginal estrogen for postmenopausal women is proven in small randomized clinical trials to decrease the risk of recurrent UTI. There are varying formulations and dosing schedules that can be prescribed. One trial studied 0.5 mg estriol cream nightly for 2 weeks then twice weekly for 8 months.
Which of the following is a treatment choice for female sexual interest and arousal disorder in a premenopausal woman?
Flibanserin is a serotonin receptor agonist/antagonist approved by the FDA as a treatment option for women with female sexual interest and arousal disorder. It can be prescribed to premenopausal women without depression suffering from hypoactive sexual desire. Women must also be counseled appropriately about the risks of concurrent alcohol use during treatment: Alcohol use during treatment leads to an increased risk of hypotension and syncope.
The American Psychiatric Association’s Diagnostic and Statistical Manual of Mental Disorders, Fifth Edition (DSM-5) outlines four classifications of female sexual dysfunction:
female sexual interest/arousal disorder
female orgasmic disorder
genito-pelvic pain and penetration disorder
substance/medication-induced sexual dysfunction
Which of the following lists the CORRECT sequence of the layers of the bladder from inner to outer?
Transitional epithelium can be found lining the urinary bladder, ureters, superior aspect of the urethra, and gland ducts of the prostate. It is the layer of the bladder that is exposed to the lumen. Deep to this epithelium is the lamina propria, followed by the submucosa. The transitional epithelium, lamina propria, and submucosa collectively make up the mucosa. Deep to the mucosa is the detrusor muscle, followed by the adventitia, which contains the vessels that feed and drain the bladder.
What is the lifetime incidence of completed or attempted rape for women living in the United States?
The lifetime incidence of completed or attempted rape for women in the United States is roughly 1 in 5, or 19% per ACOG and the CDC.
Forty-seven percent of sexual assault victims reported the perpetrator was a current or former intimate partner, and 45% reported the perpetrator was an acquaintance. One-third of sexual violence victims seek some sort of medical care as a result of the assault.
Knowing the incidence of sexual assault and being aware of the prevalence may help the healthcare provider empower women and provide proper care and support. Knowledge of local laws will also help evaluate a sexual assault victim.
A 22-year-old woman presents for worsening hirsutism over the past 6 months as well as temporal balding. On exam, the physician notes a Ferriman–Gallwey score of 18. Lab tests return and are significant for total testosterone of 62 ng/dL, a normal 17-hydroxyprogesterone level, and a DHEA-S level of 850 mcg/dL. What is the next BEST step in the workup of her hirsutism?
Hirsutism is defined as excessive male-pattern hair and facial and body hair that affects 5%–10% of reproductive-age women.
The major circulating hormones in women (in descending order) are:
DHEA-S
DHEA
androstenedione
testosterone
dihydrotestosterone (DHT)
Evaluation of hirsutism includes measurement of total testosterone, DHEA-S, and 17-hydroxyprogesterone. DHEA-S is derived almost exclusively from the adrenal glands, and the upper limit of normal is typically 350 mcg/dL but can vary between laboratories. DHEA-S levels greater than 700 mcg/dL may indicate a rare androgen-secreting adrenal tumor such as an adrenal adenoma or carcinoma.
An adrenal CT is the imaging study of choice due to excellent sensitivity and specificity in detecting these tumors.
Answer A: Adrenocorticotropic hormone stimulation test would be an appropriate next step in the workup of a patient with an elevated 17-hydroxyprogesterone level and concern for nonclassic congenital adrenal hyperplasia (NCCAH). This test should be performed when the 17-hydroxyprogesterone level is between 200 and 800 ng/dL. An ACTH stimulation test is thus performed, and the 17-hydroxyprogesterone level is rechecked. If it is over 1500 ng/dL, NCCAH is diagnosed.
Answer C: Observation is not an appropriate next step because her DHEA-S level is elevated above normal, which may signify a possible androgen-secreting adrenal tumor.
Answer D: Evaluating for an adrenal adenoma is best accomplished by an adrenal CT. A pelvic MRI may be useful if evaluating for pelvic pathology, including malignancy or concern for ovarian masses if transvaginal ultrasonography is inconclusive; however, it is not the next best step when assessing adrenal pathology.
Answer E: The gold-standard imaging modality for adrenal masses is an adrenal CT. Ultrasonography has unacceptably low sensitivity and specificity in this scenario.
What is the sexual disorder described by tension, pain, or burning with penetration?
This patient with pain with penetration has genito-pelvic pain and penetration disorder.
Vaginismus and dyspareunia are now combined into genito–pelvic pain and penetration disorder. This disorder can be lifelong or acquired.
Symptoms of Genito–Pelvic Pain and Penetration Disorder
Tightening of the vaginal muscle with decreased ability or inability to accommodate penetration
Tension, pain, or burning felt when penetration is attempted
Decrease in or no desire to have intercourse; avoidance of sexual activity
Intense phobia or fear of pain
Symptoms must have persisted for at least 6 months and be distressing to the patient.
Pelvic floor physical therapy is recommended for the treatment of genito–pelvic pain and penetration disorders to restore muscle function and decrease pain. Intravaginal prasterone, low-dose vaginal estrogen, and ospemifene can be used in postmenopausal women for the treatment of moderate-to-severe dyspareunia that is due to genitourinary syndrome of menopause. Lubricants, topical anesthesia, and moisturizers may help reduce or alleviate dyspareunia.
All of the following are signs of hypotonic fluid overload after use in a hysteroscopic procedure EXCEPT:
Distention fluid used in the expansion of the uterine cavity requires pressure between 45 and 80 mm Hg to be effective. Intrauterine pressure greater than 100 mm Hg increases the risk for intravasation of the distention medium because the average mean arterial pressure for an adult female is typically around 100 mm Hg. Once hypotonic fluid intravasates, it is metabolized and free water is left. This causes dilution of electrolytes that leads to hyponatremia, hypokalemia, etc. Dilution leads to respiratory depression and seizures.
What is the lifetime incidence of intimate partner violence for women living in the United States?
The lifetime incidence of intimate partner violence for women in the United States is estimated at 22%–39%.
Intimate partner violence can have several meanings, but it includes actual or threatened harm—psychological, physical, or sexual. It can occur among heterosexual or same-sex couples.
The lifetime incidence of intimate partner violence for all women in the world is estimated at 69%.
Which of the following is a step in the “5A’s” algorithm used to promote smoking cessation?
The 5A’s for smoking cessation: Ask, Advise, Assess, Assist, Arrange
Cigarette smoking is the leading preventable cause of mortality. Smokers who quit smoking reduce their risk of developing and dying from tobacco-related diseases. There is a simple five-step algorithm called the “5 A’s” (Ask, Advise, Assess, Assist, Arrange) that can be implemented to aid with smoking cessation. Five “A’s” for assessing for tobacco use and addressing smoking cessation: Ask: Discussing with the patient if they ever thought about quitting tobacco. This would involve asking every patient that is checked into the office their tobacco-use status and document it. This repeated assessment should be done at each visit. Advise: Tie tobacco use to current health/illness, and/or its social and economic costs, motivation level/readiness to quit, and/or the impact of tobacco use on children and others in the household. Discussing with the patient the benefits of tobacco cessation. Assess: Determine the patient’s willingness to quit within the next 30 days. Assist: Providing aid for the patient to quit. Arrange: Schedule follow-up contact, either in person or by telephone. Follow-up contact should occur soon after the quit date, preferably during the first week. A second follow-up contact is recommended within the first month. Schedule further follow-up contacts as indicated. Congratulate success during each follow-up. If tobacco use has occurred, review circumstances and elicit recommitment to total abstinence. Remind the patient that a lapse can be used as a learning experience. Identify problems already encountered and anticipate challenges in the immediate future. Assess pharmacotherapy use and problems. Consider use or referral to more intensive treatment.
Which of the following is the MOST common type of kidney stone?
The most common type of kidney stone is composed of calcium oxalate, in roughly 70% of cases.
Calcium phosphate stones are next, comprising approximately 10%. The calcium-based stones (80% of all stones) are the easiest to see on a non-contrast CT scan—the diagnostic image of choice in this setting.
Uric acid stones tend to be more radiolucent and not easily imaged with the other imaging forms, such as abdominal X-ray.
The MOST common findings of sexual assault include which of the following?
Sexual assault affects one in eight women. Injuries sustained depend on the severity of the attack.
The most common injuries associated with sexual assault are abrasions of the head, neck, and arms. Genital injuries accompanied with bleeding and pain also occur. Genital findings are erythema and small tears of the vulva, perineum, and introitus. If there was oral penetration, injuries to the mouth and pharynx, tearing of the frenulum, and broken teeth may be present.
All areas penetrated should be thoroughly inspected and samples obtained.
What is the BEST initial management of a palpable Bartholin’s cyst in a postmenopausal woman?
Bartholin gland cysts are rare in postmenopausal women. There is a chance that this lesion could represent a carcinoma, and some pathology should be analyzed.
However, the incidence of Bartholin gland carcinoma is still quite low—roughly 0.114 per 100,000 woman-years in postmenopausal women. A biopsy is therefore warranted to avoid a larger procedure. The detection rate from a biopsy seems to be sensitive. If the biopsy is concerning for carcinoma, a referral to gynecology oncology should be made.
Which of the following structures may be affected by 5a-reductase deficiency?
In 5-alpha reductase deficiency, the internal structures (epididymis, seminal vesicles, ejaculatory ducts, and vas deferens) form normally, while the external genitalia, prostate, and urethra develop abnormally.
When a patient has 5a-reductase deficiency, it is often diagnosed at birth. It is an autosomal recessive disorder with 46, XY karyotype. It is the result of impaired virilization during embryogenesis secondary to defective conversion of testosterone (T) to dihydrotestosterone (DHT). External structures are unable to virilize normally as they require the conversion of T to DHT to form. Classically, there is generally a fusion of the labial folds with or without clitoromegaly. There is usually normal male breast/chest development, helping to distinguish it from androgen insensitivity syndrome.
The internal structures are stimulated by testosterone and generally develop normally. These include the epididymis, vas deferens, seminal vesicles, and ejaculatory ducts. External structures develop abnormally, including the external genitalia (penis, scrotum), urethra, and prostate.
Which of the following medications is approved by the Food and Drug Administration (FDA) for the treatment of vasomotor symptoms?
Paroxetine is the only FDA-approved nonhormonal treatment for vasomotor symptoms
Vasomotor symptoms are a bothersome component of the postmenopausal changes experienced by women. They can persist for up to 10 years without any relief. Hormonal therapy in the form of estrogen is usually indicated to manage these symptoms; however, not everyone is a candidate for hormonal therapy or they do not desire hormonal therapy.
Paroxetine and venlafaxine are the best-studied nonhormonal drugs for the treatment of vasomotor symptoms, but the FDA has approved only the 7.5-mg dose of paroxetine for this purpose. Unlike with the larger doses used to treat psychiatric conditions, patients who wish to discontinue the use of paroxetine will not need to taper the dose.
Other nonhormonal drugs such as methyldopa, gabapentin, and clonidine have performed successfully in research trials; however, because of the side effect profiles, they are not recommended as first-line treatments.
A 29-year-old woman presents for a routine annual visit. She has no medical or surgical history. She reports a regular menstrual cycle and is not interested in contraception. She is sexually active with one partner and is in a same-sex relationship. Which of the following may result in sub-standard care for this patient?
Lesbian and bisexual women face healthcare disparities that include but are not limited to concerns about confidentiality, discriminatory sentiments, restricted access to care and health insurance, and often an insufficient understanding of the health risks they face. It is the responsibility of the physician to make the office environment welcoming to such patients, to ensure that the best and most efficient healthcare is delivered. Examples of measures that can facilitate this include providing educational materials, making yourself available as a resource, and using neutral language during the history and physical examination.
Although the subject of mental health remains a sensitive topic, physicians should include questions concerning it in their history and physical because women who identify as lesbian are more likely to admit to having depression and/or to taking antidepressants.
A patient with a documented intrauterine pregnancy at 6 weeks undergoes surgery for ovarian torsion complicated by emergent salpingo-oophorectomy and removal of the corpus luteum. During which period is progesterone supplementation recommended for this patient?
At approximately 8 weeks of gestation the progesterone production begins to shift from the corpus luteum to the placenta. When a corpus luteum, or ovary that was suspected to have the corpus luteum, is removed progesterone supplementation should continue until approximately 10 weeks. After 10 weeks the progesterone is made at the placental level and supplementation is no longer indicated.
Without supplemental progesterone this pregnancy will likely abort.
Progesterone supplementation is required to maintain an early pregnancy through 10 weeks when a corpus luteum has been removed.
TrueLearn Insight: There are many different uses for progesterone in pregnancy. In the setting of luteal-phase deficiency or removal of a corpus luteum, progesterone supplementation should continue until the time the production of progesterone has shifted to the placenta, approximately 10 weeks. Progesterone supplementation is also used for the prevention of preterm birth in patients with a history of a prior spontaneous preterm birth. When progesterone is used for prevention of preterm birth it is given weekly from weeks 16–36.
According to the latest USPSTF guidelines, what is the MOST appropriate age to begin routine screening for colon cancer?
the latest USPSTF recommendations 2021 advise that routine screening for all adults begin at age 45 years for colorectal cancer regardless of risk factors, as nearly 94% of all new colorectal cancer cases are occurring in adults 45 years and older. While rates of colorectal cancer are higher in Black adults, the age to begin routine screening is not younger than the general population.
What is the failure rate for the copper IUD within its first year of typical use?
The 1-year failure rate with typical use of the copper IUD is 0.8%. With perfect use, the failure rate in the first year is 0.6%.
The copper IUD has the highest failure rate among long-acting reversible contraceptives. Although its 1-year failure rate is higher than that of female sterilization, the 10-year failure rate of the copper IUD is similar to that of female sterilization (1.9 per 100 women over 10 years).
The 1-year failure rate for the copper intrauterine device is 0.8% with typical use. This is the highest failure rate among long-acting reversible contraceptives.
nexplanon has the lowest failure rate short/long term 0.05%
pneumococcal vaccine timeline
Patients with alcoholism, chronic lung disease, chronic heart disease, and diabetes mellitus should receive the pneumococcal vaccination between the ages of 19 and 64 years. All other patients, including those with hypertension, should start receiving vaccinations at age 65 years.
Which of the following cancers is associated with BRCA1 mutation carriers?
A BRCA1 pathogenic variant increases the lifetime risk of breast, ovarian, fallopian tube, peritoneal, pancreatic, and prostate cancers above the baseline population risk – simliar impact to BRCA2 except it also includes melanoma
Which of the following is a recommended initial step in the treatment of fibromyalgia?
For mild disease, nonpharmacologic measures such as education about fibromyalgia, review of sleep hygiene, exercise, and optimization of other medical conditions are the first steps. First-line pharmacotherapy includes amitriptyline, cyclobenzaprine, and certain SSRIs. Pregabalin is recommended for patients with more severe disease that is refractory to first-line nonpharmacologic and pharmacologic therapies.
Uterine adenomyosis is a histologic diagnosis made based on pathology evaluation of the uterus after hysterectomy.
The most common symptoms found with adenomyosis are pain and heavy bleeding. Heavy menstrual bleeding and dysmenorrhea (painful menstrual cycles) are the typical symptoms of adenomyosis, occurring in approximately 60% and 25% of women, respectively. Heavy menstrual bleeding is possibly related to the increased endometrial surface of the enlarged uterus, whereas pain may be due to bleeding and swelling of endometrial islands confined by myometrium.
Irregular cycles are not a hallmark symptom of adenomyosis and are often related to hormonal or ovulatory dysfunction, for example, polycystic ovarian syndrome (PCOS).
Adenomyosis is diagnosed on histological review of uterine tissue after hysterectomy. The most common symptoms are heavy and painful menstrual cycles; however, one-third of women with adenomyosis are asymptomatic.
Which of the following is MOST effective for relief of refractory mastalgia for a woman already taking an oral contraceptive pill?
Cyclic mastalgia is a component of fibrocystic changes of the breast and can usually be controlled with oral contraceptive pills (OCPs) or supplemental progestins given during the secretory phase of the cycle.
Severe symptoms, as evidenced in the patient above, are best treated with danazol (androgen), which inhibits the release of gonadotropins, leading to a hypoestrogenic, hyperandrogenic state. Danazol relieves symptoms in approximately 90% of patients, but it should not be continued for more than 6 months because of unwanted side effects (i.e., hirsutism).
After failing a course of danazol, patients may attempt a course of bromocriptine or tamoxifen. Supplementation with alpha-linoleic acids and evening primrose oil have effects in some women with resistant disease (58% response).
When all else fails, GnRH agonists (e.g., leuprolide acetate) may be used for short periods of time.
Therapeutic Agents for Mastalgia
NSAIDs
Combined Hormonal Contraceptive Pills
Danazol
Bromocriptine
Tamoxifen
Premenstrual dysphoric disorder (PMDD)
severe form of premenstrual syndrome. Per DSM-5, five or more symptoms (physical plus behavioral) need to be present for the last week before menses and resolve in the first few days of start of menses, for most of the preceding year. Excluding hypo- or hyperthyroidism, drug use, mood disorders, or medications is important before diagnosis of PMDD.
Premenstrual dysphoric disorder (PMDD) is a psychiatric disorder representing a severe form of premenstrual syndrome. It consists of the cyclic recurrence of severe changes in affect such as mood lability, irritability, dysphoria, and anxiety, that occur in the luteal phase of a woman’s menstrual cycle
5 or more symptoms must be
present during the week prior to
menses, resolving within a few
days after menses starts.
The presence of hypo- or hyperthyroidism, underlying chronic psychiatric disorders, drug use, medications, or hormone treatment such as combined OCPs must be ruled out prior to documenting such a diagnosis. Cyclic symptoms must be present for most of the preceding year. These should be documented using a menstrual diary.
Workup includes thyroid function tests, serum electrolytes, liver function tests, urine toxicology screen. Specific serotonin reuptake inhibitors such as sertraline can help relieve symptoms of PMDD.
After obtaining a culture, what is the BEST management for unilateral mastitis in the postpartum period?
Mastitis generally occurs after the third to fourth week postpartum. Infection is usually unilateral, and the patient may present with fever, chills, or tachycardia. Staphylococcus aureus is the most common agent and complicates 40% of cases.
A culture should be drawn from the affected breast’s milk. Dicloxacillin may be started empirically. Breastfeeding should be continued.
If nursing is difficult because of engorgement, pumping should be continued, as abscess formation has been reported to occur in women who stopped breastfeeding.
You obtain a full list of medications from the patient because you know all of the following agents can cause galactorrhea
Answer A: Codeine has been known to cause galactorrhea because it inhibits dopamine release.
Answer C: Methyldopa has been known to cause galactorrhea because it depletes dopamine.
Answer D: Metoclopramide has been known to cause galactorrhea because it blocks dopamine receptors.
Answer E: Risperidone has been known to cause galactorrhea because it blocks dopamine receptors.
Which of the following is the MOST accurate statement regarding the use of intrauterine contraceptive devices (IUDs) and pelvic inflammatory disease (PID)?
The relative risk of developing pelvic inflammatory disease (PID) is increased with intrauterine device (IUD) placement, but the absolute risk of developing PID is <0.5%.
Women with an undiagnosed sexually transmitted infection (STI) at the time of IUD insertion are more likely than women without an STI at the time of insertion to develop PID. However, even in women with an STI, the risk of developing PID is low. The greatest risk of IUD-associated PID occurs within the first few weeks to months after insertion, which suggests that bacterial contamination of the endometrial cavity at the time of IUD insertion, rather than the IUD itself, is the cause of PID.
Which of the following falls within the class of medication that is considered first-line pharmacologic treatment of depression?
Depression is a common disease in the United States. It is generally diagnosed through a series of questions. Patients often complain of sadness, hopelessness, guilt, moodiness, angry outbursts, and loss of interest. They may complain of difficulty concentrating, making decisions, or remembering things that they should know. If severe enough, it can lead to thoughts of harm to oneself or others, delusions, or hallucinations. Behavior is affected by withdrawing from people, substance abuse, missing time at work or school, or attempts to harm oneself. Finally, physical issues such as tiredness, unexplained aches/pains, changes in appetite and weight, or changes in sleep are possible.
When depression is diagnosed, the type of treatment administered is based on the patient’s preferences. Psychotherapy helps many patients improve their symptoms. Furthermore, lifestyle changes such as physical activity and stress-reduction exercises may help.
Selective serotonin reuptake inhibitors (SSRIs) are generally considered the first-line medication for depression. SSRIs include those listed below, of which escitalopram is among the answer choices.
Common SSRI Medications
citalopram
escitalopram
fluoxetine
paroxetine
sertraline
fluvoxamine
Which of the following findings on colposcopy with use of acetic acid is associated with a high-grade dysplastic lesion of the cervix?
Dull/dense white epithelium is a high-grade finding on colposcopy when acetic acid is applied, while normal colposcopic epithelium appears shiny. The margins of these dull white areas may be rolled or peeled or may even have internal borders (a lesion within a lesion). The vessels noted are often coarse and/or dilated in high-grade or cancerous lesions.
Colposcopy is a skill that takes time and experience to master. Although pathologic results are necessary for management, the clinician should be able to make an educated diagnosis on the grade of the lesion that is to be biopsied. Adequate visualization of the squamous columnar junction is necessary and sometimes requires the aid of an endocervical speculum. Knowing the grade of a lesion at the time of colposcopy may help determine whether additional procedures are needed at this time, and can help with counseling the patient on future management plans.
Colposcopy is a skill that takes time and experience to master. Although pathologic results are necessary for management, the clinician should be able to make an educated diagnosis on the grade of the lesion that is to be biopsied. Adequate visualization of the squamous columnar junction is necessary and sometimes requires the aid of an endocervical speculum. Knowing the grade of a lesion at time of colposcopy may help determine whether additional procedures are needed at this time, as well as help with counseling the patient on future management plans.
Which of the following is the gold standard for the diagnosis of renal artery stenosis?
Renal artery stenosis should be high on the differential diagnosis for any young, healthy patient with sudden-onset severe hypertension. Renal arteriography is the gold standard for diagnosis of renal artery stenosis.
Duplex Doppler ultrasonography, computed tomographic angiography, and magnetic resonance angiography are alternative, less-invasive tests that are appropriate for initial testing. If results of noninvasive testing are inconclusive and/or suspicion for the diagnosis of renal artery stenosis is high, traditional renal arteriography should be performed.
Among pregnant women, which of the following is the MOST commonly abused substance?
During pregnancy, 16.3% of women admit to continued tobacco use; this percentage is thought to be an underestimate secondary to underreporting by patients.
Smoking is the single most important modifiable risk factor associated with poor pregnancy outcomes.
Tobacco is the most commonly abused substance among pregnant women; however, it is not uncommon to abuse alcohol, cocaine, marijuana, and opioids as well. During pregnancy, 16.3% of women report continued tobacco use, 10.8% report continued alcohol use, and 4.4% report illicit drug abuse.
Which of the following is the MOST appropriate age at which to begin osteoporosis screening in women?
The most appropriate age at which to begin osteoporosis screening is 65 years or when the patient is deemed to have the fracture risk of a 65-year-old patient. This can be calculated using the FRAX calculation tool. The risk at this age is approximately 1%.
It should be repeated every 15 years if there are no other risk factors or treatments in place.
Components of FRAX Score
Age
Sex
Weight (kg)
Height (cm)
Previous fracture (yes/no)
Parent with fractured hip (yes/no)
Current smoking (yes/no)
Glucocorticoid use (yes/no)
Rheumatoid arthritis (yes/no)
Secondary osteoporosis (yes/no)
3+ alcoholic drinks/day (yes/no)
Femoral neck bone mineral density (BMD)
A urethral diverticulum classically presents with which 3 symptoms?
The classic presentation of a urethral diverticulum consists of the triad of postvoid dribbling, dysuria, and dyspareunia.
A urethral diverticulum is a localized outpouching of the urethral mucosa into the surrounding non-urothelial tissues. Urethral diverticula are most often acquired and may result from vaginal, bladder, or urethral surgery or following a vaginal delivery. The diagnosis of urethral diverticulum is made with MRI or ultrasound +/- urethroscopy. Patients with bothersome symptoms may choose to undergo surgical management with a transvaginal diverticulectomy.
Which of the following MOST accurately describes the recommended process for the sterilization of a transvaginal ultrasound probe?
Condoms have been shown to be less prone to leakage than commercial use probe barriers.
The rate of leakage with condoms is approximately 0.9%–2% versus commercial probe covers, which were noted in recent observational studies to leak 8%–81% of the time.
Transvaginal probes should be covered with a single-use barrier, cleaned after use to remove any debris or visible soiling, and then properly sterilized with a high-level disinfectant after cleaning.
Which of the following answers represents the BEST tests to order to rule out antiphospholipid antibody syndrome (APS)?
APS is a syndrome that has the potential to adversely affect women of reproductive age. Women with a history of recurrent miscarriages may be at risk for APS. If APS is present, they may benefit from treatment to prolong their pregnancy to viability.
The recommended tests are all 3 of the tests mentioned above: IgG and IgM anti-β2-glycoprotein antibodies AND IgG and IgM anticardiolipin antibodies AND lupus anticoagulant. Additionally, there must be a positive result on at least 2 occasions, ≥12 weeks apart.
Which of the following is the MOST effective form of birth control?
The etonogestrel implant, or Nexplanon, is the most effective birth control on the market. The estimated unintended pregnancy rate is 0.05 per 100 women in the first year of use.
A number of variables should be considered when discussing birth control options. One of these variables is the unintended pregnancy rate, as estimated by typical use. Birth control options that require patients to either take a pill, use a barrier, or receive a dose every 3 months have a much higher unintended pregnancy rate than the reversible long-acting methods.
A number of variables should be considered when discussing birth control options. One of these variables is the unintended pregnancy rate, as estimated by typical use. The most effective form of birth control available is the Nexplanon implant. The second-most effective female form of contraception is the progestin-containing IUD, followed by the copper IUD. Options that require patients to either take a pill, use a barrier, or receive a dose every 3 months have a much higher unintended pregnancy rate than the reversible long-acting methods.
lichen sclerosus
Lichen sclerosus, on the other hand, is of the chronic variety and typically presents as porcelain-white papules and plaques, often with areas of ecchymosis or purpura. The skin may appear thin, whitened and crinkling, leading to the description of “cigarette paper.” Because other vulvar diseases can mimic lichen sclerosus, a biopsy is necessary to confirm the diagnosis, except in a prepubertal child.
What is the MOST common cause of vaginitis in children?
Vaginitis is one of the most common reasons for patient visits to gynecologists. It is defined as abnormal vaginal discharge accompanied by burning, irritation, or itching.
Etiologies for vaginitis can differ depending on the age of the patient. In women of reproductive age, 90% of causes are infectious, of which Candida albicans (which causes candidiasis), Gardnerella vaginalis (which causes bacterial vaginosis), and Trichomonas vaginalis are the most common. Other less frequent infectious causes include herpes, gonorrhea, and chlamydia.
Before puberty, however, the common causes of vaginitis are different. The most common cause in this demographic is the introduction of bacteria from the anal region to the vaginal region with improper hygiene.
Which of the following is TRUE regarding the pudendal nerve?
The pudendal nerve provides both sensory and motor innervation to the perineum. It arises from the sacral plexus at S2–S4 and leaves the pelvis through the greater sciatic foramen. It then hooks around the ischial spine and sacrospinous ligament to the enter the pudendal (Alcock’s) canal through the lesser sciatic foramen.
The branches of the pudendal nerve are:
The clitoral branch: supplies the clitoris
The perineal branch: supplies the bulbospongiosus, ischiocavernosus, and transverse perineal muscles (motor) as well as the skin of the inner portions of the labia majora, minora, and vestibule (sensory)
The inferior hemorrhoidal branch: supplies the external anal sphincter (motor) and perianal skin (sensory)
How often should a 13-year-old girl receive an injection of the human papillomavirus (HPV) quadrivalent vaccination after her initial injection?
The Human Papillomavirus vaccination significantly reduces the incidence of anogenital cancer and genital warts. The human HPV vaccine is indicated for males and females age 9–26 for the prevention of diseases caused by HPV.
For girls and boys who receive their first dose of HPV vaccine before 15 years of age, only two doses are needed. The timing of the two doses is 0 (baseline) and 6–12 months. If the interval between the two doses is less than 5 months, a third dose is recommended. If females or males receive their first dose at 15 years of age or older, three doses are needed and given at 0 (baseline), 1–2 months after the first dose, and 6 months after the first dose.
New FDA and CDC updates on HPV vaccine use in women and men aged 27 through 45 years are under review by ACOG to determine if updates are needed based on this recommendation.
What is the BEST treatment option for a nonfunctional pituitary macroadenoma causing significant visual symptoms?
Macroadenomas are defined as pituitary tumors >10 mm. They are often discovered during the workup for neurological and/or visual symptoms. Nonfunctional pituitary macroadenomas that are large enough to cause neurological and/or visual problems require surgery as the initial approach. Medical therapy has not proven effective enough to reduce the size of the tumor to prevent long-term damage. The surgical approach is nearly always transsphenoidal. The transcranial approach may be used as little as 1% of the time.
For functional pituitary (prolactinoma), medical therapy is the preferred treatment option.
Which of the following symptoms/findings is consistent with irritable bowel syndrome?
The Rome criteria for diagnosis of irritable bowel syndrome include abdominal pain or discomfort coupled with altered defecation or bowel habits.
Atypical symptoms or symptoms that are not compatible with irritable bowel include progressive or nocturnal abdominal pain, rectal bleeding, weight loss, and laboratory test abnormalities. If any of these symptoms are present, further evaluation should be completed because these are not symptoms attributable to irritable bowel syndrome.
The risk of pelvic inflammatory disease with an intrauterine device (IUD) in place is approximately what percentage?
<1%
Having an IUD in place is not a risk factor for pelvic inflammatory disease. In fact, many studies show that the risk of pelvic inflammatory disease in women who have an IUD in place is equal to, or maybe even less than, in women who do not have an IUD in place. The Dalkon Shield, which was an IUD used frequently in the 1970s, did have an association with pelvic inflammatory disease. However, the new IUDs have a different construction than the Dalkon Shield, and do not carry that same risk.
A 28-year-old woman presents to the office for an annual exam. She has no complaints. She is a 1 pack a day smoker for the last 10 years. She has tried quitting several times but has been unsuccessful. Which of the following is MOST likely to help her succeed at quitting?
Varenicline (most effective), bupropion, and nicotine-replacement products (nicotine patches, gum, and lozenges, as well as nicotine nasal and oral inhalers) are first-line pharmacotherapies for smoking cessation.
Varenicline is a partial agonist for the nicotinic acetylcholine receptor (initial dose 0.5 mg daily, increasing to 1 mg twice daily at day 8; treatment duration up to 6 months). Clonidine or nortriptyline may be useful for patients who have failed with first-line pharmacologic treatment or who are unable to use other therapies.
An exam for an alleged sexual assault is likely to include which of the following?
Sexual assault is a crime of violence, conquest, control, and aggression. Sexual assault does not include passion, and has a continuum of sexual activity from sexual coercion to contact abuse, and forcible rape. Sexual assault is common among women. About 1 of 8 women will experience an assault at some time in their life
Sexual assault most often affects women, but it can also happen to men. Most women who are raped know their assailant. The assailant is usually a relative or acquaintance of the victim or her friends and family. Only 20%–25% of women do not know their assailant.
Many hospitals utilize trained sexual assault nurses for the examination of a rape victim. The examination includes a detailed history and physical exam, gynecologic history, description of assailant, description of type of sexual contact, observation of the survivor’s emotional state, collection of vaginal secretions, Gram stain, examination with Woods light, collection of pubic hair, collection of victim’s saliva, fingernail scrapings of the victim, wet prep, and a urine specimen.
It is important to note if the patient took a bath/shower, used a douche, used a tampon, urinated, defecated, brushed her teeth, used an enema, used mouth wash, or changed clothes because any of those actions can impair collection of the forensic evidence.
A 14-year-old patient who has been sexually molested presents with a painless chancre on her left labia that she has noticed over the last 3 weeks. Which of the following is the MOST appropriate treatment of primary syphilis?
The treatment for primary syphilis is benzathine penicillin G 2.4 million units once.
When a patient has a painless, nonindurated, singular chancre with raised rounded borders, the most common diagnosis is syphilis. The mean incubation period for these lesions is 3 weeks. Left untreated it will resolve the lesion in up to 6 weeks.
The most important aspect once syphilis is diagnosed is determining whether the syphilis is primary, secondary, early latent (< 1 year), late latent, tertiary, or neurosyphilis. Secondary syphilis occurs 6 weeks to 6 months following the initial chancre. It generally leads to a maculopapular rash, which is most commonly associated with lesions on the palms, soles, or mucous membranes. It will actively shed spirochetes and it may produce broad, grayish condylomata lata. Early latent syphilis is the period up to 1 year while late latent syphilis is any period beyond 1 year. This determines whether the patient needs higher dose, more prolonged treatment. Tertiary syphilis can take as long as 20 years to develop and its sequelae can include cardiovascular, CNS, and musculoskeletal involvement.
Diagnosis is by dark field examination or direct fluorescent antibody testing. Venereal disease research laboratory/rapid plasma reagin (VDRL/RPR) testing can give a presumptive diagnosis. Fluorescent Treponema antibody absorption may be used as a confirmatory test. Following treatment, VDRL or RPR can be monitored for response to treatment. A fourfold decrease is required by 6 months for primary or secondary syphilis, while it is expected in 12–24 months with latent syphilis.
If primary, secondary, or early latent syphilis is diagnosed, treatment is benzathine penicillin G once. If it is late latent or tertiary syphilis, the treatment is at least benzathine penicillin G for 3 weeks.
Which of the following strategies is MOST supported for the prevention of ovarian cancer in patients with a BRCA gene mutation?
Oral contraceptive use is associated with a 50% decreased risk of developing ovarian cancer.
Oral contraceptive use decreases the risk of ovarian cancer in BRCA1 and 2 carriers who are NOT candidates for or who do not desire prophylactic oophorectomy.
Which of the following findings on cystoscopy is MOST indicative of interstitial cystitis?
Hunner’s ulcers are pathognomonic for interstitial cystitis (IC). These are described as red lesions on the bladder mucosa with attached fibrin deposits that typically bleed after hydrodistention.
Cystoscopy is not necessarily required to diagnose IC, but it is often performed to exclude additional diagnoses.
What is the MOST common autosomal trisomy detected in first trimester abortions?
Trisomy is the presence of three copies of a given chromosome and is the most frequently identified chromosomal anomaly in early pregnancy losses, with an incidence of 22–32%.
Of the trisomies, trisomy 16 is the most common detected in first trimester abortions. This is a lethal anomaly and may explain why this anomaly is detected more commonly than the less lethal trisomies listed above.
Below is a table on the most common trisomies overall.
Which of the following is CORRECT regarding initiation of treatment and the FRAX score?
When using the FRAX system, treatment should be considered when the hip fracture risk exceeds 3% or the major osteoporotic fracture risk exceeds 20% over the next 10 years.
The FRAX system evaluates a patient’s risk of fracture. The FRAX calculates risk based on clinical risk factors and bone mineral density.
Which of the following scenarios describes a patient who should undergo an endometrial biopsy?
A 40-year-old obese, nulliparous woman with heavy intermenstrual bleeding should undergo an endometrial biopsy to assess for endometrial hyperplasia or neoplasia.
Endometrial biopsy (EMB) is a quick and relatively inexpensive way to sample the endometrial lining and aid in the diagnosis of a number of different disease processes. The mainstay of diagnosing endometrial cancer remains endometrial sampling (either via dilatation and curettage or EMB). A Pipelle® is a common method for sampling the endometrium and it samples 5–15% of the endometrial surface area.
This vignette describes a woman with abnormal uterine bleeding; in any woman with these symptoms who is over the age of 35 years, endometrial sampling to rule out endometrial cancer is warranted, particularly before prescribing medications to treat assumed ovulatory dysfunction. In addition, this patient is obese and nulliparous, which increases her chances of having endometrial cancer.
Occasionally, patients undergo imaging unrelated to a gynecologic issue and a thickened endometrial stripe is incidentally found. Postmenopausal patients without vaginal bleeding who have an endometrial thickness > 11 mm should undergo endometrial sampling.
All patients with atypical endometrial cells on cervical cytology should undergo endometrial and endocervical sampling ± colposcopy. However, this patient has atypical glandular cells (not atypical endometrial cells). Patients with atypical glandular cells (other than atypical endometrial cells) who are ≥ 35 years of age should undergo colposcopy, endocervical sampling, and endometrial sampling; whereas patients < 35 years of age who are not at elevated risk of endometrial neoplasia, such as this patient, only require colposcopy.
Tamoxifen is a nonsteroidal antiestrogen agent that is widely used as adjunctive therapy for women with breast cancer. Premenopausal women taking tamoxifen do not have an increased risk of uterine cancer and do not require any additional monitoring beyond routine gynecologic care. However, postmenopausal women taking tamoxifen should be closely monitored for symptoms of endometrial hyperplasia or cancer.
This 70yo vaginally bleeding patient’s transvaginal ultrasound is reassuring with an endometrial lining measuring 2 mm and she does not need an endometrial biopsy at this time. If her endometrial stripe were > 4 mm, an endometrial biopsy would be indicated. The sensitivity and specificity of transvaginal ultrasound for detection of endometrial cancer at a threshold of 4 mm are 96% and 53%, respectively.
A 60-year-old woman presents with a complaint of bilateral, milk-like nipple discharge. Upon exam, fluid from multiple ducts is expressed. The patient’s medical history is significant for depression and poorly controlled diabetes with retinopathy and gastroparesis. Which of the following medications could be responsible for this patient’s nipple discharge?
This patient has galactorrhea, which is defined as non-pathologic discharge unrelated to pregnancy or breast-feeding. Galactorrhea typically presents as bilateral, multi-ductal nipple discharge of a white substance. One common cause of galactorrhea is medication-induced hyperprolactinemia.
This patient presents with poorly controlled diabetes and subsequent gastroparesis. Nausea and vomiting associated with gastroparesis is commonly treated with metoclopramide. This medication is a prokinetic, antiemetic that works by blocking dopamine receptors in the chemoreceptor trigger zone of the CNS. It also modulates acetylcholine’s activity in the GI tract to enhance motility. In some cases, dopamine receptor blockade leads to hyperprolactinemia, which can cause galactorrhea.
This patient also suffers from depression. Though many antipsychotic medications cause hyperprolactinemia by dopamine receptor blockade, only a few antidepressant medications such as clomipramine, a tricyclic antidepressant, may cause an increase in prolactin; this is thought to be related to gamma-aminobutyric acid (GABA) stimulation. Antidepressants such as buproprion, mirtazapine, trazodone, and venlafaxine do not cause hyperprolactinemia or galactorrhea.
A 60-year-old woman presents with a complaint of bilateral, milk-like nipple discharge. Upon exam, fluid from multiple ducts is expressed. The patient’s medical history is significant for depression and poorly controlled diabetes with retinopathy and gastroparesis. Which of the following medications could be responsible for this patient’s nipple discharge?
This patient has galactorrhea, which is defined as non-pathologic discharge unrelated to pregnancy or breast-feeding. Galactorrhea typically presents as bilateral, multi-ductal nipple discharge of a white substance. One common cause of galactorrhea is medication-induced hyperprolactinemia.
This patient presents with poorly controlled diabetes and subsequent gastroparesis. Nausea and vomiting associated with gastroparesis is commonly treated with metoclopramide. This medication is a prokinetic, antiemetic that works by blocking dopamine receptors in the chemoreceptor trigger zone of the CNS. It also modulates acetylcholine’s activity in the GI tract to enhance motility. In some cases, dopamine receptor blockade leads to hyperprolactinemia, which can cause galactorrhea.
This patient also suffers from depression. Though many antipsychotic medications cause hyperprolactinemia by dopamine receptor blockade, only a few antidepressant medications such as clomipramine, a tricyclic antidepressant, may cause an increase in prolactin; this is thought to be related to gamma-aminobutyric acid (GABA) stimulation. Antidepressants such as buproprion, mirtazapine, trazodone, and venlafaxine do not cause hyperprolactinemia or galactorrhea.
A 56-year-old woman is undergoing preoperative evaluation prior to placement of a suburethral sling secondary to stress urinary incontinence. She states that her cardiologist recently told her she has heart failure. On further prompting, she states that she is able to walk outside daily but becomes fatigued and short of breath faster than she used to. Otherwise, she is able to get around her house without issues. To which NYHA functional class does she belong?
This patient fits into NYHA function class II for heart failure. It is not clear exactly how she attained the diagnosis of heart failure. However, she does provide a good description of her functionality. She is able to do a more strenuous task such as walking outside, but it makes her short of breath and tire faster. Although this could simply be deconditioning or secondary to multiple other factors, for that background of this question, it is assumed to be secondary to her heart failure. Because she is not completely functional and without symptoms, she is not class I. Class III describes an inability to do even light activities around the house, which she is perfectly capable of // This class describes difficulty doing daily low-energy tasks. Class IV requires the inability to function normally at rest. Generally, with this class, any activity causes inability to function.
Which of the following is the MOST likely diagnosis in a woman with multiple genital sores associated with scarring and foul-smelling discharge?
Hidradenitis suppurativa presents with recurrent papular lesions that may lead to abscesses, fistulas, and scarring and can be associated with foul-smelling discharge, primarily in areas of the body with apocrine glands.
Crohn’s disease of the vulva and hidradenitis suppurativa are difficult to distinguish. Perianal and vulvar manifestations of Crohn’s disease include abscesses, rectoperineal and rectovaginal fistulas, sinus tracts, fenestrations, and scarring, as well as classic “knife-cut” ulcers in the inguinal, genitocrural, or interlabial folds. The lack of GI symptoms in this patient supports the diagnosis of hidradenitis suppurativa over Crohn’s disease.
Folliculitis is simple inflammation of the hair follicle and presents with multiple superficial, inflammatory papules each surrounding a hair follicle, with or without an overlying pustule. Infections can progress if untreated or only partially treated to a nodular abscess which is termed a furuncle. Folliculitis is transient, responds rapidly to appropriate antibiotic therapy, and does not cause the comedones, persistent sinus tracts, or hypertrophic scarring which are common in hydradenitis suppurativa.
Granuloma inguinale, also known as Donovanosis, is a sexually transmitted infection caused by Klebsiella granulomatis that usually occurs on the vulva, inguinal folds, or perianal skin. Lesions usually present as enlarging red ulcers with irregular borders and granulation tissue that bleeds easily and with a foul odor. The causative organism cannot be cultured, and diagnosis requires visualization of Donovan bodies on pathology from tissue biopsy.
Lymphogranuloma venereum (LGV) is caused by serotypes L1, L2 and L3 of Chlamydia trachomatis, and presents clinically with genital papules and ulcers and painful inguinal adenopathy (buboes).
Which of the following is true regarding long-acting reversible contraception use in the adolescent population?
Long-acting reversible contraceptives (LARC) have higher efficacy, higher continuation rates, and higher satisfaction rates compared with short-acting contraceptives among adolescents who choose to use them.
What time of a woman’s menstrual cycle is the ideal timing for a diagnostic hysteroscopy?
In premenopausal women with regular cycles, the optimal timing for a diagnostic hysteroscopy is during the follicular phase of the menstrual cycle after menstruation, with reasonable exclusion of pregnancy. At this time, the lining of the endometrium is thin and should not obscure any small fibroids or irregular contours.
During the secretory phase, the endometrial lining is thick and it can mimic small polyps and give a false diagnosis. Active bleeding may obscure visualization and therefore hysteroscopy during menstruation is not ideal. Pretreatment with progestins may improve visualization.
A 23-year-old woman is admitted to the hospital for complications from primary pulmonary artery hypertension. The physician reads in her history that she has a family history of ovarian cancer. The physician sees that she has a history of taking appetite suppressants. She has had pelvic inflammatory disease in the past and also has moderate persistent asthma. What is the STRONGEST risk factor for primary pulmonary hypertension in this patient?
Appetite suppressants have a definite association with primary pulmonary artery hypertension. Aminorex, fenfluramine, and dexfenfluramine all increase the risk. The mechanism is unknown, but altered serotonin chemistry likely plays a role.
Pulmonary hypertension is separated into 5 groups. The first group is due to pulmonary artery hypertension and is further classified by cause. Causes include:
drugs and toxins
HIV
heart disease
connective tissue disorders
A 23-year-old woman is admitted to the hospital for complications from primary pulmonary artery hypertension. The physician reads in her history that she has a family history of ovarian cancer. The physician sees that she has a history of taking appetite suppressants. She has had pelvic inflammatory disease in the past and also has moderate persistent asthma. What is the STRONGEST risk factor for primary pulmonary hypertension in this patient?
Appetite suppressants have a definite association with primary pulmonary artery hypertension. Aminorex, fenfluramine, and dexfenfluramine all increase the risk. The mechanism is unknown, but altered serotonin chemistry likely plays a role.
Pulmonary hypertension is separated into 5 groups. The first group is due to pulmonary artery hypertension and is further classified by cause. Causes include:
drugs and toxins
HIV
heart disease
connective tissue disorders
A 19-year-old G1P0 woman presents for a new obstetrical appointment. She has a history of intimate partner violence. What is the prevalence of intimate partner violence in pregnancy?
A meta-analysis of 13 studies revealed that the prevalence of intimate partner violence in pregnancy is anywhere from 5% to 20%.
The table below lists some of the risk factors for intimate partner violence during pregnancy.
Risk factors for intimate partner violence in pregnancy
history of intimate partner violence
age less than 24 years
at-risk alcohol or drug use
female sex
In a woman with a known mutation for hereditary nonpolyposis colorectal cancer, or Lynch syndrome, which of the following is the MOST appropriate screening modality?
Lynch syndrome refers to individuals and families with a pathogenic germline mutation in one of the DNA mismatch repair genes (MLH1, MSH2, MSH6, and PMS2) or the EPCAM gene.
Screening and Surveillance Recommendations for Women With Lynch Syndrome
Colonoscopy every 1–2 years, beginning at age 20–25 years, OR 2–5 years before the earliest cancer diagnosis in the family, whichever is earlier
Endometrial biopsy every 1–2 years, beginning at age 30–35 years
Keeping a menstrual calendar and evaluating abnormal uterine bleeding
For endometrial cancer screening, we perform yearly endometrial sampling starting at age 30–35 or 5–10 years prior to the earliest age of Lynch-associated cancer in the family. For ovarian cancer screening, we perform an annual pelvic examination and transvaginal ultrasound (TVUS) examination, with or without cancer antigen 125 (CA 125), every 6 to 12 months starting at age 30–35 or 5–10 years prior to the earliest age of Lynch-associated cancer of any kind in the family.
However, not screening is also reasonable given that no screening strategy (CA 125, TVUS, or multimodal testing) has been shown to reduce mortality, and all surveillance strategies are associated with a high rate of false-positive tests and a risk of harm from invasive testing. For patients with Lynch syndrome who have completed childbearing, we continue to suggest risk-reducing total hysterectomy with bilateral salpingo-oophorectomy (TH-BSO) rather than surveillance and/or chemoprevention.
Individuals with Colorectal or Endometrial Cancer for Whom Genetic Risk Assessment is Recommended
Patients with endometrial or colorectal cancer diagnosed before age 50 years
Patients with endometrial or ovarian cancer with a synchronous or metachronous colon or other Lynch/HNPCC-associated tumor* at any age
Patients with colorectal cancer with tumor-infiltrating lymphocytes, peritumoral lymphocytes, Crohn-like lymphocytic reaction, mucinous/signet-ring differentiation, or medullary growth pattern diagnosed before age 60 years
Patients with endometrial or colorectal cancer and a first-degree relative† with a Lynch/HNPCC-associated tumor* diagnosed before age 50 years
Patients with colorectal or endometrial cancer diagnosed at any age with 2 or more first-degree or second-degree relatives† with Lynch/HNPCC-associated tumors*, regardless of age
* = Lynch/HNPCC-related tumors include colorectal, endometrial, stomach, ovarian, pancreas, ureter and renal pelvis, biliary tract, and brain (usually glioblastoma) tumors, sebaceous gland adenomas and keratoacanthomas, and carcinoma of the small bowel
† = First-degree relatives are parents, siblings, and children. Second-degree relatives are aunts, uncles, nieces, nephews, grandparents, and grandchildren
An 18-year-old nulligravid female presents for an annual examination and would like to start birth control. The physician discusses long-acting options, and the patient chooses the hormonal IUD. Which of the following is true regarding the Mirena IUD?
Mirena placement is contraindicated in postpartum endometritis, infected abortion, or pelvic inflammatory disease (PID) within the past 3 months. If PID is to occur in the presence of an IUD, do NOT remove the IUD unless 48–72 hours have passed with no clinical improvement despite optimal treatment (possible tubo-ovarian abscess) or if PID is a result of Actinomyces.
The levonorgestrel IUD (Mirena) is one of the LARCs. The failure rate is 0.2% with typical use. It works by thickening cervical mucous, causing endometrial decidualization, and glandular atrophy. The levonorgestrel IUD can cause problems with menses. Amenorrhea occurs in up to 20% of users in the first year and rises to 30–50% after 2 years of use. Patients should be counseled that menstrual bleeding patterns may be altered during the first 3–6 months after placement and may result in irregular bleeding, heavy bleeding, or amenorrhea.
Which of the following statements is true regarding Tdap vaccination and pregnancy?
If a pregnant woman has no known history of immunization to tetanus and diphtheria, she should receive all 3 Td immunizations at the appropriate intervals. One of the 3 vaccines should be administered as Tdap between 27 and 36 weeks.
if a mother does not receive a Tdap vaccine during pregnancy, she can receive it immediately postpartum, even if she is breastfeeding. However, infant immunity through breastfeeding may not occur until approximately 2 weeks after immunization.
If a pregnant women lives in an area experiencing a pertussis outbreak, she may receive her Tdap immunization at any time during her pregnancy.
The recommended time for a pregnant women to receive a Tdap booster is between 27 and 36 weeks.
Which of the following is the MOST likely diagnosis in a woman with multiple hypopigmented, nonpruritic patches on her vulva?
Vitiligo is an acquired pigmentary disorder commonly associated with disorders of autoimmune origin, such as hyperthyroidism or hypothyroidism, diabetes mellitus, and pernicious anemia. Additionally, the development of vitiligo may be precipitated by stress. It is characterized by white or hypopigmented lesions that are usually well demarcated and range in size.
Treatment is usually with phototherapy, but some forms of the disease are permanent.
What is the next BEST step for a 21-year-old woman with an ASCUS (atypical squamous cells of undetermined significance) Pap result?
In women between the ages of 21 and 24, either reflex human papillomavirus (HPV) testing or repeat cytology in 12 months is acceptable following an atypical squamous cells of undetermined significance (ASCUS) Pap.
An endometrial biopsy is required for which of the following clinical scenarios?
An office endometrial biopsy is the first-line procedure for tissue sampling in the evaluation of patients with abnormal uterine bleeding (AUB). The primary role of endometrial sampling in patients with AUB is to determine whether carcinoma or premalignant lesions are present, although other pathology related to bleeding may be found.
Endometrial sampling is recommended in many circumstances including in women over the age of 35 who have atypical glandular cells on cervical cytology. It may also be considered for women with atypical glandular cells under 35 years of age if they have risk factors.
Endometrial tissue sampling should also be performed in patients with AUB who are older than 45 years as a first-line test. Endometrial sampling also should be performed in patients younger than 45 years with a history of unopposed estrogen exposure (such as seen in obesity or PCOS), failed medical management, or persistent AUB.
Answer A: Transvaginal ultrasonography has excellent negative predictive value for endometrial cancer in women with postmenopausal bleeding. When transvaginal ultrasonography is performed for patients with postmenopausal bleeding and an endometrial thickness of 4 mm or less is found, endometrial sampling is not required because of the very low risk of uterine malignancy in these patients . Per ACOG, an endometrial thickness greater than 4 mm in a patient with postmenopausal bleeding should trigger alternative evaluation (such as sonohysterography, office hysteroscopy, or endometrial biopsy).
Answer B: Patients with AUB and risk factors above the age of 35 should undergo EMB.
Answer C: Screening and surveillance recommendations for women with Lynch Syndrome include endometrial biopsy every 1-2 years, beginning at age 30-35 years.
Answer E: In asymptomatic women using tamoxifen, screening for endometrial cancer with routine transvaginal ultrasonography, endometrial biopsy, or both has not been shown to be effective.
Bottom Line: Endometrial tissue sampling should be performed in patients with women over the age of 35 who have atypical glandular cells on cervical cytology.
Which of the following is TRUE regarding obesity and incontinence?
Patients with class 3 obesity treated with a midurethral sling for stress urinary incontinence are 2x more likely to fail treatment when compared with normal-weight women.
More than 50% of American women are overweight (BMI 25 to 29.9 kg/m2) or obese (BMI ≥30 kg/m2) and the prevalence of obesity is increasing almost 6% per year. Urinary incontinence affects almost 50% of middle-aged and older women. A retrospective cohort study by Elshatanoufy et al. showed that women with class 3 obesity (BMI ≥40 kg/m2) were 2x more likely to fail treatment with a midurethral sling after controlling for other comorbidities. In this same study, class 3 obesity was the only group that revealed an increased failure rate compared with normal-weight individuals.
Which of the following is TRUE of interstitial cystitis (IC)?
Interstitial cystitis is a diagnosis of exclusion and is no longer based on cystoscopic findings
Interstitial cystitis (IC), or painful bladder syndrome (PBS), is characterized by chronically occurring suprapubic pain related to bladder filling, usually associated with dysuria, for which acute cystitis can never be diagnosed due to negative urine cultures. Other symptoms include daytime frequency and nocturia in the absence of proven infection or other obvious pathology. The syndrome is midline and, thus, dyspareunia is a common complaint.
The pathognomonic finding is Hunner’s ulcers seen on cystoscopy, although these only occur in approximately 5–10% of women with IC.
Treatment of IC involves dietary changes (i.e., avoiding acidic, alcoholic, carbonated beverages, spicy foods, and artificial sweeteners) and pentosan polysulfate sodium (Elmiron), which is the only FDA-approved oral drug for IC.
What percentage of women experience postpartum blues?
Postpartum blues, which is a transient condition characterized by mild depressive symptoms, develops in 40%–80% of women in the first few days following delivery. Symptoms typically peak over the next few days and resolve within 2 weeks.
Of the following cancers, which is she MOST at risk for if she also has Lynch syndrome?
Individuals with Lynch syndrome, or mutations of DNA mismatch repair genes associated with Lynch syndrome, are at increased risk of colorectal, endometrial, ovarian, hepatobiliary, stomach, and small bowel cancers, cancers of the renal pelvis, and gliomas in the brain. There may be an increased risk of ureteral cancers and cancers of the renal pelvis.
Patients with Lynch syndrome have the highest lifetime risk of colon and endometrial cancers. Lynch syndrome is least likely to be directly associated with breast cancer, although there is a slightly increased risk.
The diagnosis of gender dysphoria is generally made by a mental health professional; however, other healthcare professionals who have the appropriate experience and training can also diagnose gender dysphoria. Mental health providers typically use the Diagnostic and Statistical Manual of Mental Disorders, Fifth Edition (DSM-V) to make the diagnosis.
Core components of the DSM-V diagnosis of gender dysphoria include long-standing discomfort with the incongruence between gender identity and external sexual anatomy at birth, along with interference with social, school, or other areas of function.
What is the longest time a GnRH agonist should be used, according to the FDA, if add-back therapy is utilized?
GnRH agonists have proven efficacy in decreasing pelvic pain in women with surgery-diagnosed endometriosis. A maximum of 6 months of treatment with GnRH analogues is currently licensed for ovarian downregulation. Evidence has shown that the add-back therapy does not affect the pain-reducing qualities of the GnRH agonist. However, GnRH agonists in combination with this add-back therapy may make the use of 12-month therapy safer, as opposed to 6 months.
The use of GnRH agonists is associated with subsequent hypoestrogenic symptoms. The rate of loss of bone density is in the range of 0.5–1.0% per month, which over time increases the risk of osteoporosis. Restriction of use of GnRHa to 6 months severely limits clinical efficacy: no sooner has the patient derived maximum benefit from the treatment than she is advised to stop. However, several studies have shown the efficacy of hormone replacement “add-back” therapy prescribed in conjunction with GnRHa to reduce the impact of low circulating oestrogens on bone density.
Hypoestrogenic state symptoms also include hot flushes, insomnia, and decreased libido; quite concerning is the loss of bone mineral density. This latter point is the main driver limiting the time of treatment with GnRH agonists. This add-back therapy may make the use of 12-month therapy safer.
Lupron Depot (3.75 mg leuprolide acetate) is used for management of endometriosis, including pain relief and reduction of endometriotic lesions. It is used monthly with norethindrone acetate (aygestin) 5 mg daily for initial management of endometriosis and for management of recurrence of symptoms. Concomitantly with iron therapy is indicated for the preoperative hematologic improvement of patients with anemia caused by uterine leiomyomata (fibroids).
What is the BEST next step in management of a patient with DEXA scan T-score of −2.7 in the right femoral neck and −2.6 in the left femoral neck?
Treatment for osteoporosis typically starts with lifestyle modifications with first-line medications typically being the bisphosphonates. The main side effect of bisphosphonates is GI irritation and it is recommended that women take this medication with a full glass of water and remain upright for 30 minutes.
Osteoporosis is a skeletal disorder characterized by a loss of bone mass. The main determinants of peak bone mass and density are thought to be genetic with many genes identified that are related to bone quality.
The diagnosis of osteoporosis is made with a DEXA scan, which all major guidelines recommend be performed at age 65, with earlier screening reserved for women who are postmenopausal with risk factors. Diagnosis of osteoporosis in postmenopausal women is a T-score that is ≤ −2.5. Osteopenia is defined by a T-score between −1.0 and −2.5. Bone health should be addressed at each annual visit, with calcium, vitamin D, and weight-bearing exercises being recommended.
An 87-year-old woman presents with complete procidentia and multiple comorbidities. She does not desire surgery for this issue. Which of the following is the MOST appropriate pessary choice?
There are generally considered to be 2 types of pessary: support and space-filling. The support pessary exerts its support by using the pubic bone anteriorly and behind the cervix posteriorly. It generally is supported by these harder or more bony structures. A space-filling pessary is used more frequently in severe prolapse or complete procidentia.
A Gellhorn pessary is the most commonly used form of space-filling pessary. It is a complete concave disk that sets against the cervix or vaginal cuff and has a stem that points outward. This assists in placement and removal of the device.
Which of the following patients is a candidate to receive the meningococcal vaccine?
According to the CDC, the meningococcal vaccine should be offered to individuals in the following groups:
military recruits
microbiologists exposed to Neisseria meningitidis
individuals with functional or surgical asplenia
individuals with complement deficiencies
travelers to areas of the world with endemic meningococcal infection.
In a 42-year-old G2P2 woman with abnormal uterine bleeding, which of the following is MOST suggestive of adenomyosis as the primary cause?
Adenomyosis is the presence of endometrial glands and stroma in the musculature of the uterus. Heavy menstrual bleeding occurs because of the distortion of the uterine cavity from the displaced endometrial glands.
Symptoms of Adenomyosis
Heavy menstrual bleeding
Dysmenorrhea
Enlarged and boggy uterus
Dyspareunia
Symptoms of adenomyosis often mimic those with endometriosis without adenomyosis. However, of the options, the finding most suggestive of adenomyosis is an enlarged and boggy uterus.
Fibroids can form an enlarged uterus, but it usually isn’t boggy.
Five mismatch repair genes are strongly associated with Lynch syndrome: MLH1, MSH2, MSH3, MSH6, and PMS2.
A 22-year-old G0 woman presents to urgent care complaining of painful bumps around her genitals. She has a new partner, but he does not have any issues. On exam, there are multiple small, clear, fluid-filled vesicles over her vulva. The physician suspects genital herpes. What is the BEST way to confirm your diagnosis?
Polymerase chain reaction (PCR) assays have emerged as a sensitive method to confirm HSV infection in clinical specimens from genital ulcers.
Although viral culture remains the gold standard, the overall sensitivity is only 50%, and it typically requires at least 5 days.
Which of the following are appropriate first-line management strategies for a patient with newly diagnosed IBS?
Initial treatment of a patient with IBS should include dietary modification and increased physical activity. The low FODMAP diet (a diet low in fermentable oligo-, di-, and monosaccharides and polyols) and the strict traditional IBS diet (regular meal pattern; avoidance of large meals; reduced intake of fat, insoluble fibers, caffeine, and gas-producing foods such as beans, cabbage, and onions) have both been shown to reduce symptom severity in patients with IBS.
In a randomized trial of 102 patients with IBS, those assigned to the physical activity group showed a trend toward clinical improvement in the severity of IBS symptoms and a reduced likelihood of worsening of their IBS symptoms compared with controls.
Food diaries are a helpful tool for any patient with chronic gastrointestinal symptoms, to help identify foods that trigger or are associated with increased symptom frequency and/or severity.
If lifestyle and dietary modifications fail to adequately control the patient’s IBS symptoms, pharmacologic treatment with fiber supplements, laxatives, antidiarrheal agents, antispasmodic agents, and others may be considered.
Which of the following is the STRONGEST contraindication to the use of combined oral contraceptives?
TrueLearn Insight: Some commonly tested contraindications to combined hormonal contraceptive use include smoking at age ≥ 35 years, systolic blood pressure greater than 160 mm Hg or diastolic blood pressure greater than 100 mm Hg, high risk of DVT/PE, and migraines with aura. decompensated cirrhosis
On evaluation of a newborn, the baby is noted to have ambiguous genitalia. She undergoes an extensive evaluation, which includes labs notable for hyponatremia and hyperkalemia. What other lab value would be MOST likely to be elevated?
When evaluating an infant with ambiguous genitalia, it is imperative to obtain a karyotype to evaluate the sex chromosomes, serum electrolytes, and 17-hydroxyprogesterone levels.
Congenital adrenal hyperplasia (CAH) is the most common cause of ambiguous genitalia and is inherited in an autosomal recessive fashion. Ambiguous genitalia arise in 46,XX fetuses who are exposed to excess androgens during the time of external genital differentiation (9–14 weeks). The most common type of congenital adrenal hyperplasia (CAH) is due to a 21-hydroxylase deficiency, which is best assessed by measurement of 17-hydroxyprogesterone, which will be elevated.
A physician is seeing a 24-year-old patient in the emergency department 4 days after a sexual assault. She has no allergies, and urine pregnancy test is negative. She has never used contraception nor does she desire to start a long-term method; she also does not want to be pregnant. She has not received vaccines.
Which of the following medication regimens is the MOST appropriate at this time following a sexual assault?
The correct regimen is ceftriaxone, doxycycline, metronidazole, and ulipristal acetate.
Treatment after sexual assault should include prophylaxis for gonorrhea, chlamydia, trichomonas, and HIV for high-risk encounters within 72 hours of sexual assault. Patients should also be offered emergency contraception.
There are currently no recommendations that support treating a victim of sexual assault with prophylaxis for hepatitis C, as it is more commonly associated with dirty needles and transfusions than with sexual contact.
For HIV post-exposure prophylaxis, the CDC recommends treatment if the patient presents within 72 hours of assault and there is a substantial exposure risk.
For emergency contraception, options include intrauterine device placement or oral medications. This patient does not want to be on long-term contraception. Typically, ulipristal acetate is recommended within 5 days of unprotected intercourse, whereas oral levonorgestrel is only used within 3 days of unprotected intercourse. This patient is outside of this window for treatment with levonorgestrel.
A regimen of ceftriaxone, doxycycline, metronidazole, ulipristal acetate, and HIV post-exposure prophylaxis would be the correct option if the patient had presented within 72 hours of assault with a substantial exposure risk to HIV. She could alternatively be offered oral levonorgestrel instead ulipristal acetate. This option has medications for prophylaxis against gonorrhea, chlamydia, trichomoniasis, pregnancy, and HIV.
Which of the following findings would be suggestive of a higher fatality rate in a patient with known infective endocarditis caused by IV drug abuse?
IV drug abusers are at increased risk of developing infective endocarditis. The most common agent is Staphylococcus aureus, present in more than 50% of cases.
The mortality rate overall is approximately 3.6%. However, fungal etiology carries a 67% mortality rate with an odds ratio of 46.2.
A 35-year-old G1P1001 woman presents to the office for her postpartum visit. She had a vacuum-assisted vaginal delivery of a 9 lb 4 oz infant that was complicated by a third-degree perineal laceration. Her chief complaint is feculent, foul-smelling vaginal discharge and irritation of the vaginal introitus. The physician diagnoses her with a rectovaginal fistula. Her anal sphincter complex is intact, and the fistula is low in the vagina. What is the BEST surgical approach for repair of this fistula?
Rectovaginal fistulas may be acquired from obstetrical injury during delivery. When the anal sphincter complex is intact, a simple fistulectomy without sphincteroplasty can be performed via a vaginal or rectal route. If the anal sphincter complex has been compromised, a sphincteroplasty should be performed concurrently.
A 32-year-old G1P1 presents for counseling for a copper intrauterine device (IUD). Which of the following is the MOST common side effect?
dysmenorrhea and menorrhagia. not irregular bleeding.
Which of the following describes the normal physiological events during the menstrual cycle that are responsible for the development of fibrocystic change in the breast?
During days 14–28 (luteal or secretory phase) of the menstrual cycle, the theca externa produces progesterone. This leads to an increase in progesterone at day 14, after which it becomes the predominant hormone.
In the breasts, progesterone stimulates glandular growth; overstimulation of these glands can lead to cyst formation, which in turn leads to fibrocystic change of the breast.
The theca interna does not produce progesterone. It produces estrogen during the proliferative phase of the menstrual cycle. Moreover, progesterone is responsible for glandular growth, not ductal growth, in the breasts.
Which of the following is the MOST effective smoking cessation therapy?
The most effective method for smoking cessation is the combination of behavioral therapy and pharmacologic treatments. Appropriate pharmacologic treatments include buproprion, varenicline, and nicotine replacement therapy.
Compared with women who are nonsmokers, women who smoke cigarettes have greater risks of:
Reproductive health problems
Many forms of gynecologic cancer and other types of cancer
Coronary and vascular disease
Chronic obstructive lung disease
Osteoporosis.
Behavioral counseling and the use of evidence-based smoking cessation aids are the most effective strategies for achieving smoking cessation, even for very heavy smokers, with a 6-month abstinence rate of 25–35%.
Varenicline is a partial agonist for a subunit of the nicotinic acetylcholine receptor and prevents nicotine stimulation of mesolimbic dopamine system associated with nicotine addiction. this is the most effective med method. Varenicline stimulates dopamine activity but to a much smaller degree than nicotine does, resulting in decreased craving and withdrawal symptoms.
Which of the following is TRUE regarding intrauterine device (IUD) use and the risk of ectopic pregnancy?
Intrauterine devices (IUDs) are associated with a lower incidence of ectopic pregnancy due to the overall lower incidence of pregnancy.
However, if a woman becomes pregnant with an IUD, the risk of ectopic pregnancy is higher than if she had no IUD. One study demonstrated the risk of ectopic pregnancy with copper IUDs (1 in 50 pregnancies) was much lower than with levonorgestrel IUDs (1 in 2 pregnancies). Previous IUD use is associated with a slightly increased risk of ectopic pregnancy compared with the risk in the general population.
You are seeing a patient in the office who complains of abnormal discharge as well as postcoital bleeding. After a thorough exam, you diagnose the patient with cervicitis. She asks you if her cervicitis is due to her sexual activity. You inform the patient that her cervicitis may very well be secondary intercourse, as all of the following are sexually transmitted causes of cervicitis EXCEPT:
Chlamydia trachomatis, herpes simplex, human papillomavirus, and trichomonas vaginalis are all sexually transmitted causes of cervicitis. Although gardnerella vaginalis is known to cause cervicitis as well, it is not currently considered to be sexually transmitted
A 47-year-old G1P1001 woman with chronic pelvic pain presents requesting a hysterectomy. What should the physician counsel her is the rate of persistent pelvic pain following hysterectomy in a patient with no identifiable pelvic pathology?
More than one-third of patients fitting this case will have persistent pelvic pain. In patients with no identifiable pelvic pathology, the rate of persistent pelvic pain following hysterectomy is 40%.
Which of the following is associated with intimate partner violence (IPV)?
Intimate partner violence (IPV) can include psychological, physical, or sexual abuse; progressive isolation; stalking; deprivation; intimidation; or reproductive coercion.
In the United States, women experience 4.8 million incidents of physical or sexual assault annually; however, the true incidence of IPV is unknown. This is most likely because many victims are afraid to disclose their personal experiences of violence. In addition, the severity of IPV can increase during pregnancy and the postpartum period. In fact, homicide is a leading cause of maternal mortality with the majority involving former or current intimate partners. IPV has been associated with poor pregnancy weight gain, infection, anemia, tobacco use, stillbirth, pelvic fracture, placental abruption, fetal injury, preterm delivery, and low birth weight.
Which of the following is the CORRECT lifetime probability of urinary tract infections in women?
Urinary tract infections (UTIs) are exceedingly common, particularly in females.
More than half of women will have at least one UTI during their lifetime, and 3%–5% of all women will have multiple recurrences. In fact, an estimated 11% of US women report at least one physician-diagnosed UTI per year, and the lifetime probability that a woman will have a UTI is 60%.
Which of the following gynecological entities has been documented as a cause of appendicitis?
Appendicitis can be caused by gynecological pathology including an ectopic pregnancy, right-sided salpingitis, a tubo-ovarian abscess, PID, and a torsed ovarian cyst.
Several gynecologic conditions can mimic appendicitis, and should all be considered in the differential diagnosis.
Gynecological Causes of Appendicitis
ectopic pregnancy
right-sided salpingitis
a tubo-ovarian abscess
pelvic inflammatory disease
torsed ovarian cyst
A 71-year-old patient complains of intense vulvar itching and has significant irritation from scratching. Use of topical emollients and improved vulvar hygiene have not improved the patient’s symptoms after 3 weeks. Which of the following is the MOST appropriate next step?
This patient presents at the appropriate age and with the signs and symptoms of lichen sclerosus. She is having itch-scratch cycles which occur at night and in anal and genital areas. She has failed conservative therapy, which is not uncommon. After 3 weeks, the appropriate next step is a biopsy of the concerning lesion.
This patient, having a likely diagnosis of lichen sclerosus, has a 4–6% chance of developing a vulvar cancer, much higher than the general population. The biopsy on that lesion, if lichen sclerosus, often reveals hyperkeratosis with thinning of the rete ridges and plugging of follicular infundibulum.
All the other options listed may be options to use, but cancer should be ruled out and a diagnosis made when initial treatment is ineffective and diagnosis is not biopsy-proven.
A 30-year-old woman presents to the office for chronic pelvic pain. She complains of intermittent pain in her left labium for the past 2 months. The pain is moderate in intensity and waxes and wanes. She has never been pregnant. She has type 2 diabetes mellitus, but this has been well controlled for several years. The only surgery she has had is a diagnostic laparoscopy, which did not reveal endometriosis or any visible anatomic cause of her pain. Her vital signs are normal today. A physical exam reveals no abdominal pain and no point tenderness or cervical tenderness. She denies a bulge in her labium. Urinalysis reveals 3+ blood, no leukocytes, and no nitrates. Urine pregnancy test is negative. Further detailed history reveals that she notes left back pain that consistently precedes the labium pain. What is the MOST LIKELY diagnosis?
Kidney stones (also called nephrolithiasis) are a common complaint in the primary care setting. They commonly present with intermittent back/flank pain that waxes and wanes. As the stones move farther down the ureter, the location and nature of the pain can change. Ultimately, the patient may experience pain in the labium.
Hematuria in such a patient is very consistent with kidney stones. However, a patient with classic symptoms of kidney stones without hematuria can still have kidney stones.
Obesity is a risk factor associated with which of the following cancers?
Obesity is a risk factor for the following cancers more common in women:
Endometrial cancer
Obese and overweight women are two to about four times as likely as normal-weight women to develop endometrial cancer
Liver
Kidney
Colorectal
slightly (about 30%) more likely to develop colorectal cancer
Breast
among postmenopausal women, those who are obese have a 20% to 40% increase in risk of developing breast cancer compared with normal-weight women
Ovarian
associated with (10%) increase in the risk in women who have never used HRT
Thyroid
slight (10%) increase in the risk of thyroid cancer
A 37-year-old G1P1 woman with endometriosis presents to clinic. Which of the following is CORRECT in counseling this patient about endometriosis?
Endometriosis is a chronic gynecological disorder that can often manifest as pelvic pain, infertility, dyspareunia, or lower back pain. Endometrial lesions can lead to a chronic inflammatory disorder, causing severe pain. Risk factors include early menarche occurring before age 11 years, shorter menstrual cycles occurring less than monthly, and heavy prolonged periods.
Incidence of endometriosis in women of reproductive age is 6–10%. There is no evidence to suggest that the incidence is increasing.
There has been a familial association for affected patients, specifically with first-degree relatives. There is a nearly 7-fold to 10-fold increased risk of developing endometriosis when this is the case.
What percentage of women experience pelvic pain shortly after a uterine artery embolization (UAE) due to a fibroid uterus?
Uterine artery embolization (UAE) occludes blood flow to the uterus and subsequently the fibroids. This procedure is typically performed under conscious sedation.
The procedure itself is short, and there is a predictable pain pattern afterward. As the blood within the fibroid clots, ischemia occurs, and women develop intense pelvic cramping. This occurs roughly 4–6 hours after the procedure. Given the physiology behind the pain, almost all women experience pelvic pain after a UAE procedure.
Hereditary nonpolyposis colorectal cancer refers to individuals and/or families who fulfill Amsterdam criteria. Approximately one-half of families that fulfill Amsterdam criteria have Lynch syndrome.
Amsterdam II criteria for Lynch syndrome:
There should be at least three relatives with any Lynch syndrome-associated cancer (colorectal cancer, cancer of the endometrium, small bowel, ureter, or renal pelvis)
One should be a first-degree relative of the other two
At least two successive generations should be affected
At least one should be diagnosed before age 50
Familial adenomatous polyposis should be excluded in the colorectal cancer case(s), if any
Tumors should be verified by pathological examination
Screening and Surveillance Recommendations for Women With Lynch Syndrome:
Colonoscopy every 1–2 years, beginning at age 20–25 years, OR every 2–5 years before the earliest cancer diagnosis in the family, whichever is earlier
Endometrial biopsy every 1–2 years, beginning at age 30–35 years
Keeping a menstrual calendar and evaluating abnormal uterine bleeding
What is the BEST next diagnostic step in a patient who has irritability, depression, and headache that is disruptive to her social life and is associated with her menses?
This patient is having classic symptoms of premenstrual syndrome. This syndrome presents as affective symptoms (angry outbursts, depression, irritability, confusion, social withdrawal, fatigue) coupled with somatic symptoms (bloating, headache, swollen extremities, breast tenderness) and evidence of dysfunction in social or economic performance in a temporal relationship with menses.
Per ACOG, symptoms must start 5 or more days prior to the onset of menstruation and resolve within 4 days after the onset of menses. Diagnoses can be made with a symptom diary detailing daily symptoms for 2–3 menstrual cycles.
Other psychiatric disorders can be worsened with premenstrual syndrome and therefore psychiatric screening should be considered. Other systemic medical conditions (hypothyroidism, systemic lupus, chronic fatigue syndrome, etc.) should be considered as well.
For which of the following male conditions would it be advisable to offer a testicular biopsy to retrieve sperm for possible intracytoplasmic sperm injection (ICSI)?
Infertility is defined as the inability to achieve pregnancy after 12 months of appropriately timed intercourse. However, in patients older than age 35, workup and treatment may begin after 6 months of appropriately timed intercourse without conception.
An evaluation for infertility typically consists of a semen analysis in the male partner. In addition, male partners should have a complete medical, surgical, and reproductive history performed. An endocrine evaluation is recommended when the sperm count is less than 10 million/mL with signs of impaired sexual function or an endocrine abnormality. A basic workup includes FSH and testosterone; if results are abnormal, ordering LH, free testosterone, and prolactin levels is recommended.
There are many causes of oligospermia, and assessing laboratory studies as well as physical exam findings can typically clue one in to the diagnosis. A karyotype is indicated in men with severe oligospermia less than 5 million/mL. Microdeletions in the azoospermic factor region (AZF) can lead to oligospermia or azoospermia. Deletions in the AZF a or b region lead to azoospermia, and typically sperm cannot be extracted even with a testicular biopsy. Thus, couples with male partners affected by this diagnosis are advised to use donor sperm to conceive.
Although deletions in the AZF c region are associated with severe oligospermia, these men have the ability to have sperm extracted during a testicular biopsy, and their partners can subsequently undergo in-vitro fertilization (IVF) with intracytoplasmic sperm injection (ICSI).
Which of the following is the MOST common sign or symptom in a woman diagnosed with systemic lupus erythematosus (SLE)?
Fatigue is the most common symptom noted in women with systemic lupus erythematosus (SLE). Depending on the study, fatigue is present in 74%–100% of women at any given time. Other symptoms suggestive of the disease are arthralgia, fever, photosensitivity, butterfly rash, and alopecia.
SLE is a chronic autoimmune disorder affecting multiple organs. It is more prevalent among women than men, especially during their reproductive years.
Risk factors associated with the disease include:
genetic predisposition
long estrogen exposure (early menarche, late menopause, OCPs)
exposure to Epstein–Barr virus
exposure to ultraviolet light
Which of the following CORRECTLY describes the ages and vaccine schedule for the human papillomavirus quadrivalent vaccine?
The quadrivalent human papillomavirus (HPV) vaccine is indicated for use between ages 11–26 (but it may be given from the age of 9 years) as part of the adolescent immunization platform).
Dosing schedule varies depending on the age of when the first dose was given:
Between 9–14 years old: only two doses of vaccine are sufficient at 0 and 6–12 months
Between ages 15–26: includes three doses, separated at 0, 2, and 6 months
Recent ACOG recommendations explain the need of a shared-clinical decision with women aged 27–45 years who are previously unvaccinated regarding the HPV vaccination, considering the patient’s risk for acquisition of a new HPV infection and whether the HPV vaccine may provide benefit.
What is the earliest age to consider herpes zoster vaccination in an otherwise healthy female patient?
Per Centers for Disease Control and Prevention, the recombinant zoster vaccine Shingrix is recommended for immunocompetent adults aged 50 years and older.
Shingles is a painful rash that can develop from herpes zoster and potentially result in long-lasting pain called postherpetic neuralgia (PHN). Shingles is the result of reactivation of varicella zoster virus (VZV).
Dosage Adults 50 years and older
2 doses
Second dose 2–6 months after the first
*It is not necessary to screen (either verbally or via laboratory serology) for a history of varicella. However, if a person is known to be varicella-negative via serologic testing, guidelines for varicella vaccination should be followed.
A patient requires a repeat laparoscopy for endometriosis symptoms following a total laparoscopic hysterectomy with bilateral salpingo-oophorectomy. Where would it be MOST likely to find lesions in this patient?
Women with endometriosis may continue to have symptoms following treatment with hysterectomy with oophorectomy. There has been shown to be a recurrence rate of about 15% following bilateral oophorectomy regardless of whether a woman has hormone replacement after surgery or not. Lesions are most often found on the small and large bowel when repeat surgery is performed as these lesions may have been present but not excised at the initial surgery. Treatment of these lesions with estrogen suppression or aromatase inhibitors has not been shown to be helpful. These lesions likely persist due to local expression of aromatase activity and surgical excision is often required.
In general, common findings on laparoscopy in a patient with endometriosis include the following:
powder-burn or purplish-blue lesions throughout the peritoneal surfaces
old brown scarring lesions
red or white lesions
Allen-Masters window (a peritoneal defect).
Disease is often found where one might expect to see direct spread from retrograde menstruation in the posterior cul-de-sac or at the adnexa. Dense adhesions may be found as well and should be completely resected, as simple lysis often leads to undiagnosed endometriosis in the adhesions, which will return. Peritoneal stripping should be attempted when possible, and removal of an entire cyst should be performed as well to ensure that as little disease as possible is left in the abdomen. Furthermore, a 20% rate of normal-appearing laparoscopy findings is observed in those undergoing diagnostic laparoscopy, and a biopsy should always be taken for tissue diagnosis. Endometrial epithelium, glands, and stroma, as well as hemosiderin-laden macrophages, are common on histopathology.
Which of the following is the BEST treatment regimen for a patient with diabetes who has a complicated urinary tract infection?
Levofloxacin 750 mg one tab PO daily for 10 days is an appropriate treatment.
Urinary tract infections (UTIs) are exceedingly common, particularly in females. More than one-half of women will have at least one UTI during their lifetime, and 3%–5% of all women will have multiple recurrences. There is some concern regarding multidrug resistant bacteria and fluoroquinolone resistance. Depending on the area you are practicing, this may determine which medications you use. Generally, fluoroquinolones are still first-line treatment for complicated cystitis.
Uncomplicated acute bacterial cystitis was previously treated with 7–10 days of antimicrobial therapy, but recent data have shown that 3 days of therapy is equivalent in efficacy. Complicated UTIs, however, require longer durations and higher antimicrobial dosages because of their high risk of failure. Patients with diabetes, renal failure, function or anatomic abnormalities of the urinary tract, renal transplantation and an indwelling catheter stent are more likely to fail more conservative management and are considered to have complicated UTIs.
First-Line Treatment for Complicated UTIs
Medication Dose Duration
Ciprofloxacin 500mg PO BID 7–14 days
Levofloxacin 750mg PO Daily 7–14 days
Ciprofloxacin extended release 1000mg ER PO Daily 7–14 days
*If multidrug-resistant organisms (MDR) are present or the community has >10% known resistance of E. coli to fluoroquinolones, complicated UTIs can be first treated with IV medication and then finish with oral treatment.
**Of note, there is a black box warning for fluoroquinolones. Fluoroquinolones are associated with tendinitis/tendon rupture, peripheral neuropathy, and CNS effects. Tendonitis/tendon rupture can occur during treatment or months after. There is increased risk for patients >60 yo, those taking corticosteroids, and patients with kidney/heart/lung transplant.
Which of the following is the CORRECT percentage of gallstones that are radiopaque?
Cholesterol is the main component of approximately 80% of gallstones, with 10% being purely cholesterol. Pigment stones tend to have calcium bilirubinate as the main component and thus are visible on plain radiographs.
However, because only 15%–20% of gallstones are radiopaque, plain radiography is a poor screen examination and has largely been replaced by abdominal ultrasonography, which relies on shadowing and surrounding characteristics to determine the presence of gallstones.
Obesity is considered a risk factor for which of the following malignancies?
Obesity is a risk factor for the following malignancies:
endometrial
kidney
gastric cardia
colon
rectal
biliary tract
pancreatic
breast
esophageal adenocarcinoma
ovarian
multiple myeloma
hepatocellular carcinoma
meningioma
Which of the following is TRUE regarding treatment of lichen sclerosus?
The recommended treatment of lichen sclerosus is a high-potency topical steroid, usually clobetasol propionate.
Although there is a lack of randomized controlled clinical trials, a reasonable regimen is:
begin with once-daily application for 4 weeks
taper to alternate days for 4 weeks
followed by 4 weeks of twice-weekly application
Which of the following is considered a Lynch-related tumor?
Lynch syndrome, otherwise known as hereditary nonpolyposis colorectal cancer, is a hereditary cancer syndrome caused by a defect in mismatched repair genes MLH1 and MSH2, among others. Lynch-related tumors do not include breast cancer, although this does play a major role in another hereditary cancer syndrome, BRCA syndrome. Lynch-related tumors include colorectal, endometrial, stomach, ovarian, pancreatic, ureter and renal pelvis, biliary tract, and brain (usually glioblastoma as seen in Turcot syndrome) tumors, sebaceous gland adenomas and keratoacanthomas in Muir-Torre syndrome, and carcinoma of the small bowel.
A 21-year-old nulliparous woman presents to the office complaining of a painless vulvar lesion of a few days’ duration. She first noticed the lesion a few weeks after beginning a new sexual relationship. She has no medical problems and has never had surgery. The lesion is on her labia minora and is approximately 1 cm in diameter. Unroofing the lesion and looking under dark-field microscopy reveals spirochetes. Which of the following is the MOST appropriate treatment regimen for this patient?
Primary syphilis presents with the appearance of a painless vulvar ulcer that contains the spirochete Treponema pallidum. The treatment of primary syphilis is benzathine penicillin G, 2.4 million units IM in 1 dose. Alternative regimens (penicillin-allergic, nonpregnant patients) include doxycycline, 100 mg orally BID for 2 weeks, or tetracycline, 500 mg PO QID for 2 weeks.
Aqueous crystalline penicillin via the IV route is the treatment of choice for neurosyphilis. Alternative regimens include procaine penicillin, 2.4 million units IM daily for 10–14 days, plus probenecid, 500 mg PO QID for 10–14 days.
Benzathine penicillin G, 2.4 million units IM every week for 3 doses, is the treatment of choice for late latent syphilis, evidenced by >1 year duration, gummas, and cardiovascular syphilis. A total of 7.2 million units should be administered. Alternative regimens include doxycycline, 100 mg PO BID for 2 weeks if <1 year or 4 weeks if unknown duration or >1 year, as well as tetracycline, 500 mg PO QID for 2 weeks if <1 year, otherwise for 4 weeks.
Doxycycline, 100 mg PO twice daily for 28 days, is the acceptable alternative regimen for late latent syphilis of >1 year duration.
Procaine penicillin and probenecid is the alternative regimen for neurosyphilis if aqueous crystalline penicillin cannot be used.
What is the prevalence of a uterine sarcoma in women undergoing hysterectomy or myomectomy for a presumed leiomyoma?
Uterine sarcomas are significantly rarer than leiomyomas (3 to 7/100,000 in the United States population) and have a poor prognosis. The prevalence of a uterine sarcoma in the setting of surgery for a presumed leiomyoma ranges from 0.05 to 0.28% and is an active area of current investigation. This is a rare but relevant risk. The majority of cases are leiomyosarcoma, a highly aggressive cancer, but there are also reports of cases of endometrial stromal sarcoma, generally a more indolent cancer. Most of these are identified on histology after their removal for presumed benign conditions.
A uterine sarcoma is difficult to identify pre-operatively. The sensitivity of an office endometrial biopsy or dilation and curettage (D&C) to detect sarcomatous elements is lower than that for endometrial carcinomas. Specifically, with leiomyosarcoma, symptomatic women receive a correct preoperative diagnosis in only 35–50% of cases. This inability to accurately sample the tumor is probably related to the origin of these neoplasms in the myometrium rather than the endometrium.
Preoperative consultation with a gynecologic oncologist is recommended for any patient with a biopsy suggesting uterine sarcoma.
Which of the following neurotransmitters causes bladder contraction when released?
Acetylcholine is the neurotransmitter that is released to cause bladder contraction.
This is the neurotransmitter that is targeted by a class of antispasmodics/anticholinergic medications to treat overactive bladder and urge incontinence.
Which of the following is a common feature of systemic lupus erythematosus (SLE)?
Systemic lupus erythematosus (SLE) is not associated with weight gain. However, patients may report other constitutional symptoms, such as weight loss, fatigue, and lymphadenopathy.
Answer A: Patients with SLE may have dyspnea or chest pain associated with serositis, as well as findings that suggest pleural effusion, pneumonitis, or interstitial lung disease.
Answer B: Painless mucosal oral or nasal ulcers are a common finding in patients with SLE.
Answer C: A pathognomonic finding for SLE is a malar rash, also described as a butterfly rash, associated with skin photosensitivity.
Answer D: Raynaud phenomenon refers to the discoloration and decreased blood flow noted at the tips of the fingers and toes in response to cold or emotional stress. Raynaud phenomenon is a common complaint of patients with SLE.
What is the lifetime incidence of depression in women in the United States?
The lifetime incidence of depression is estimated at 17% for women in the United States. This is nearly double the rate for men.
Depression is more common in younger women than in older women.
Which of the following IS a common side effect of oxybutynin use?
Oxybutynin is an anticholinergic that antagonizes acetylcholine at the muscarinic receptor to relax the bladder and inhibit involuntary detrusor contractions.
This medication is commonly used for treatment of overactive bladder or urge incontinence, and the side effects are well known and often tested.
Side Effects of Anticholinergic Medications
Constipation
Dry mouth
Dizziness
Somnolence
Confusion
Double vision
Hyperkalemia
Prolonged QTc interval
Oxybutynin is an anticholinergic that antagonizes acetylcholine at the muscarinic receptor to relax the bladder and inhibit involuntary detrusor contractions. This medication is commonly used for treatment of overactive bladder or urge incontinence, and the side effects are well known and often tested.
Which of the following describes the primary pathophysiology of dysmenorrhea?
This patient has dysmenorrhea or, more generally, cyclic pelvic pain. When this occurs, multiple etiologies may be possible. However, when the pain is cyclic, menstrual-related disease is most often given credit as the cause of the pain. Whether the pain is simply the result of an amplified pain response or the result of endometriosis, the pathophysiology is essentially the same. Estradiol is produced as a result of increased aromatase activity. In turn, COX-2 is stimulated in uterine endothelial cells. PGE2 production then ensues, leading to a pain response after binding to pain receptors and creating inflammation. These mechanisms are the basis for aromatase inhibitor use in endometriosis.
A physician is seeing a patient in the office for contraceptive counseling. She is interested in an oral contraceptive pill; however, she is concerned that she will not be able to get a prescription for it, as her best friend was recently told by her doctor that her medical issues excluded her from using the oral contraceptive pill. While counseling the patient, the physician informs her that contraindications to the combined oral contraceptive pill include which of the following?
The main contraceptive efficacy of combined oral contraceptives (COCs) is suppression of ovulation by inhibition of gonadotropin-releasing hormone (GnRH), luteinizing hormone (LH), follicle-stimulating hormone (FSH), and the mid-cycle LH surge. This effect is mediated by both the progestin and estrogen components of the COC working synergistically, but estrogen suppression of FSH, which in turn prevents folliculogenesis, is likely the most important mechanism.
Combined hormonal methods have been linked to a small risk of stroke and heart attacks. They are not recommended in patients older than 35 years who smoke; have high blood pressure or a history of stroke, heart attack, or deep vein thrombosis (DVT); have a history of migraine headaches with aura; have certain medical conditions; or have breast cancer or a history of breast cancer.
Answer A: Women who smoke >15 cigarettes per day and are over the age of 35 years are at significant risk of a thrombotic event; adding a prothrombotic in hormonal form is contraindicated in these patients.
Answer B: Oral contraceptive pills (OCPs) may be considered in healthy, nonsmoking women older than 35 years if there are no other contraindications to combined hormonal contraceptives.
Answer C: There is a very small risk that the estrogen in these methods can affect milk supply if a patient is breastfeeding. Patients should avoid these methods for the first 4 to 6 weeks after childbirth, until breastfeeding is established.
Answer E: Migraine headaches with aura have been associated with up to a twofold increased risk of stroke in otherwise healthy women taking OCPs. Smoking further increases this risk. For this reason, migraine headache with aura is a contraindication to combined hormonal contraceptives. Stroke risk is not increased in patients with migraine without aura; therefore, combined hormonal contraceptives are not contraindicated unless the patient has other major risk factors for stroke (e.g., smoking, hypertension, diabetes).
Bottom Line: Combined hormonal methods have been linked to a small risk of stroke and heart attacks. They are not recommended if you are older than 35 years and smoke; have high blood pressure or a history of stroke, heart attack, or deep vein thrombosis; have a history of migraine headaches with aura; have certain medical conditions; or have breast cancer or a history of breast cancer.
According to the American College of Obstetricians and Gynecologists, which is the MOST correct statement regarding lipid profile assessment?
ACOG endorses the American College of Cardiology (ACC) and American Heart Association (AHA)’s 2013 blood cholesterol guidelines, which recommend all adults aged 21 years or older be screened for dyslipidemia with a fasting lipid panel and have their overall cardiovascular disease risk assessed every 4–6 years.
Women with a normal lipid screen at age 21 who are not at high risk for cardiovascular disease should have a repeat lipid screen at age 45 years; whereas women with a normal lipid screen at age 21 years who are at high risk for cardiovascular disease should have a repeat lipid screen at age 35.
Which of the following is recommended for the prevention of urinary tract infections in a patient of reproductive age?
Recurrent urinary tract infection (UTI) refers to ≥ 2 infections in 6 months or ≥ 3 infections in 1 year. UTI recurrences are typically acute simple cystitis rather than complicated UTI. Most recurrences are thought to represent reinfection rather than relapse (even recurrences caused by the same uropathogenic strain), although occasionally a persistent focus can produce relapsing infection.
Recurrent simple cystitis is common among women, even among young, healthy women who have anatomically and physiologically normal urinary tracts. In a study of college women with their first episode of cystitis, 27% experienced at least 1 culture-confirmed recurrence within the 6 months following the initial infection. When the first infection is caused by Escherichia coli, women appear to be more likely to develop a second cystitis episode within 6 months than those with a first infection due to another organism.
Patients should be counseled on the risk factors for recurrent simple cystitis and behavioral changes (such as increased fluid intake) that might reduce the risk. For postmenopausal women, vaginal estrogen is recommended. Otherwise healthy women with recurrent cystitis should increase their fluid intake, which can reduce the risk of recurrence. Although the optimal amount of fluid is unknown, a general daily target of 2–3 liters of fluid is encouraged.
Answer B: Vaginal estrogen is recommended for postmenopausal women with recurrent cystitis to reduce the incidence of cystitis.
Answer C: Clinical studies to date have not definitively demonstrated efficacy in prevention of recurrent simple cystitis with cranberry products.
Answer D: It is reasonable to suggest to women that early postcoital voiding might be helpful. However, this has not been shown in controlled studies to result in a reduced risk of recurrent cystitis.
Answer E: Clinical trials of oral probiotics have not been encouraging; studies do not suggest probiotics reduce the risk of recurrent cystitis. Given the lack of demonstrated clinical efficacy, providers should not routinely recommend them to their patients.
A 23-year-old woman presents to the clinic with symptoms of endometriosis and infertility. She would like to become pregnant as soon as possible. Which of the following is the MOST appropriate option for this patient?
This is a patient with symptoms of endometriosis but with a goal of fertility. She is presenting to the clinic with the hope of identifying a means to become pregnant.
It appears that the cause of her symptoms is endometriosis, and a laparoscopy will be both diagnostic and curative in many cases of endometriosis. The other options listed are commonly used to reduce the symptoms of endometriosis and are reasonably successful. However, none of the other options is faster than diagnostic laparoscopy: the gold standard for endometriosis.
A 29-year-old woman presents to the office with complaints of pelvic pain and abnormal discharge after having sexual intercourse with a new male partner 2 weeks ago. On examination, cervical motion tenderness is noted. A urine pregnancy test is negative, and she is otherwise healthy and asymptomatic. What is the next BEST step in treatment?
Pelvic inflammatory disease (PID) refers to acute infection of the upper genital tract structures in women, involving any or all of the uterus, fallopian tubes, and ovaries and may involve the neighboring pelvic organs. The 2 most important sexually transmitted organisms associated with acute PID, Chlamydia trachomatis and Neisseria gonorrhoeae, should be the main targets for treatment. There is favor for treatment of possible anaerobic organisms as well.
The presumptive clinical diagnosis of PID is made in sexually active young women or women at risk for sexually transmitted infections (STIs) who present with pelvic or lower abdominal pain and have evidence of cervical motion, uterine, or adnexal tenderness on exam. Additional criteria to aid in the diagnosis include fever above 38.3°C (101°F), abnormal cervical or vaginal mucopurulent discharge, presence of abnormal white blood cell count, and recent diagnosis of gonorrhea or chlamydia.
There has been a persistent trend towards outpatient treatment of PID with only the minority of women being hospitalized. Outpatient treatment consists of ceftriaxone (500 mg IM x 1 dose if < 150 kg or 1,000 mg IM x 1 dose if > 150 kg), doxycycline 100 mg twice daily for 14 days, and flagyl 500 mg twice a day for 14 days. These treatment regimens cover the most common pathogens responsible for PID, including chlamydia, gonorrhea, and other anaerobic organisms, and cure rates are greater than 90% in studies.
Indications for Inpatient Treatment of Pelvic Inflammatory Disease
Pregnancy
Severe clinical illness
Lack of response to oral antibiotics
Adnexal abscess (TOA)
Persistent nausea and vomiting
Inability to adhere to therapy
Possible need for surgical intervention
Inpatient treatment options are cefoxitin 2 g IV every 6 hours IV and doxycycline 100 mg twice daily (oral and IV have the same bioavailabilty, and oral is preferred if tolerable due to pain associated with IV treatment) until clinical improvement is noted for at least 24 hours followed by transition to oral therapy with doxycycline for a 14-day antibiotic course (+/- metronidazole 500 mg twice daily x 14 days if pelvic abscess or trichomonas present).
Which of the following procedures needs to be performed in order to diagnose adenomyosis?
Adenomyosis is best characterized as the presence of endometrial tissue within the myometrium. The clinical features of the disease are menorrhagia and dysmenorrhea, especially when there is an enlarged uterus.
Imaging studies can assist in the differential, but ultimately, the diagnosis is made at the time of hysterectomy.
A 27-year-old woman presents complaining of abnormal uterine bleeding (AUB). On pelvic ultrasonography, the physician notices that the patient has an 8-cm type I fibroid. She does not desire surgery at this time and she wishes to maintain her future fertility. Which of the following options can be used to manage her bleeding symptoms as well as her uterine enlargement?
Uterine leiomyomas are solid neoplasms composed of smooth muscle cells and fibroblasts. Leiomyomas vary in size and location. Uterine leiomyomas are common and estimated to occur in up to 70% of women by menopause with approximately only 25% being clinically significant enough to require intervention. The prevalence rate of uterine leiomyomas is 2–3 times higher among Black women compared with White women.
Prolonged or heavy menstrual bleeding, with or without anemia, and the sequelae of uterine enlargement are the most common presenting symptoms of patients with uterine leiomyomas. Pelvic pressure, urinary frequency, and constipation also can result from the presence of large leiomyomas within the pelvis and are collectively referred to as bulk symptoms.
Uterine fibroids are a common cause of AUB and are the leading indication for hysterectomy. Although hysterectomy is a definitive treatment choice, medical therapy and myomectomy are also appropriate. Myomectomy is indicated in patients who want to maintain their fertility, especially when they have bulk symptoms (pelvic pressure, early satiety, and changes in bowel habits). A laparoscopic approach is appropriate when a skilled laparoscopic surgeon is available; however, an abdominal approach is also acceptable.
Treatment with GnRH agonists is associated with reduction in leiomyoma size and overall size of the uterus, decreased AUB-L and dysmenorrhea, and improvement in quality-of-life measures (i.e., days of bleeding, pelvic pressure, pelvic pain, urinary frequency, and constipation).
Transvaginal ultrasonography is a useful screening test to assess for leiomyomas. Sonohysterography is useful to distinguish between type 0, type 1, and type 2 leiomyomas. Hysteroscopy is useful to distinguish between type 2 and type 3 leiomyomas. Magnetic resonance imaging can be useful in surgical planning, determining vascularity and degeneration, and distinguishing between type 4 and type 5 leiomyomas.
Which of the following is the MOST appropriate pharmacologic agent for suppression of recurrent urinary tract infections?
For women with recurrent UTIs (two or more within 6 months), antibiotic prophylaxis can be started. Treatment has been shown to decrease the risk of recurrence by 95%. This treatment may be continued for 6–12 months prior to being reassessed.
If recurrent UTIs are associated with sexual activity, the patient may benefit from postcoital prophylaxis.
Options for antibiotic prophylaxis include nitrofurantoin, trimethoprim-sulfamethoxazole, trimethoprim, cephalexin, cefaclor, and fosfomycin. These medications are given as once-a-day dosing.
Answer B: Nitrofurantoin can be given as urinary tract infection prophylaxis but at the dose of 50–100 mg daily at bedtime. The every 6–12 hour dosing is recommended for a 7-day treatment of an acute urinary tract infection, not for daily prophylaxis.
Which of the following is the MOST appropriate pharmacologic agent for suppression of recurrent urinary tract infections?
For women with recurrent UTIs (two or more within 6 months), antibiotic prophylaxis can be started. Treatment has been shown to decrease the risk of recurrence by 95%. This treatment may be continued for 6–12 months prior to being reassessed.
If recurrent UTIs are associated with sexual activity, the patient may benefit from postcoital prophylaxis.
Options for antibiotic prophylaxis include nitrofurantoin, trimethoprim-sulfamethoxazole, trimethoprim, cephalexin, cefaclor, and fosfomycin. These medications are given as once-a-day dosing.
Answer B: Nitrofurantoin can be given as urinary tract infection prophylaxis but at the dose of 50–100 mg daily at bedtime. The every 6–12 hour dosing is recommended for a 7-day treatment of an acute urinary tract infection, not for daily prophylaxis.
Strategies approved to reduce the risk of ovarian cancer:
Combined oral contraceptives
Breastfeeding
BSO at the appropriate age
Salpingectomy may be considered, however is not currently recommended for risk reduction in those with BRCA mutation
BRCA 1/2 monitoring plan
Screening Guidelines
Age Breast cancer screening Ovarian cancer screening*
25–30 Breast MRI every 6 months
Clinical breast exam every 6 months Not recommended
>30 Breast MRI and mammography alternating at 6-month intervals
Clinical breast exam every 6 months Can consider annual US with CA-125
Which of the following is the MOST important modifiable risk factor for stroke in women?
The most important modifiable risk factor for stroke in women is hypertension.
A gradual increase in blood pressure above 120/80 mm Hg shows a linear increase in risk of stroke. A reduction of blood pressure may reduce the risk of stroke by 25%.
All of the listed factors are risk factors associated with stroke.
Answer B: Diabetes is associated with stroke. However, glycemic control has not been shown to be a cost-effective method to reduce strokes.
Answer C: Statin use decreases the risk of stroke by about 18%. This risk reduction may be appreciated in individuals with normal cholesterol levels as well.
Answer E: Smoking cessation is recommended for all women, but has not been shown to decrease the risk of stroke more efficiently than blood pressure.
In a healthy individual without risk factors, which of the following is an appropriate recommendation regarding pneumococcal vaccination?
All adults aged 65 years or older who have not previously received a pneumococcal conjugate vaccine should be vaccinated with either 1 dose of PCV15 followed by PPSV23 at least 1 year later, or 1 dose of PCV20. For patients who receive PCV15 and have an immunocompromising condition, cochlear implant, or cerebrospinal fluid (CSF) leak, PPSV23 can be given as early as 8 weeks after PCV15.
Vaccination is indicated in patients aged 19–64 with certain medical conditions.
Pneumococcal vaccination is an important preventive health care measure that substantially reduces the burden of pneumococcal disease in vaccinated individuals and in the population.
A 19-year-old woman presents to the emergency department 6 days after an elective termination of pregnancy at an outside facility. She is febrile, tachycardic, and hypotensive. Which of the following is the MOST appropriate first step in management of this patient with suspected septic abortion?
Resuscitation with IV fluids and early administration of IV fluids are imperative and are the first steps in resuscitation of any patient who is thought to be septic, regardless of the etiology, so long as their respirations and oxygenation are adequate. As such, establishing IV access as soon as possible is important.
Septic abortion is a form of spontaneous abortion or early pregnancy loss that is complicated by intrauterine infection. It is more common in women who have medical or surgical management versus spontaneous abortion.
Initial resuscitation is as follows:
Establish IV access for IV fluids and blood products
Obtain blood cultures
Initiate broad spectrum antibiotics
Evacuate the uterus of any contents
Antibiotics are typically given until the patient has been afebrile for 48 hours and has shown clinical improvement. Oral antibiotics are given for a total of 10 to 14 days. If the patient does not respond to antibiotics and uterine evacuation or has clostridial necrotizing myonecrosis, then total hysterectomy and adnexectomy should be performed.
There are different types of abortion and if hemodynamically stable, do not necessitate emergent surgery as in the case of a septic abortion.
Which of the following is the MOST common side effect of anticholinergic medications used for urge incontinence?
The primary treatment of urge incontinence is lifestyle changes and bladder training. Urge incontinence can be treated medically with anticholinergic medications or a beta-3 adrenergic agonist.
The most common side effect of anticholinergic medications is xerostomia.
The most common side effect of Mirabigron (beta-3 adrenergic agonist) is hypertension which occurs in about 11% of patients. All other side effects were reported as <5%.
What is the MOST likely follicle-stimulating hormone (FSH) level and estradiol level in a 23-year-old woman with Turner syndrome and primary amenorrhea?
Turner syndrome is classically associated with the 45,X karyotype but, in fact, can occur with numerous types of chromosomal arrangement, including with a Y chromosome due to mosaicism.
Karyotypes with a Y chromosome are important to identify early on, as there is increased risk of malignancy in the streak gonads that are present. The classic phenotype is short stature, a webbed neck, sexual immaturity, wide-spaced nipples, and low-set ears. Typically, this syndrome is recognized early in development, but if it is not, it can present with primary amenorrhea at the time of expected puberty. Sexual immaturity occurs because of failure of normal ovarian development, which is characteristic of hypergonadotrophic hypogonadism, according to WHO criteria. These girls typically have elevated FSH levels and low E2 levels.
However, those with mosaicism can also present with partial sexual development, delayed puberty, and irregular menses. Therefore, spontaneous conception can occur, although rarely. Typically, pregnancy can be achieved with oocyte donation; however, patients should be informed that pregnancy in women with Turner syndrome can have a mortality rate almost 100-fold more than that of the general population, typically from aortic dissection or rupture. Thus, extensive counseling is recommended, including strong consideration of a gestational carrier.
Screening for thyroid disease in a nonpregnant patient reveals a TSH level of 6.5 μU/mL (normal: 0.5–5 μU/mL) and free thyroxine level of 5.2 μg/dL (normal: 5–12 μg/dL). The physician informs her that the next BEST step is which of the following?
Subclinical hypothyroidism is defined as an elevated TSH with a normal free thyroxine level. Because these laboratory findings can frequently revert to normal, the initial managment of these laboratory findings is to repeat the tests.
With a mildly elevated TSH, they should be repeated in 1-3 months. If the TSH remains elevated with a normal free T4, the diagnosis of subclinical hypothyroidism is confirmed. With a TSH equal to over 15 μU/mL, the tests should be repeated in 1–2 weeks.
Initial evaluation of subclinical hypothyroidism
TSH When to repeat test
5–14.9 μU/mL 1–3 months
15 or greater μU/mL 1–2 weeks
Screening for thyroid disease in a nonpregnant patient reveals a TSH level of 6.5 μU/mL (normal: 0.5–5 μU/mL) and free thyroxine level of 5.2 μg/dL (normal: 5–12 μg/dL). The physician informs her that the next BEST step is which of the following?
Subclinical hypothyroidism is defined as an elevated TSH with a normal free thyroxine level. Because these laboratory findings can frequently revert to normal, the initial managment of these laboratory findings is to repeat the tests.
With a mildly elevated TSH, they should be repeated in 1-3 months. If the TSH remains elevated with a normal free T4, the diagnosis of subclinical hypothyroidism is confirmed. With a TSH equal to over 15 μU/mL, the tests should be repeated in 1–2 weeks.
Initial evaluation of subclinical hypothyroidism
TSH When to repeat test
5–14.9 μU/mL 1–3 months
15 or greater μU/mL 1–2 weeks
A new patient presents to the office with a history of recurrent pregnancy loss and a diagnosis of antiphospholipid antibody syndrome (APS). She is currently 14 weeks’ pregnant. What is the BEST treatment option to prolong this pregnancy?
Peripartum Management of Women with APS
History of thrombotic event Prophylactic heparin while pregnant and for 6 weeks postpartum
can be transitioned to warfarin after delivery
added benefit of aspirin 81 mg is unknown
No history of thrombotic event Prophylactic heparin and aspirin 81 mg while pregnant and for 6 weeks postpartum
may reduce pregnancy loss by 50%
Other
Prednisone
IV IG
Which of the following nerves, if damaged, will cause a footdrop injury?
The common peroneal nerve is one of the most commonly injured nerves during gynecological surgery. This nerve traverses the lateral portion of the calf and is likely to be injured during prolonged surgery and/or incorrect positioning while using stirrups. The abnormal compression causes the injury.
Damage to the common peroneal nerve causes footdrop injury. Correct positioning and limited time of surgery are paramount to preventing nerve injuries. Appropriate placement of retractors and judicious surgical dissection may also play a role. Understanding of proper nerve innervation and motor control is essential to properly diagnosing a postoperative nerve injury.
Swift diagnosis and treatment are the keys to regaining full function. Conservative therapy is warranted in most cases as injuries caused by compression or stretch due to positioning will resolve spontaneously with supportive care. Tricyclic antidepressants, anticonvulsants, GABA antagonists, serotonin and norepinephrine specific reuptake inhibitors can be effective. Physical therapy and avoiding positions that worsen nerve compression are also helpful.
Answer B: The femoral nerve is responsible for flexion of the hip, extension of the knee, and tendon reflexes. This large nerve also carries sensation to the anteromedial thigh, calf, and foot. Deep retractors or excessive hip flexion (such as candy-cane stirrups) can injure the femoral nerve.
Answer C: The genitofemoral nerve is responsible for sensation to the mons and labia majora. This nerve may be injured or transected during pelvic sidewall dissection, especially in oncologic lymph node dissection.
Answer D: The lateral femoral cutaneous nerve is responsible for sensation to the anteroposterior thigh. Deep retractor use and pressure on the psoas muscle may cause injury.
Answer E: The obturator nerve has motor control for adduction as well as sensation to the medial thigh. Retroperitoneal dissection or paravaginal repair surgery may interrupt its route.
Which of the following is a risk factor for vaginal mesh erosion?
Risk Factors for Vaginal Mesh Erosion
Poor wound healing (eg, due to vaginal atrophy, smoking, diabetes, history of bariatric surgery, use of immunosuppressive medications)
Vaginal dissection that was too superficial
Excessive tension on the mesh
Contaminated vaginal field during placement of the foreign body
A 27-year-old female presents to the office with complaints of bilateral milky breast discharge that she noticed 1 month ago. She reports that it occurs only with expression. Her history is otherwise benign. On examination, she has bilateral milky discharge with expression, there is no mass, and she denies pain. What is the next BEST step in management?
Medications that can lead to elevated prolactin levels include those listed in the table below. Serum prolactin levels secondary to drug-induced causes can elevate prolactin levels to 25–100 ng/mL.
Medications That Can Elevate Prolactin Levels
Antipsychotics
Domperidone
Metoclopramide
Methyldopa
Verapamil
Opioids
Nipple discharge is a common gynecological complaint, and milky discharge outside of pregnancy, or galactorrhea, is most commonly present. Most cases of galactorrhea are benign and are associated with elevated prolactin levels. Elevated prolactin levels can be caused by medications, pregnancy, trauma, endocrinopathies, hypothyroidism, prolactin-secreting adenomas, and stress.
Patients with galactorrhea should have a complete history and physical exam with focused diagnostic imaging as needed, usually reserved for females with pathologic or more concerning nipple discharge.
A physician removes an adnexal cyst on a woman with a positive pregnancy test. She is at 7 weeks by her LMP. There is the chance she may still have a viable intrauterine pregnancy. Given the potential for a viable pregnancy, what is the BEST next step?
The corpus luteum provides the necessary progesterone for pregnancy prolongation. The placenta takes over this function at around 10 weeks. If the corpus luteum is removed prior to 10 weeks, then progesterone supplementation is required.
There are 3 basic supplementation options. First is weekly 17-hydroxyprogesterone caproate injections through 10 weeks’ gestation. The second option is oral micronized progesterone, dosed at 200–300 mg orally through 10 weeks. The last option is 8% progesterone vaginal gel PLUS 100–200 mg of micronized progesterone orally. Any of these 3 regimens is sufficient.
A physician removes an adnexal cyst on a woman with a positive pregnancy test. She is at 7 weeks by her LMP. There is the chance she may still have a viable intrauterine pregnancy. Given the potential for a viable pregnancy, what is the BEST next step?
The corpus luteum provides the necessary progesterone for pregnancy prolongation. The placenta takes over this function at around 10 weeks. If the corpus luteum is removed prior to 10 weeks, then progesterone supplementation is required.
There are 3 basic supplementation options. First is weekly 17-hydroxyprogesterone caproate injections through 10 weeks’ gestation. The second option is oral micronized progesterone, dosed at 200–300 mg orally through 10 weeks. The last option is 8% progesterone vaginal gel PLUS 100–200 mg of micronized progesterone orally. Any of these 3 regimens is sufficient.
Which of the following is the MOST common female sexual dysfunction?
The most common female sexual dysfunction is sexual interest/arousal disorder. Overall, 43% of American women report experiencing sexual problems, and 12% report the problem causes personal distress. The prevalence of sexual distress increases from 10% among women aged 18–44 years to 15% among women 45–64 years old and then decreases to 9% in women aged 65–85 years old.
A 62-year-old postmenopausal woman presents to the office for an annual exam. She is doing well overall and her only significant medical history is well-controlled hypertension. On exam, the physician notes adnexal fullness. A transvaginal ultrasound is performed which shows a 7-cm simple cyst with no septations or papillary projections on the left ovary. She is otherwise asymptomatic. What is the BEST next step in management?
Simple ovarian cysts up to 10 cm can be safely observed and followed in women, given the extremely low risk of malignancy (even postmenopausal women). If there are septations, papillary projections, thick-walled borders, or cystic and solid components, these should be further assessed with tumor markers, and surgical intervention for diagnostic and therapeutic purposes should be considered
Which of the following is most likely to cause pancreatitis?
There are many known risk factors for developing acute pancreatitis. The most common causes are gallstones and alcohol use.
Hyperlipidemia is also a known cause as well, with levels generally above 1,000 mg/dL.
Causes of Pancreatitis
Gallstones (30-45%)
Excessive alcohol use
(30%)
Hyperlipidemia
Medication induced
(eg, HIV medications)
Mumps infection
Contraindications to Hormone Replacement Therapy
Coronary heart disease
Personal history of breast cancer
History of venous thromboembolic event or stroke
Active liver disease
Unexplained vaginal bleeding
High-risk endometrial cancer
A 28-year-old woman presents for a well-woman visit. She has a social history significant for alcoholism. She reports that she was in jail for the last year and was clean while in prison; however, she is fearful that she will relapse because of her recent release. After discussing medication options, the physician elects to place her on a medication that inhibits aldehyde dehydrogenase. Which of the following medications utilizes this mechanism?
Disulfiram inhibits aldehyde dehydrogenase, causing patients to feel fatigue, nausea, and a headache if they relapse.
There are many therapeutic options for patients who suffer from alcohol dependence.
Medications to treat alcohol dependence
First-line treatment
naltrexone
acamprosate
Second-line treatment
disulfiram
gabapentin
topiramate
baclofen
Which of the following is the BEST definition of a spontaneous miscarriage?
Spontaneous abortion is defined by the loss of a pregnancy prior to 20 weeks gestation, and less than or equal to 500 grams. These definitions are key in defining recurrent risks and help identify the underlying disorder leading to the miscarriage.
Which of the following statements is TRUE regarding the risk of an intrauterine device (IUD) and ectopic pregnancy in a patient with a prior history of an ectopic pregnancy?
Both copper and levonorgestrel IUDs are effective contraception. These IUDs do not increase the risk of ectopic pregnancy because they prevent pregnancy so effectively.
The proportion of pregnancies that are ectopic is higher with an IUD in place compared with when there is not an IUD in place. However, because IUDs decrease the pregnancy rate so effectively, the absolute risk of an ectopic pregnancy with an IUD in place is decreased compared with not having an IUD in place.
Answer C: The ectopic pregnancy rate of women with an IUD in place is 0–0.5 per 1,000 women-years, compared with the rate of 3.25–5.25 per 1,000 women-years in women without the use of contraception.
Which of the following is TRUE regarding HIV screening?
Women should be screened for HIV with every pregnancy.
Some states mandate screening at the initial visit as well as a repeat test in the third trimester or on presentation to the hospital in labor.
Screening in pregnancy allows for the identification of individuals who may benefit from treatment of HIV during pregnancy and labor and to reduce disease morbidity and vertical transmission.
Answer A: Homosexual and bisexual men should be considered for HIV screening as frequently as every 3–6 months.
Answer B: Patients with high-risk behaviors for the transmission of HIV should be screened every year.
Answer D: HIV testing is recommended for unscreened adolescents and adults aged 15–65 years, even in low-risk individuals. Persons outside this age range with high-risk factors for HIV should be considered for screening.
Which of the following is the BEST next step in the evaluation or management in a 15-year-old who has symptoms suggestive of primary dysmenorrhea, is not sexually active, and has attempted management with occasional over-the-counter ibuprofen?
Primary dysmenorrhea is defined as painful menstruation in the absence of pelvic pathology. Secondary dysmenorrhea is defined as painful menstruation with a pathologic cause.
Possible causes of secondary dysmenorrhea
Endometriosis
Obstructive Müllerian anomalies
Cervical stenosis
Ovarian cysts
Uterine polyps
Uterine leiomyomata
Adenomyosis
Pelvic inflammatory disease
Pelvic adhesions
Symptoms suggestive of secondary dysmenorrhea
Failure of empiric therapy
Mid-cycle bleeding or pain
Heavy menses
Severe pain immediately upon menarche
Renal anomaly
Progressively worsening symptoms
Family history of endometriosis
When primary dysmenorrhea is suspected, empiric therapy with a nonsteroidal anti-inflammatory drug (NSAID) is the first-line treatment. Often, patients may have tried self-treatment with NSAIDs but with incorrect dosing and timing. Therefore, the first evaluation should include a detailed evaluation of their treatment regimen. Ideally, the treatment should be started 1–2 days prior to menses and then continued through the first 2–3 days of bleeding. Other options include combined oral contraceptive pills or the levonorgestrel IUD.
For a woman with infertility who is found to have mild endometriosis during diagnostic laparoscopy, how does operative laparoscopy impact her infertility?
Endometriosis, a disease characterized by ectopic endometrial implants throughout the pelvis, negatively impacts fertility. Of women who present with infertility, 25–50% are estimated to also have endometriosis.
Laparoscopic treatment of minimal and moderate endometriosis improves the pregnancy and live birth rates in couples with otherwise unexplained infertility. In a meta-analysis, it was reported that operative laparoscopy nearly doubled the live birth or ongoing pregnancy rates (57 versus 34 total live births or pregnancies over 382 surgeries, odds ratio [OR] 1.94, 95% CI 1.20–3.16).
This patient has stress urinary incontinence. Urinary incontinence is a common problem, affecting approximately 44–57% of middle-aged and post-menopausal women.
Stress urinary incontinence (SUI) is the involuntary loss of urine with effort or physical exertion (ie, sporting activities, lifting heavy objects) or when sneezing or coughing. A midurethral sling is often the first-line treatment for stress urinary incontinence.
Answer A: Sacrocolpopexy is a surgical procedure to treat pelvic organ prolapse, not stress urinary incontinence. Patients who undergo surgical repair of their pelvic organ prolapse may present with postoperative stress urinary incontinence because the prolapse kinks and obstructs the urethra but this obstruction is alleviated when the prolapse is repaired.
Answer C: Intradetrusor onabotulinumtoxinA (also known as Botox A) is used for the treatment of overactive bladder, not stress urinary incontinence. Botulinum toxin is a neurotoxin that acts as a muscle paralytic by inhibiting the presynaptic release of acetylcholine from motor neurons at the neuromuscular junction.
Answer D: Mirabegron is a beta-agonist that activates the beta-3 adrenergic receptor in the detrusor muscle, causing muscle relaxation and increase bladder capacity. Beta-agonists are used to treat urinary urgency and urge incontinence, not stress incontinence.
Answer E: Urethral bulking agents are used to treat stress incontinence with intrinsic sphincter deficiency. The bulking agents are injected transurethrally into the periurethral tissue around the bladder neck and proximal urethra to increase urethral resistance. Urethral bulking agents are less effective than surgical procedures such as midurethral sling placement and are rarely used as a first line treatment for stress urinary incontinence.
A 23-year-old G1P1001 woman presents to the office for family planning. She opts for injectable DMPA. After 8 weeks of injections, she presents to the office complaining of irregular heavy bleeding, requiring 10 pads per day. On examination, the physician notes a moderate amount of bleeding in the vaginal vault, but an otherwise normal physical examination. She denies a history of heavy menstrual bleeding, coagulation problem, or any other medical problems. Which of the following is acceptable management for her bleeding?
Amenorrhea occurs in 12% of women during the first 3 months of use and 46% of women after 1 year of use of DMPA.
Despite this, 25% of women discontinue DMPA after the first year because of unscheduled bleeding. Light, unscheduled bleeding after DMPA administration may be treated with 5–7 days of NSAID treatment. Heavy bleeding may be treated with NSAIDs for 5–7 days or hormonal treatment with 1.25 mg of conjugated estrogen for 10–20 days.
Which of the following is a key feature of female sexual arousal/interest disorder?
A number of conditions lead to the broad diagnosis of female sexual dysfunction, but the key component is that the symptoms must be sufficient to result in personal distress. For female sexual arousal/interest disorder, the symptoms must be present for at least 6 months.
The DSM-V re-classifed female sexual dysfunction into five categories.
DSM-V Female Sexual Dysfunction
Female sexual arousal/interest disorder
Female orgasmic disorder
Genito-pelvic pain/penetration disorder
Substance/medication-induced sexual dysfunction
Other specified/unspecified sexual dysfunction
Which of the following is the MOST likely cause of recurrent pregnancy loss?
Uterine anomalies are associated with 15% of recurrent pregnancy losses. Anomalies include submucosal leiomyomas, endometrial polyps, intrauterine synechiae, Müllerian anomalies, or cervical insufficiency. Saline infusion sonohysterography is the imaging modality of choice to evaluate uterine anomalies.
Recurrent pregnancy loss was previously defined as three or more consecutive pregnancy losses; however, this has recently been redefined as two or more pregnancy losses that need not be consecutive. Pregnancy loss remains defined as a loss <20 weeks’ gestation or birth weight <500 g.
There are three generally accepted broad causes of recurrent pregnancy loss:
parental chromosomal anomalies
antiphospholipid antibody syndrome
uterine anomalies (detailed in the table below)
Other possible causes may include environmental toxins, endocrinopathies, or alloimmunity.
Uterine Anomalies Associated with Recurrent Pregnancy Loss
Submucosal leiomyomas
Endometrial polyps
Intrauterine synechiae
Müllerian anomalies
Cervical insufficiency
A positive antinuclear antibody (ANA) has not been clearly associated with an increased risk of recurrent pregnancy loss. Only lupus anticoagulant and anticardiolipin antibody are definitively associated with recurrent pregnancy loss, and therefore routine ANA testing is not recommended.
Which of the following is the CORRECT diagnosis for a patient with white, lacy, fernlike lesions on her oral mucosa and painful erythematous erosions on her vaginal mucosa?
Lichen planus is an inflammatory disorder of the genital mucosa most likely related to cell-mediated immunity; it can occur on any mucosal surface, including the oral and genital cavities. Approximately 1% of the population has oral lichen planus, and 20%–25% of women with oral lichen planus have genital vulvovaginal disease.
The classic presentation is white, reticulate, lacy or fernlike striae (Wickham striae) on mucous membranes along with pruritic, purple, shiny papules on the trunk, buccal mucosa, or flexor surfaces of the extremities. Deep, painful erosions in the posterior vestibule can extend to the labia minora and result in agglutination and distortion of normal labial architecture.
Which of the following is TRUE regarding the use of OCPs in a woman with endometriosis?
The differential diagnosis of chronic pelvic pain includes disorders of the genitourinary, gastrointestinal, and reproductive tracts.
Endometriosis can cause chronic pelvic pain, which usually occurs in a cyclical fashion and is due to the migration of endometrial glands and stroma outside the uterine cavity. NSAIDs are typically first-line therapy, but OCPs are equally efficacious.
OCPs’ mechanism of action results in a hypoestrogenic environment and lack of proliferation of these ectopic glands. They tend to be well tolerated; however, unfortunately for some, they can lead to rupture of large endometriomas within the first 6 weeks of therapy. The mechanism of this is not clear, particularly because smaller endometriomas (<3 cm) can undergo necrobiosis and resorption.
Caution should be used when prescribing OCPs to those with large endometriomas.
Answer A: Continuous-dose regimens are aimed at a complete suppression of the hypothalamic-pituitary-ovarian axis for a prolonged period and are well tolerated by patients. Intermittent use is also acceptable, but it may lead to increased dysmenorrhea at the time of menses.
Answer B: Historically, when OCPs were first mass-produced, higher estrogen contents were shown to have high efficacy. As techniques and synthesis have improved, lower estrogen formulations are equally as effective and are much better tolerated than those with >20 mcg of estradiol per pill.
Answer D: Continuous OCPs have been used successfully and have been shown to decrease symptomatology in approximately 80% of patients.
Answer E: The most common side effect with continuous OCP use is amenorrhea and concurrent weight gain and breast tenderness, and approximately 33% of patients discontinue use because of these adverse outcomes. Breakthrough bleeding, however, is more common in the 10-mcg dose pill than in the 20-mcg pill.
Which of the following is TRUE regarding pituitary prolactinomas?
Pituitary adenomas are the most common cause of acquired pituitary dysfunction and constitute ~15% of all intracranial tumors. Clinically, symptoms of galactorrhea, menstrual disturbances, or infertility may lead to its diagnosis. Most tumors are benign, and only ~0.1% of adenomas develop into frank carcinoma with metastasis.
Tumors are classified by their hormonal expression pattern as determined by immunohistochemistry and then further grouped by size into microadenomas (<10 mm in diameter) OR macroadenomas (>10 mm in diameter).
Microadenomas are typically diagnosed during evaluation of an endocrinopathy.
Macroadenomas frequently present with patient symptoms from invasion of surrounding structures. The anterior pituitary gland neighbors both the optic chiasm and cavernous sinuses. Disruption of the optic chiasm by suprasellar growth of the pituitary mass may create bitemporal hemianopsia, in which the outer portion of the right and left visual fields is lost.
The pituitary gland enlarges during pregnancy, primarily due to hypertrophy and hyperplasia of the lactotropes in response to elevated serum estrogen levels. Tumor growth leading to significant symptoms complicates ~2% of microadenomas and 21% of untreated macroadenomas. However, because significant expansion may lead to headaches or compression of the optic chiasm and blindness, visual field testing is considered in every trimester for women with macroadenomas.
In general, first-line treatment is medical for both microadenomas and macroadenomas. Specifically, women should receive a dopamine agonist such as the nonspecific dopamine-receptor agonist bromocriptine, or the dopamine-receptor type 2 agonist cabergoline.
Neurosurgical evaluation is mandatory when visual field defects or severe headaches are present. Neurosurgery is required for refractory tumors or those causing acutely worsening symptoms. The pituitary is approached through a transsphenoidal route whenever possible.
A 28-year-old patient with a history of premenstrual syndrome presents to the office with mood swings, irritability, cramping, and episodes of depression for 9 months only around the time of her menses. She is unable to go to work during these episodes. Which of the following is the BEST treatment?
Treatment for premenstrual syndrome (PMS) is based on either symptom reduction, or modification of underlying hormonal dysregulation.
Mild to moderate symptoms can be relieved by changes in lifestyle and/or diet. However, when symptoms begin to interfere with the patient’s life then medical treatment should be recommended in addition to changes in lifestyle and/or diet. If treatment fails or symptoms are severe then psychiatric referral may be necessary.
Well-controlled trials of selective serotonin-reuptake inhibitors (SSRIs) have shown these drugs to be efficacious and well tolerated.
Efficacy is highly variable with progesterone and estrogen. Data are limited in support of combination oral contraceptive pills for PMS.
Answer A: SNRIs can be used for the treatment of PMS, however SSRIs are the first line.
Answer C: Ibuprofen will help her dysmenorrhea, but will not resolve all her symptoms.
Answer D: Studies to examine the effects of combined OCP on premenstrual syndrome (PMS) found little difference in PMS symptoms between combined OC users and nonusers.
Answer E: GnRH agonists are usually reserved for when other treatment modalities have failed.
A 17-year-old presents for routine gynecologic care. She complains of painful and crampy menses, and reports that her periods are not particularly heavy. She is not sexually active. The first-line medication for this acts primarily by which mechanism?
Primary dysmenorrhea presents as cramping and pelvic pain at time of menses. It may be associated with heavy menstrual flow. It can be treated empirically with medications as there are no current laboratory or imaging diagnostic criteria.
The first-line treatment for primary dysmenorrhea is nonsteroidal anti-inflammatory drugs (NSAIDs). While most NSAIDs inhibit both cyclooxygenase-1 (COX1) and COX2, the primary anti-inflammatory process is through COX2. COX1 is expressed in platelets.
chronic vaginitis – tx
When a patient has chronic vaginitis, vulvar care recommendations are the first treatment to utilize. The measures used in these treatments include avoiding scented products or other chemical wipes or gels on the sensitive areas of the body, using aqueous creams, avoiding washcloth use, gently drying the vulva with dabbing, avoiding tight-fitting pants, using white cotton underwear, avoiding excessive detergents, and wearing skirts or no underwear when at home or in bed to avoid friction.
Medicated creams are second line, and surgery should be used only as a last resort.
An 85-year-old G6P6 woman presents complaining of a vaginal bulge. She has stage IV prolapse and is no longer sexually active. The physician counsels her on an obliterative procedure, and she agrees. Which of the following will be performed?
When a patient presents with prolapse and says she desires surgical management, an obliterative procedure can be performed if she is no longer sexually active. The colpocleisis is an excellent surgical option in patients with multiple comorbidities, and where the concerns of being under anesthesia are great. A colpocleisis can be performed under regional and local anesthesia if necessary.
The procedure is as follows: A strip of the anterior and posterior vaginal wall is removed. Purse-string sutures are placed with a delayed absorbable material. The vagina can be divided into quadrants, and the first stitch to be tied is the one placed closest to the cervical os, followed by the second, third, and so on. In the Le Fort partial colpocleisis, the anterior and posterior vaginal walls are again removed. The edge of the anterior vaginal wall is sewn to the cut edge of the posterior vaginal wall. As the edges are sewn to each other, the vagina begins to invert on itself.
Answer C: McCall culdoplasty is performed at the time of a vaginal hysterectomy. The uterosacral ligaments are plicated at the midline. Vaginal coitus is still possible.
Bottom Line: Colpocleisis is a vaginal obliterative procedure that can be performed when prolapse is present in older women who no longer desire vaginal coitus.
Which of the following is TRUE regarding hemorrhagic strokes compared with ischemic strokes?
Cerebrovascular accidents are a common form of severe morbidity and even mortality, particularly hemorrhagic strokes.
In a study of more than 40,000 patients in Denmark, the 90-day unadjusted case fatality rate was 25% for hemorrhagic strokes compared with 10.9% for ischemic strokes. This is presumably due to the devastating nature of extravasated blood on the brain; and prompt diagnosis and drainage can be life-saving.
Answer A: Ischemic strokes were found to be more common in patients with a preexisting diagnosis of diabetes: a finding that has been consistently shown across many studies. Hypertension can be associated with either stroke type, but the cited study claims it was more likely to be associated with the hemorrhagic subtype.
Answer B: Hemorrhagic strokes tend to be more severe; in patients presenting with a “severe” stroke based on clinical manifestations, 30% had hemorrhagic stroke compared with 2% of patients with a “less severe” presentation.
Answer C: Hemorrhagic strokes are less common than ischemic strokes, with 89.9% of patients presenting with ischemic strokes and 10.1% presenting with hemorrhagic stokes.
Answer E: No differences have been found between males and females in terms of propensity to be diagnosed with a stroke, although women tend to be older than men at their diagnosis. This is likely because of the protective effect of endogenous estrogen on the vasculature, a phenomenon that does not translate into exogenous administration.
According to the latest USPSTF guidelines, at which age should routine colorectal screening begin?
Prior to October 2020, the U.S. Preventative Services Task Force (USPSTF) had recommended that colorectal cancer screening be initiated at age 50 for all average-risk adults, but begin earlier at age 45 years for Black adults due to the higher rates of cancer.
However, the latest USPSTF recommendations advise that routine screening for all adults begin at age 45 years for colorectal cancer regardless of risk factors, as nearly 94% of all new colorectal cancer cases are occurring in adults 45 years and older. While rates of colorectal cancer are higher in Black adults, the age to begin routine screening is not younger than the general population.
All women should be screened for colorectal cancer, regardless of family history. For women with an increased risk of colon cancer based on history, in general, screening is recommended to begin 10 years prior to the earliest familial colorectal cancer diagnosis or the routine recommended age, whichever is youngest.
Of the following, which is MOST related to pelvic congestion syndrome?
PCS is a disease characterized by pelvic pain for longer than 6 months and is described as a dull ache.
The pain has been reported as worsening premenstrually or with periods of prolonged standing, walking, or other activities that may increase intraabdominal pressure.
The patient often has generalized tenderness on exam.
Which of the following is the BEST management for a 49-year-old patient with debilitating fibroids and NYHA Class IV congestive heart failure, chronic obstructive pulmonary disease, a severe allergy to iodinated contrast material, and a BMI of 40?
Fibroids are the most common indication for hysterectomy in the United States, but surgery may not be feasible for every patient, particularly if multiple medical comorbidities make the patient a poor surgical candidate.
A patient with significant medical comorbidities or who is perimenopausal may benefit from gonadotropin-releasing hormone (GnRH) agonist therapy, such as leuprolide acetate. GnRH agonists are primarily used as either preoperative therapy (typically 3–6 months in duration) or as transitional therapy for patients in late perimenopause as they move to menopause. Good quality data exist that GnRH agonists decrease uterine and fibroid volume, increase hemoglobin, and improve perioperative outcomes in patients prior to undergoing hysterectomy and myomectomy.
Answer A: Abdominal myomectomy is a risky procedure in this patient given her multiple medical comorbidities. In addition, myomectomy is typically reserved for those who desire future childbearing.
Answer C: Observation is not appropriate for this patient who desires definitive therapy and is frustrated by receiving multiple transfusions without a cure being tendered. Although leuprolide acetate is not a cure, it is a better option than watchful waiting.
Answer D: A relative contraindication to robotic hysterectomy is elevated BMI (although this is debatable); however, the main reason this is incorrect is that it is a surgical procedure associated with severe morbidity and mortality, and this patient is a poor surgical candidate.
Answer E: Uterine artery embolization is an excellent option for women who are poor surgical candidates. Unfortunately, our patient has a severe allergy to iodinated contrast material, which is essential to performing the uterine artery embolization; therefore, it is not recommended.
Which of the following is TRUE regarding treatment of vulvar psoriasis?
Vulvar psoriasis usually does not result in the classic scaly appearance, and increased moisture and heat in the region may make it resemble candidiasis. Besides biopsy, the key to diagnosis is that psoriasis does not involve the vagina.
Treatment settles around the use of hydrocortisone for mild disease and prolonged topical steroids for moderate disease, with dermatology referral if no improvement is seen. Systemic steroids, although sometimes useful for psoriasis in other body areas, usually result in a rebound flare-up in the vulvar region.
Answer A: For moderate to severe disease, dermatology referral is appropriate after failure of prolonged topical steroid treatment, rather than on initial diagnosis.
Answer B: Four weeks of topical steroid therapy is the mainstay of treatment for refractory psoriasis that does not respond to conservative hydrocortisone treatment.
Answer C: Hydrocortisone 1% cream is used in the initial treatment course, rather than being reserved for refractory cases.
Answer D: As previously stated, psoriasis of the vulva does not include the vagina, which is necessary for the distinction between psoriasis and candidiasis.
Which of the following is a criterion for metabolic syndrome?
Metabolic syndrome has been used to describe the coexistence of risk factors for type 2 diabetes and cardiovascular disease, including abdominal obesity, hyperglycemia, dyslipidemia, and hypertension. The prevalence of metabolic syndrome among adolescents is estimated to be approximately 9%. Half of all adolescents initially classified with metabolic syndrome will no longer meet the criteria after 3 years; others will acquire the diagnosis.
Awareness of these facts is highly important in order to counsel and motivate your patient to assist in making better, healthier choices associated with more positive outcomes.
The recognition of metabolic syndrome as a distinct entity is to draw attention to the vastly increased morbidity associated with it.
Criteria for metabolic syndrome:
The diagnosis of metabolic syndrome requires 3 out of 5 of the following:
female waist circumference over 88 cm
elevated blood pressure greater than or equal to 130 systolic; 85 diastolic
triglyceride level greater than or equal to 150
HDL level less than or equal to 50
glucose level greater than or equal to 100 or previously established diabetes
Which of the following is the MOST accurate failure rate of ParaGard intrauterine device (IUD) in the first year of perfect use?
Contraceptive Method Typical use % Perfect use %
Copper IUD 0.8 0.6
Levonorgestrel IUD 0.2 0.2
Implant 0.05 0.05
Long-acting reversible contraceptives (LARC) are the most effective reversible contraceptive methods. They include intrauterine devices (IUDs) and contraceptive implants. The major advantage of LARC compared with other reversible contraceptive methods is that they do not require ongoing effort on the part of the patient for long-term and effective use. The return of fertility is usually rapid once the device is removed.
Typical use failure rate is described as the percentage of women who experience accidental pregnancy during the first year of initiating the use of a method of contraception. However, perfect use failure rate is described as the percentage of women who experience accidental pregnancy during the first year of initiating the use of a method of contraception when this method is used perfectly (consistently and correctly).
The failure rate of the ParaGard IUD is approximately 0.6% in the first year of perfect use. It has a reported failure rate of 0.8 per 100 women at 1 year of typical use.
A 35-year-old patient elects to use a GnRH agonist for treatment of her endometriosis. Which of the following is the MOST appropriate time frame to start add-back hormonal therapy?
The patient in the question stem has endometriosis. Endometriosis occurs in as many as 32% of patients with pelvic pain and as many as 50% of patients with infertility.
Although pain is not directly related to the amount of endometriosis, it is directly related to the depth of endometriosis and the involvement of neurologic structures. Inflammatory cytokines (specifically IL-1, which activates IL-8) are released along with TNF. These cytokines lead to activation of COX and release of prostaglandins, increasing uterine contractions and inflammation of the direct sites of disease. These occur in the background of an estrogen-laden environment.
Treatment for Endometriosis
NSAIDs are the most common first-line treatment
Combined oral contraceptive pills (OCPs) are the 2nd most common treatment, may be taken continuously to create a pseudopregnant state with constant estrogen level
GnRH agonist has superior efficacy to OCPs
Depot leuprolide may lead to osteoporotic symptoms, hot flashes, or other menopausal symptoms (ie, vaginal dryness, dyspareunia, decreased sexual desire, or pain)
To offset these menopausal symptoms, add-back therapy may be used immediately when taken with combined estrogen and progesterone. There is no evidence to support waiting until the patient is symptom-free before starting add-back therapy
Add-back therapy reduces or eliminates GnRH agonist-induced bone mineral loss and provides symptomatic relief without reducing the efficacy of pain relief
Add-back regimens (using either sex-steroid hormones or other specific bone-sparing agents) have been advocated for use in women undergoing long-term therapy (more than 6 months). Regimens include progestins alone, progestins and bisphosphonates, low-dose progestins, and estrogens. The FDA has approved the daily use of norethindrone 5 mg, as add-back therapy with a GnRH agonist. If women do not tolerate high-dose norethindrone, transdermal estradiol with medroxyprogesterone acetate can be used (transdermal estradiol 25 μg per day plus medroxyprogesterone acetate 2.5 mg orally daily), although this has not been approved by the FDA.
Which of the following is the MOST common single chromosomal abnormality detected in first-trimester miscarriages?
The most common single chromosomal abnormality is monosomy X, or Turner syndrome (also known as absent Barr body, 45,X, or XO), with an approximately 20% incidence. However, as a group, trisomies are the most common chromosomal abnormalities leading to first trimester miscarriage.
Trisomies AS A GROUP are the most common chromosomal abnormalities leading to miscarriage, and the most common trisomy is 16; however, this single abnormality accounts for only 13% of first-trimester miscarriages
On physical examination, the patient appears to have erythema with excoriations, skin thickening, and leathery sensation on palpation. Which of the following BEST describes the mechanism of actions for this patient’s disease?
Lichen simplex chronicus is a dermatologic disorder characterized by a chronic itch-scratch cycle which leads to chronic trauma secondary to rubbing and scratching. This disease is generally started when there is an exposure to a detergent, soap, or other agent such as heat. This irritation secondary to scratching leads to more excoriation within a background of erythema. In return, more scratching causes a lasting trauma to the skin. It can cause sleep disruption and symmetric skin changes.
The treatment of this disease involves the halting of the itch-scratch cycle generally eliminating stimuli. Steroid topical cream can be used to decrease inflammation. Lubricants or other oils and baths may be used to help in curing this disease. Oral antihistamines, trimming fingernails, and wearing cotton gloves at night may help as well. If there is no resolution in 1 to 3 weeks, a biopsy may be needed.
The usual pathologic finding on biopsy is thickening of the epidermis and stratum corneum.